Viva Stems Flashcards
“The Emergency Department registrar calls you about a 45 year old man who presents with a 24 hour history of sore throat and painful swallowing after a meal out with friends. He suspects epiglottitis and has already called the ENT surgeon who is on his way in to the hospital. He would like your assistance in making the diagnosis and managing the patient.
Initial Observations: Sitting up, no noisy breathing, Mallampati 2, thyromental distance > 6.5cm, full beard, own teeth
Respiratory Rate: 20 breaths per minute
SpO2 95% on 2l/min O2
Chest is clear on auscultation
Heart Rate 110 Sinus Rhythm
BP 160/85 mmHg
He is alert
Temperature: 38.9oC
Fasting status: Not eaten for 4 hours
How do you interpret this information?”
An otherwise healthy 50 year old man was admitted to the Intensive Care Unit 24 hours ago, after multi level cervical fusion. The anaesthetist noted that he was difficult to intubate and he is now being considered for extubation. The anaesthetist recorded a grade IV Cormack and Lehane view at laryngoscopy on the anaesthetic record and that intubation had been achieved with the aid of a gum elastic bougie. Pre-operatively his Mallampati score was a grade III.
The aims of the viva are to assess whether the candidate can:
formulate a plan for safe extubation
manage a difficult intubation after failed extubation
diagnose and treat bilateral tension pneumothoraces from barotrauma during resuscitation
An 18-year-old man has been brought into the Accident & Emergency Department by ambulance. He has attempted suicide by hanging. His Glasgow Coma Scale score is 5 and he is being ventilated via a laryngeal mask which was inserted at the scene by the ambulance paramedic. You have been called to provide airway management in the acute phase. Outline how you would assess this patient’s airway.
A 45-year-old policeman attending a domestic dispute was punched in the throat. He presents to ED with a severe pain in his throat and a hoarse voice. You are called to provide emergency airway assistance if required. How are you going to assess his airway?
A 30-year-old man remains intubated and ventilated following complications during the drainage of an acute dental abscess at another hospital. Great difficulty was experienced with intubation resulting in a hypoxic cardiac arrest and possible airway soiling. The airway was secured at that time by fibreoptic nasotracheal intubation. Following surgery, the decision was made to electively ventilate the patient overnight via nasal endotracheal tube. Intensive Care requests your assistance with extubation. What is your plan of management for extubation?
A 70-year-old man is having radiotherapy for carcinoma of the tongue, and has been admitted to ICU with breathing difficulties. He has mucositis and neck swelling. He is unable to lie flat and is receiving oxygen via a CPAP mask. The intensivist has asked you to secure his airway. The ear, nose and throat surgeon is reluctant to perform awake tracheostomy. The patient has a history of cardiac disease with previous myocardial infarction and coronary stenting. He is on aspirin and clopidogrel. How will you assess and manage this man’s airway?
An otherwise well 38-year-old woman presents to the Emergency Department with gradually worsening exertional stridor over several weeks. There is no stridor at rest. At nasendoscopy she is found to have a large papilloma involving the posterior laryngeal inlet. The ENT surgeon wishes to treat this with CO₂ laser therapy under general anaesthesia. The diameter of the laryngeal inlet was estimated during nasendoscopy as 6 mm. Recent hospital records indicated that there have been no previous difficulties with laryngoscopy or endotracheal intubation. The patient has never smoked and does not take any medications. Please describe your anaesthetic management plan for this patient.
A 22-year-old male with developmental delay and epilepsy sustained facial trauma after falling down several steps onto a concrete path during a seizure 3 hours ago. At a previous anaesthetic, 6 years earlier, he was documented to be a ‘Grade 3 view’ at direct laryngoscopy. He is currently in the holding bay awaiting surgical intervention for his ongoing facial bleeding. You are called urgently to the holding bay as he is having a generalised tonic clonic seizure. How are you going to manage this situation?
You are called to attend a 60-year-old man who has presented to your Emergency Department with breathing difficulty that has worsened over the last four hours. He has known carcinoma of the tongue and had been scheduled for hemiglossectomy and radical neck dissection in one week’s time. His only known past history is heavy alcohol consumption and 40 pack-years of cigarette smoking. Describe your airway management plan.
It is 1100 hours on Saturday morning and you are working in a large metropolitan hospital. You have just finished a case when you are phoned by the registrar in the high dependency unit. An 84-year-old man who underwent neck dissection for melanoma two days ago is now in respiratory distress. A large right-sided neck swelling has developed over the last two hours. His SpO2 is 97% on 15 l/minute of oxygen; his blood pressure is 140/80 mmHg and his heart rate is 95 /minute. He has a background history of atrial fibrillation and transient ischaemic attacks, for which he takes dabigatran, 110 mg twice daily, as his only medication. Dabigatran was ceased two days preoperatively and recommenced orally three hours ago. He was bridged with enoxaparin; the last dose of enoxaparin 1.5 mg/kg was 12 hours ago. You have two minutes to consider the case as you proceed to the high dependency unit. What further key information do you want to obtain once you get to the bedside?
“You are the on-call consultant anaesthetist in a tertiary hospital. You are called by your advanced anaesthetic trainee to the Emergency Department to assist with the airway management of a 56-year-old woman. She has a three-month history of progressive dyspnoea, with acute deterioration in the past 12 hours requiring ambulance transfer to hospital. A diagnosis of asthma was recently made, but she has had no significant response to medical treatment. She has a known history of a previous thyroid tumour. There are no known allergies, and her only medication is salbutamol prn.
The Emergency Department registrar has noted a large thyroid mass on examination. When you arrive, it is apparent that the patient is tiring and clearly distressed. Her chest X-ray is shown here. Outline your assessment of this patient’s dyspnoea.”
“Your junior anaesthetic registrar calls you at home at midnight from the Emergency Department. He has been requested to assist with the intubation of a 72-year-old woman who has presented with severe epistaxis. Her medical history includes mechanical aortic valve replacement, hypertension, and past
cerebrovascular accident.
Her current medications are:
Warfarin 5 mg daily
Metoprolol 12.5 mg bd
Perindopril 4 mg bd
Amlodipine 10 mg daily
Your registrar informs you that preparations are underway to intubate this patient. How will you respond to his phone call?”
You are in the pre-anaesthetic assessment clinic. A 62-year-old man is scheduled for microlaryngoscopy and excision of vocal cord lesions. The patient has been referred to the clinic because of a history of difficult intubation during elective carotid endarterectomy under general anaesthesia five years ago. The surgery is planned for next week. A picture of the patient is shown here. What are the key issues in his airway assessment?
A 35-year old man is booked on the emergency theatre list for drainage of a submandibular abscess. The patient is homeless, a known intravenous drug user and seems unwell. You meet him in the holding bay area outside the operating theatre suite. What are the main issues that worry you?
You are today’s Team Leader of the hospital Medical Emergency Response Team. You have been called to assess a 33-year-old woman in the surgical ward, who had a mandibular molar abscess drained 36 hours ago. She is 160 kg and 170 cm tall. She is confused and refusing any oxygen therapy. Pulse oximetry erratically reads 84% and a heart rate of 110 bpm. How will you respond to this situation?
There has been a brawl at a nearby prison. One of the inmates is transferred to your hospital for treatment. He is a 24-year-old man with a mandibular fracture, and a background history of hepatitis C, smoking and intravenous drug use. You are seeing him in the waiting area immediately before surgery. What features on examination of this patient would suggest a difficult airway?
You are the consultant on call at a tertiary referral hospital on a Sunday afternoon. You receive a call from the intensive care unit registrar asking for assistance in managing the airway of one of their patients. The patient is a 20-year-old apprentice chef who suffered facial and neck burns this morning after a deep fryer caught fire at work. She was intubated on admission, and this afternoon the pilot tube on her endotracheal tube was cut whilst dressings were being changed. What is your initial response?
“You are the duty consultant anaesthetist at a tertiary hospital. You are called to come immediately to the intensive care unit (ICU) to help other medical staff struggling with the airway management of a patient with a tracheostomy.
On arrival you see:
* the ICU consultant attempting to insert a laryngeal mask airway (LMA)
* an ear, nose and throat (ENT) surgical registrar attempting to replace the tracheostomy tube
* oxygen saturation SpO2 86%
* heart rate 122 beats per minute
* blood pressure 195 / 115 mmHg
The ICU consultant tells you this is a 55-year-old man weighing 165kg who has community- acquired pneumonia. The patient had a surgical tracheostomy three days ago. The tracheostomy tube has now become dislodged while turning the patient. What are your priorities?”
“A 56-year-old man with supraglottitis required a tracheostomy under local anaesthetic on presentation to hospital three days ago. Since that time, he has been in the Intensive Care Unit receiving antibiotics and dexamethasone. Overnight the anaesthetic registrar was asked to see the man regarding some minor neck swelling. This was managed conservatively. This morning the man has been restless and has completely dislodged his tracheostomy. You are called as the duty anaesthetist to assist the junior ICU registrar. As you arrive into the ICU bay this is how the patient appears: He is moving around the bed trying to get himself comfortable. His observations as they appear on the
ICU monitor are:
RR 26
SaO2 90% on Oxygen 10l/min
BP 100/52 mmHg
HR 125 bpm
Picture of significant subcutaneous emphysema
Outline your initial assessment and urgent management”
You are called to the emergency room of a level 1 trauma centre as part of a trauma team call. A convoy of 15 motorcyclists was struck by a truck. There was a fire at the scene and 2 people have died on site. Your centre is receiving 3 patients. You are assigned as the airway doctor for 1 of the patients arriving by road ambulance. Additional information provided by paramedics: He is a 48-year-old male, weight 123 kg and height around 174 cm. He was wearing an open-faced helmet, removed at the scene. He has a partially singed beard with bruising and swelling of his midface. He is vocalising with no obvious stridor or hoarseness. What are your specific airway concerns?
As the duty anaesthetist receiving emergency bookings, you take a phone call from the ENT registrar, who is in the Emergency Department with a 26-year-old man requiring urgent surgical drainage of a suspected peritonsillar abscess. The registrar reports that the patient is very distressed, refuses to lie flat and cannot open his mouth more than 1.5cm. Outline your initial management, assessment and concerns for anaesthesia.
“You review a 67-year-old man in the Preadmission Clinic (PAC) for a left hemihepatectomy via
a roof- top incision for metastatic colon cancer.
He underwent a right hemicolectomy four months ago for primary cancer resection. The
procedure was complicated by an extended stay in hospital due to suboptimal pain
management.
Past medical history:
* Hypertension
* Lower limb peripheral neuropathy secondary to chemotherapy
* Ex-smoker with 30 pack-year history
Medications & allergies
* perindopril 5 mg daily
* amitriptyline 25 mg nocte
Nil known allergies
Observations in PAC
BP - 160/85 mmHg
HR- 75 bpm
SpO2 - 98% on room air
weight - 65 kg
height - 180 cm
BMI – 20 kg/m2
Liver function test results (table in PDF).
Outline your concerns regarding the patient’s fitness for surgery.”
“A 50-year-old woman has been transferred to your tertiary referral centre for a total thyroidectomy for a massive goitre with associated recent voice change. You review her on the ward as she is booked on your list for the following day.
Medications:
carbimazole 15 mg bd
propranolol 40 mg bd
rosuvastatin 10 mg daily
Weight 120 kg
Height 165 cm
BMI 44 kg/m2
How will you assess her airway preoperatively?”
You are asked to review a 40-year-old man in the emergency department who has trismus secondary to a dental abscess. He has severe intellectual impairment and is nonverbal. His caregiver is with him. The maxillofacial surgeon wishes to bring him to theatre urgently to remove a lower molar tooth and incise and drain his submandibular abscess. How will you assess this man’s airway?
You are asked to help a colleague with a patient that has failed to wake after a below knee amputation operation. Describe your approach
“You review a 55 year old woman in the preadmission clinic for a total abdominal hysterectomy. She has a history of chronic lower back pain for which she takes regular paracetamol and naproxen. She is otherwise well. She has a history of postoperative nausea and vomiting. formulate a plan to minimise postoperative pain and PONV
diagnose and manage postoperative leg pain from a nerve injury from regional blockade
understand the principles of management of persistent neuropathic pain in this context”
You are supervising a junior trainee. The next patient on your list is a 30-year-old booked for laparoscopy for investigation of infertility. She is in the induction room, extremely anxious and requests sedation. You ask the trainee to obtain venous access and administer midazolam while you transfer the previous patient to Recovery. When you return to the induction room, the patient is alone, unresponsive, apnoeic and cyanosed. What do you think has happened and what will your management be?
An obese 68-year-old man requires craniotomy for debulking of a tumour. Prior to induction you placed an 18G intravenous cannula in the left antecubital fossa and a 20G arterial cannula in the right radial artery. You have induced anaesthesia using total intravenous anaesthesia with propofol and remifentanil, with rocuronium as the muscle relaxant. When positioning the patient for surgery, you notice that the left hand is pale, blanched and oedematous. You suspect that the cannula may be in the artery. How will you determine whether the cannula is in the artery or in a vein?
“You are asked to assess a 72-year-old lady who presents to your local Emergency Department with a buttock abscess for surgical drainage. She has a background history of:
Type II diabetes mellitus requiring insulin therapy
Ischaemic heart disease
Hyperlipidaemia
In the Emergency Department, the patient asks you whether the drugs used for anaesthesia will cause memory problems after surgery. What is your response?”
A 42 year old intellectually impaired woman presents for elective hysterectomy for menorrhagia due to endometriosis. She is accompanied by her mother who says she hates needles and had to be held down by 4 people to have her full blood count taken. She is 150cm and weighs 85kg. How will you decide whether an inhalational induction is appropriate?
You are the duty anaesthetist at a major metropolitan hospital, and are urgently called to the computed tomography (CT) scanner in the radiology suite. As you enter the room you are asked by the emergency department registrar to provide some sedation for a 32-year-old man in a Philadelphia collar, who is having difficulty lying still for a CT scan. The radiology staff inform you that they are behind schedule and want to get this scan completed as soon as possible. How will you manage this situation?
“A 72-year-old man presents with his daughter to your pre-anaesthetic assessment clinic for assessment for a right total knee replacement at a major regional hospital. His daughter informs you that he has had:
A cerebrovascular accident with mild right-sided weakness and altered speech
An isolated episode of delirium of unknown aetiology two years ago
Ischaemic heart disease with an anterior myocardial infarction six years ago and subsequent
cardiac stenting
Hypertension
Atrial fibrillation
Severe osteoarthritis of his knees and cervical and lumbar spine
His medications are:
Warfarin
Losartan
Aspirin
Paracetamol
Diltiazem
Tramadol
Digoxin
Atorvastatin
This gentleman is most concerned about further deterioration in his neurological status. What advice can you give him about adverse neurological outcomes from surgery?”
You are the emergency anaesthetist working in a tertiary hospital. You are called by a nurse on behalf of the respiratory physician working in the bronchoscopy suite. There is a 48-year-old woman who is having a bronchoscopy as investigation for possible lung cancer. The respiratory physician has started the procedure but the patient has deteriorated acutely and her oxygen saturation is 80% and they are requesting your urgent assistance. Describe how you will immediately assess this situation on arrival in the bronchoscopy suite?
You are the duty anaesthetist and you are called to the endoscopy suite located two floors above the main operating suite in your hospital. You are requested to assist a proceduralist and two nurses with a patient who is becoming restless and uncooperative during a gastroscopy. The patient is 36 years of age and has ankylosing spondylitis. You are told the patient had a small recent upper gastrointestinal bleed that settled spontaneously. Current observations are blood pressure 88/51 mmHg, and SpO2 92% on three litres per minute of oxygen via nasal prongs. Venous access is a 22G cannula in the anterior cubital fossa and the patient is moving his arms about. Please outline your assessment and initial management.
“A 52-year-old woman with end-stage kidney disease (ESKD) presents to your pre-admission clinic prior to an iliac artery aneurysm repair. The procedure is being performed to enable a live-donor kidney transplant to be undertaken in three months’ time. The ESKD is secondary to long-standing type 2 diabetes. In addition the patient has an autonomic neuropathy secondary to diabetes and hypertension.
Medications:
Candesartan 16 mg daily
Dapagliflozin 10 mg daily
Long-acting insulin 16U nocte
Gabapentin 200 mg bd
Relevant investigations: haemoglobin 90 g/l; creatinine 585 µmol/l.
The patient describes a history of anaphylaxis during a prior anaesthetic. She has been to an allergy clinic and had skin testing. The patient has a letter stating she is allergic to oxycodone. The patient is concerned about having another anaphylactic reaction during the upcoming procedure and whether this would impact upon her ability to have a kidney transplant. What would you tell the patient?”
You are running the emergency theatre in a regional hospital on a Sunday morning when you receive a phone call from the emergency department (ED) consultant. A 23-year-old man with an intellectual disability has presented with a productive cough and haemoptysis after a choking episode the previous evening whilst eating dinner. The ED consultant asks you to provide assistance with sedation for a CT chest due to the patient’s non-compliance with lying still. What would you like to know from the ED consultant when she calls you?
You are asked to review the pain management of a 65 year old male inpatient with multiple myeloma. He has been taking oral morphine 90mg/day for widespread bone pain for almost a year. Now he has severe pain from oral mucositis (painful inflammation and ulceration of the mucous membranes lining the mouth). The patient has developed this after receiving high dose immunosuppression, followed by a bone marrow transplant 10 days ago. He has also developed acute graft-versus-host disease. What are the pain management issues with this man?
“A 73 year old man with metastatic prostate cancer had a minor fall at home. He is admitted with three fractured ribs of his lower left chest.
Past history: Hypertension
Smoker 15/day
Current medications include:
oxycodone controlled release 80mg bd (Oxycontin)
perindopril 5mg daily (Coversyl)
goserelin 3.6mg SC monthly (Zoladex)”
manage acute on chronic pain
prevent, diagnose and manage respiratory failure in this setting
negotiate cessation of active treatment measures with a family who disagree
During the Acute Pain ward round you are asked to review a 68-year-old man. He is day 3 following a laparoscopic right hemicolectomy for cancer. On Day 1 he was managing well on oral analgesia, however late on post op Day 2 he developed increasing abdominal pain. He was seen yesterday evening by the anaesthesia registrar who charted a morphine PCA (1mg bolus and 5 minute lockout). He is now drowsy but more comfortable, but he still finds moving or coughing very painful. His past medical history is of hypertension for which he takes Lisinopril, his pre operative BP was 150/85mmHg. How would you assess this patient’s pain?
You are called by the emergency department to assist in the management of a 62-year-old woman who has an isolated chest injury after a fall at home. She has sustained multiple right sided rib fractures. She has received 15 mg of intravenous morphine in the emergency department. Her comorbidities include Chronic obstructive airways disease and ischaemic heart disease. Her medications are aspirin, perindopril, salbutamol and budesonide. The emergency physician asks for your advice with regard to her ongoing pain management. How will you assess the adequacy of her pain management?
You are called by the emergency department registrar to help with the management of a 33- year-old man who has just presented, shouting about extreme pain in his left hand. He is on the methadone maintenance program and has accidently injected a solution made with crushed oxycodone tablets into his brachial artery. His hand is pale with mottled fingertips. He is restless, agitated, and behaving in a threatening manner to the staff, who have been unable to perform any interventions. Describe how you would approach this patient, and your thoughts on initial pain management.
“A 56-year-old woman with a body mass index of 45 and known metastatic ovarian cancer has presented to the emergency department of a peripheral hospital with severe pain in her right thigh following a seemingly trivial injury. You have been asked to assist with her pain management. The X-ray of her lower limb is shown.
Her usual medications are: morphine (sustained release) 60 mg tds tramadol 100 mg qid morphine syrup 30 mg q4h prn (for breakthrough pain) How will you manage this patient?”
“You see a 45-year-old man who is on your operating list for a four-level lumbar laminectomy and fusion next week.
His medical history includes:
Body mass index 42 kg/m2
Obstructive sleep apnoea
Type 2 diabetes mellitus
Hypertension
Medications:
Metformin 1 g bd
Pregabalin 75 mg bd
Oxycodone (slow release) 20 mg bd
Oxycodone (immediate release) 5–10 mg prn
Blood results from one week ago:
Full blood count, electrolytes, urea and creatinine, coagulation studies – normal
Blood glucose level 14 mmol/l (fasting reference range 4–7 mmol/l)
Preoperative observations:
Blood pressure 160/95 mmHg
Heart rate 70 /minute
SpO2 97% on room air
Describe your plan for perioperative analgesia in this patient.”
You are asked to help with the management of a 28-year-old male intravenous drug user, who is on methadone 200 mg daily, and has presented to the emergency department in severe pain with a wound to his neck sustained in a knife fight. On entering the emergency department you can hear the patient shouting at the nurse that he needs more fentanyl. The emergency department registrar has given 200 micrograms of fentanyl and is worried about giving more. How do you approach this situation?
You have commenced your shift this morning as the in-charge consultant anaesthetist in a regional hospital where you are also responsible for the intensive care unit (ICU). In the ICU, there is a 40-year-old man with isolated bilateral femoral fractures who was admitted 20 hours ago and has been booked for surgery later today. He is otherwise well with no co-morbidities or other injuries. You have been asked to review the patient as his pain is poorly controlled despite having been prescribed an intravenous morphine patient-controlled analgesia regimen. Your nursing staff also tell you that the patient is becoming increasingly confused. What are the possible reasons for this patient’s poor pain control?
You review a 42-year-old woman in the preanaesthesia clinic who is booked for left mastectomy and axillary clearance for breast cancer. She has a history of nausea with opioid analgesics and experienced severe postoperative nausea and vomiting after a previous hysteroscopic fibroid resection. What is your plan for anaesthesia and postoperative analgesia?
“A 55-year-old woman is admitted from the emergency department for management of severe low back pain. She has a history of chronic back pain and had an epidural steroid injection one week ago which has not improved her symptoms. She weighs 87 kg, BMI 31 kg/m2
Current medications:
* empagliflozin 10 mg daily
* buprenorphine patch 15 mcg/hr transdermally
* fluoxetine 20 mg daily
* gabapentin 300 mg twice daily
* metformin 1 g twice daily
* paracetamol 500mg + codeine phosphate 30 mg, 1-2 tabs 4-hourly PRN
* perindopril 10 mg daily
As a member of the acute pain service, you are asked to see her and advise on the best management of her pain. How will you assess her?”
“You are the specialist involved with the preadmission clinic. Mrs Smith is a 52 yr old patient who presents for a laparosopic sleeve gastrectomy for weight loss She is 150cm tall and weighs 126 kg (BMI 56kg/m2) and has hypertension, mild asthma and arthritis of her knees. Medications: cilazipril 2.5mg daily and salbutamol inhaler PRN. She is known to have an allergy to amide local anaesthetics (lignocaine, bupivacaine and prilocaine)
Preadmission observations:
Pulse 90 beats per minute
BP 160/85 mmHg
SpO2 92%
How would you classify Mrs Smith’s obesity and which other co-morbidities would you assess in conjunction with this?”
A 30 year old patient with Cystic Fibrosis presents for an Endoscopic Retrograde Cholangio- Pancreaticogram (ERCP) following an episode of acute pancreatitis due to a common bile duct\ stone. He has been treated in hospital for 5 days with intravenous fluids and has persistent abdominal pain. How will you assess this patient prior to his procedure?
An 80 year old man is scheduled for removal of infected aortic graft (previous infra-renal abdominal aortic aneursym repair) and axillo-bifemoral bypass on your list. You are asked to do a preoperative assessment for this patient in the ward 2 days before surgery. He has a known history of diabetes and has been treated with insulin for the past 20 years.
A 65 year old 65 kg woman with diverticular disease attends your preanaesthetic assessment clinic prior to her open hemicolectomy. She is a retired health care worker with long standing chronic obstructive airways disease and allergic rhinitis.
Two weeks after a percutaneous coronary intervention (PCI) a 60 year old man is brought into the Emergency Department by ambulance after haematemsis and melaena at home. The gastroenterologists are keen to do an urgent endoscopy to manage the bleeding. The patient has a past history of diabetes and hypertension.
You are a consultant in a major regional hospital. A 45-year-old man is booked for elective laparoscopic cholecystectomy. He underwent a heart transplant two years ago. Outline your preoperative assessment of this patient’s cardiac function.
A 49-year-old morbidly obese male (weight 135 kilograms, height 172 centimetres, body mass index 46 kilograms/metre2) is booked for a lower lumbar laminectomy. You are reviewing him in Pre-Anaesthesia Clinic. He has severe obstructive sleep apnoea responsive to CPAP mask overnight. He is on long-term oxycodone 80 mg bd for back pain. How does the combination of obesity and obstructive sleep apnoea alter your anaesthetic and postoperative care of this patient?
You are asked to assess a 50-year-old man in the preoperative clinic. He is proposed for open extended left hepatectomy for hepatocellular carcinoma. These blood test results posted on the wall to your right are available to you before you assess the patient. What are the important aspects of the preoperative consultation?
“A 40-year-old woman is booked for elective laparoscopic repair of her umbilical hernia, as a ‘day case’. She underwent bilateral lung transplantation (both lungs at once) two years ago.
Current medications are
prednisolone 5mg daily
mycophenolate 1 gm daily
cyclosporine 110 mg twice daily
omeprazole 20 mg twice daily
Her body mass index (BMI) is 30. She has had recent spirometry to assess her lung function. What are your key concerns in the preoperative assessment of this patient?”
The surgical consultant has scheduled a 90-year-old lady for left hepatectomy. She has a single liver metastasis from previously treated bowel carcinoma and there is an advanced health directive limiting postoperative care options. Her relatives are keen to proceed including her daughter who is with her today. She has a past history of hypertension and type 2 diabetes that is diet-controlled. Her blood pressure is 145/85 mmHg, heart 68 beats per minute and she weighs 65kg. She arrives at your pre-anaesthetic assessment clinic. Outline the main issues you need to address in your assessment before you will be happy to anaesthetise her for this procedure.
A 56-year-old man with end-stage renal failure presents with a blocked arteriovenous fistula in his left arm (pictured). The vascular surgeon wishes to revise the fistula. It has been three days since his last haemodialysis; he is short of breath and has obvious peripheral oedema. Please comment on his ECG (pictured - not provided).
“An 82-year-old man has been brought to the Emergency Department twenty-four hours after a fall. He has a reduced level of consciousness and left-sided hemiparesis. His background includes atrial fibrillation and transient ischaemic attacks. His
medications include digoxin and warfarin. His International Normalised Ratio (INR) is 5. A representative slice of his CT scan is pictured. Please comment on the scan and outline the management priorities in this man.”
A 70-year-old woman is booked for removal of a left cataract and insertion of intraocular lens. She has a history of obstructive sleep apnoea, but does not use a CPAP mask due to claustrophobia. She had a coronary artery stent inserted two years previously and is taking aspirin and clopidogrel. She demands a general anaesthetic for her surgery. She has no home support. How would you assess the severity of her obstructive sleep apnoea?
“A 45-year-old male presents to the Pre-Admission Clinic for anaesthetic assessment prior to elective laparoscopic cholecystectomy. He has previously undergone laparoscopic gastric banding, but still weighs 130 kg and his BMI is 40. He is housebound and undertakes only minimal exercise. He has diabetes mellitus, smokes 35 cigarettes and consumes at least 6 standard drinks per day.His pulse oximetry is 93% on room air.
He is taking the following medications:
atorvastatin 80 mg daily
telmisartan 80 mg daily
metformin 500 mg three times daily
gliclazide 60 mg morning
glargine insulin 28 units SC morning
soluble insulin 4 – 8 units SC PRN
aspirin 100 mg daily
esomeprazole 40 mg dailly
tiotropium inhalational 16 mg daily
fluticasone/salmeterol 250 mg/25 mg - 2 puffs twice daily
salbutamol inhaler 2 puffs 4 hourly PRN
What are the key issues when assessing this man for anaesthesia?”
A 75-year-old male is on your list for full revision of total hip replacement. He presents with a history of rheumatoid arthritis for which he is taking prednisone and methotrexate. What issues do you need to cover in your preoperative assessment?
“A 70-year-old man presents to Pre-Admission Clinic for anaesthetic assessment. He has a history of chemotherapy-induced cardiomyopathy (ejection fraction 25%) and impaired renal function secondary to the treatment of Hodgkin’s lymphoma three years ago. He is currently in remission. His regular medications include:
Enalapril
Frusemide
Potassium supplement
Carvedilol
He has a bleeding carcinoma of the rectum and is scheduled for laparoscopic anterior resection. The patient’s chest X-ray is shown below. (showing cardiomegaly) How would you assess his cardiac status prior to his proposed surgery?”
A 56-year-old woman with metastatic melanoma is scheduled for prophylactic intramedullary nailing of the right femur. She is on a high-dose fentanyl patch to control the pain from her widespread disease. At your preoperative consultation she is in obvious respiratory distress, tachypnoeic and dyspnoeic. Outline your preoperative respiratory assessment and management
A 35-year-old intravenous drug user has injected into her brachial artery near her elbow. Her arm is ischaemic and she requires thrombectomy and probable fasciectomies. She has asthma and a chest infection with sputum and a cough. How will you anaesthetise her for the proposed operation?
“A 62-year-old man with Parkinson’s disease is scheduled for total knee replacement. His medications include
- carbidopa/ levodopa 25/100 mg
- benztropine
- paracetamol
- tramadol
- naproxen.
The patient and his family have expressed concern about the potential effect of anaesthesia on his Parkinsonism. What advice will you give them?”
“You are the anaesthetist assigned to an orthopaedic trauma list. An 18-year-old woman is brought to your induction room with a fractured ankle. She has a history of anorexia nervosa and is extremely anxious and upset. The surgeon tells you that the foot is neurovascularly compromised and she needs to proceed urgently to surgery and may require intraoperative angiography.
Her medications include:
- fluoxetine 20 mg daily
- omeprazole 20 mg daily
- laxatives (coloxyl with senna and sorbitol) prn
- frusemide prn.
How will her history of anorexia influence your preoperative assessment?”
You go to review a 60-year-old man on the ward booked for rigid bronchoscopy and laser treatment to a right main bronchus lesion on tomorrow’s thoracic list. He is an ex-smoker with a twelve week history of breathlessness. His investigations are shown below. [CT, SPIROMETRY, ABG’s] Please assess his test results.
A 50-year-old man presents for elective pulmonary vein isolation and radiofrequency ablation for atrial fibrillation. His past history includes hypertension and paroxysmal atrial fibrillation. Outline your preoperative cardiovascular assessment of this man.
“A 68-year-old obese woman with a body mass index of 36 is booked to undergo laparoscopic cholecystectomy for recurrent biliary colic. She has known pulmonary artery hypertension, secondary to recurrent pulmonary emboli, and has an inferior vena cava filter in situ as well as being anticoagulated.
Her current medications include: Sildenafil 50 mg tds
Dabigatran 150 mg bd
Diltiazem (slow release) 240 mg daily
Spironolactone 25 mg tds
Frusemide 40 mg bd
She is under the care of a heart failure specialist in the tertiary centre where you work. What features would you seek on preoperative assessment that would indicate a high risk of perioperative mortality for this patient?”
“You are called to assess a 75-year-old lady in the Emergency Department. She has fractured the shaft of her femur and is due for surgery later that day. She has a past history of stable angina and colon cancer. She is known to have two liver metastases as well as bony metastases to the femur, pelvis and spine.Her current medications are: Atenolol 50 mg daily
Irbesartan 150 mg / hydrochlorothiazide 12.5 mg daily
Morphine sulphate (oral) 30 mg bd (plus oral morphine mixture prn for breakthrough pain)
Aspirin 100 mg daily
Paracetamol prn
Coloxyl with senna prn
What are the important issues that you need to address when assessing this lady?”
“You are asked to review a 62-year-old man with Paget’s disease who requires intramedullary nailing of his femoral fracture. The fall was from a simple trip. He had no loss of consciousness, but he lay on the floor for eight hours after his fall until he was found. He has a history of ischaemic heart disease, and his current medications are:
Quinapril 20mg daily
Amlodipine 5mg daily
Atorvastatin 20mg daily
Nitroglycerin lingual spray PRN
Aspirin 100mg daily
Alendronate 40mg daily
Celecoxib 200mg daily
Buprenorphine 20 mg transdermal patch
His vital signs are:
HR 105 /minute
BP 104/76 mmHg
SaO2 98% on 6 l/min oxygen via Hudson mask .
He is alert and conscious, but in pain, and the surgeons are keen to take him to theatre as soon as possible. What are the most relevant clinical issues in this patient?”
You are the Senior Anaesthetist in Charge of the Operating Suite at a rural base hospital. One of your registrars asks you for advice regarding a patient on his list. He is a 52-year-old man,15 years post heart transplant, booked for redo carpal tunnel surgery. The patient has been booked as a day-case. Discussion with the surgeon reveals that local anaesthetic infiltration and sedation is not an option. Describe what the registrar should do to assess this patient.
“A 72-year-old woman, with a long history of rheumatoid arthritis, presents to the Pre- Admission Clinic for anaesthetic assessment. She has a family history of bowel cancer and is scheduled for a screening colonoscopy. She is on methotrexate 10 mg orally weekly, folic acid 5mg weekly and has intermittently been on prednisolone 15 mg daily over many years. She also takes cilazapril for hypertension. Her Body Mass Index (BMI) is 23.
Her vital signs are:
HR 85 /minute
BP 145/90 mmHg
SaO2 96% on room air
In relation to her rheumatoid arthritis, what will you be looking for in your preoperative assessment?”
A 68-year-old man has a left forearm arterio-venous fistula for haemodialysis, which has become blocked. He now requires a vascath to be inserted. It is 3 days since his last haemodialysis. He has longstanding Insulin Dependent Diabetes Mellitus (IDDM), hypertension and end stage renal failure. He was also recently diagnosed with moderate obstructive sleep apnoea (OSA) by sleep studies. You are asked to assist with the insertion of a vascath by the radiologist in the Radiology Suite. He doubts that the patient will lie still during the procedure. This chest X-ray was taken 3 hours ago. Please comment on the CXR (not provided).
An 82-year-old woman is scheduled for a dynamic hip screw (DHS). She is on warfarin for atrial fibrillation. Her International Normalised Ratio (INR) is 4.2. When you see her the night before surgery, she is concerned that she will have a stroke if you stop her warfarin peri-operatively. How will you address this patient’s concern?
A 54-year-old woman with carcinoma of the colon is admitted for elective laparoscopic- assisted right hemicolectomy, which is scheduled for the following day. She had a single lung transplant for chronic obstructive pulmonary disease (COPD) 4 years ago. What aspects of her history and physical examination, with particular reference to her lung transplant, are relevant to your preoperative assessment?
“The neurosurgeon contacts you at 0600 to inform you that a 75-year-old man is booked for an occipital craniotomy today for evacuation of haematoma and resection of tumour. The patient was admitted overnight following a grand mal seizure. The patient was given a loading dose of Phenytoin 750 mg IV at the time. The patient is now intubated and ventilated. Previous medical history: Paroxysmal atrial fibrillation, ischaemic heart disease (bare metal stent 2011), type II diabetes, transient ischaemic attack 2012 & hypertension.
Current medications:
Dabigatran 150 mg bd
Metformin 500 mg bd
Candesartan 8 mg mane
Amiodarone 150 mg mane
Aspirin 150 mg mane
Phenytoin 100 mg tds; (last dose of dabigatran given @ 0800 the day before)
Investigations available:
Urea & electrolytes: Na 139 meq/l, K 4.2 meq/l, Cr 125; Cr clearance 50 ml/min/m2 FBC: Hb 110 plt 200
How would you (i) assess and (ii) manage this patient’s anticoagulation in the perioperative period?”
A 45-year-old male patient with Amyotrophic Lateral Sclerosis (Motor Neuron Disease) has sustained a fracture dislocation of the ankle. How would you assess the severity of this patient’s neuromuscular disease? How would you assess the severity of this patient’s neuromuscular disease?
“A 48-year-old lady presents for a Left Mastectomy and Axillary Clearance for a biopsy proven triple negative intraductal adenocarcinoma with palpable lymph nodes. She weighs 99kg and is 155cm tall (BMI 41). She has a lifelong history of asthma and smokes 20 cigarettes a day. Her past history includes a Motor Vehicle Accident 10 years ago where she sustained pelvic fractures and chronic hip pain. Her current medications are: Prednisone 5mg mane
Serotide 2 puffs bd
Ventolin 5 mg nebulized bd
Baclofen 20mg tds
Gabapentin 900mg/day
Movicol bd
Oxycodone 5mg qid
Esomeprazole 40 mg mane
Spirometry from 6 weeks ago (see PDF)
She arrives on the morning of surgery and your registrar rings you to tell you that she is wheezing. What is your response?”
You are reviewing a patient on the day of surgery, due to have a total knee replacement. In the preadmission clinic, he was noted to have a history of hypertension, treated with perindopril 10mg od; he is otherwise asymptomatic. On examination, you hear a harsh ejection systolic murmur, which had not been noted previously. What are the key features you would look for on the history in this patient?
You are asked by your surgical colleague to review a patient on your list today who has previously been deferred by the pre admission clinic registrar for medical review. The patient is a 68 year old man requiring an excision and local flap for an extensive Basal Cell Carcinoma of the nose and cheek. His comorbidities include end-stage renal failure requiring haemodialysis, hypertension, Chronic Obstructive Airways Disease with Obstructive Sleep Apnoea requiring CPAP and a previous history of CVA with incomplete recovery. On examination you find the patient moderately breathless (RR 15) at rest with an oxygen saturation of 90% on room air. How will you assess his suitability for surgery today?
“Surgeon’s booking form has the following information: 75 year old woman claudication and rest pain left leg for elective femoral-popliteal bypass
Atrial Fibrillation
Hypertension
Ischaemic Heart Disease
Chronic Obstructive Pulmonary Disease
Medication: rivaroxaban
perindopril
verapamil slow release
prednisolone
symbicort inhaler
salbutamol inhaler
GTN spray
ECG: rate controlled AF, old AMI
How would you assess her IHD and COPD, given she cannot exercise?”
You are the anaesthetist running the emergency list. A 50 year old man is admitted to the ward with fever, upper abdominal colic, and jaundice of 24 hour duration. The surgeon suspects ascending cholangitis and books the man for urgent laparoscopic cholecystectomy. The surgeon comments that the patient is 180 cm tall, weighs 100kg and has acromegaly. How do you plan to manage this patient?
A 58 year-old man is on your emergency list for a laparoscopic appendicectomy. He has a history of hepatitis C and chronic liver disease with ascites. How will you assess him?
“You are in the pre-anaesthesia assessment clinic. This 54-year-old truck driver is scheduled for excision of a posterior fossa meningioma in a few weeks’ time. His height is 175 cm and weight 140 kg (body mass index 46). His comorbidities include type 2 diabetes, hypertension and chronic back pain. His current medications are: metformin 1 g tds
quinapril 20 mg bd
oxycodone (sustained release) 20 mg bd
paracetamol 1 g qid
His observations are:
Blood pressure 145/85 mmHg
Heart rate 90 /minute (sinus rhythm)
SpO2 95% (on room air)
What are the important preoperative issues in this man?”
“Following a fall, a 78-year-old man needs a dynamic hip screw for an intertrochanteric fracture.
His past history includes:
1. Ischaemic heart disease
o STEMI in 2014 followed by coronary stenting for double vessel disease
o latest LVEF 23% with severe segmental left ventricular impairment and stage C heart failure
o NYHA III
2. Peripheral vascular disease
o open abdominal aortic aneurysm repair in 2010
o right leg claudication
3. Hypertension
4. Poorly controlled gastro-oesophageal reflux
His current medications:
aspirin 100 mg mane
quinapril 20 mg mane
frusemide 40 mg bd
metoprolol 25 mg bd
spironolactone 25 mg bd
omeprazole 20 mg mane
His recent blood tests show: (reference range)
Haemoglobin 110 g/l (130–170)
Platelets 95 x109 /l (150–450)
Albumin 29 g/l (35–55)
What are the important preoperative issues in this patient?”
“You are the on-call consultant anaesthetist at a large regional hospital. The anaesthetic registrar rings to inform you of a 24-year-old man booked for knee washout for septic arthritis. The patient has been unwell for one week with fever, rigors and chills. His left knee has been painful and swollen for three days. He initially presented to a peripheral hospital 12 hours earlier. After cultures were taken and antibiotics commenced he was transferred for further orthopaedic assessment and management. His only relevant past history is recreational intravenous drug abuse. The patient has arrived in the anaesthetic bay and looks unwell.
His vital signs are:
Heart rate 115 /minute
Blood pressure 80/40 mmHg
SpO2 90% (on room air)
Temperature 38.6 C
How will you respond to the registrar’s phone call?”
A 26-year-old man is booked for an urgent laparotomy by the general surgeons. He was admitted to hospital five days earlier with an acute spinal injury and his surgeons are concerned that he has infarcted bowel. What are your main concerns and what will you do to explore these concerns?
“A 46-year-old woman is attending the pre-admission clinic prior to a left total hip replacement for severe osteoarthritis. She has longstanding schizophrenia with associated intellectual impairment and lives in supported accommodation. She suffers from behavioural problems including screaming and hair pulling, and is requiring increasing analgesia and sedative medication. She is 163 cm tall and weighs 125 kg, with body mass index of 47.
Her current medications:
oxycodone 5 mg qid
chlorpromazine 200 mg bd
haloperidol 5 mg bd
diazepam 5 mg qid
sertraline 100 mg daily
Her carer states that eight years earlier, whilst the patient was having her gallbladder out, there was “difficulty placing a breathing tube”. What particular issues do you anticipate in this patient?”
“A 64-year-old woman is booked for frameless stereotactic craniotomy and debulking of a temporal lesion, to be done within 24 hours, following presentation with a generalised seizure. Her past medical history includes longstanding bronchiectasis and recently diagnosed pulmonary hypertension.
Her current medications:
fluticasone/salmeterol inhaler bd
frusemide 40 mg daily
bosentan 125 mg bd
ciprofloxacin (just completed two week course)
dexamethasone 4 mg qid (for the past 24 hours)
phenytoin 300 mg daily (for the past 24 hours)
Her MRI scan is shown here. How will you assess her fitness to undergo this procedure?”
“You are in the pre-anaesthetic assessment clinic. A 60 kg 80-year-old woman, is scheduled for excision of a large invasive skin cancer of her nose with extensive repair using a forehead advancement flap. Her past history includes rheumatoid arthritis, and an aortic valve replacement two months ago. Her current medications are:
Warfarin
Ramipril
Frusemide
Methotrexate
Prednisolone
What assessment of her cardiovascular system will you undertake?”
“Following a fall at home today, a 62-year-old woman requires a dynamic hip screw for a proximal femoral fracture. She is currently in the Emergency Department. Her medical history includes chronic obstructive pulmonary disease, hypertension, and atrial fibrillation. When you see her she is complaining of shortness of breath and hip pain. Her current medications are:
Fluticasone/Salmeterol inhaler 125/25 two doses bd
Tiotropium bromide inhaler 18 µg mane
Lisinopril 5 mg mane
Diltiazem CD 360 mg mane
Prednisone 40 mg mane
Amoxicillin 500 mg with clavulanic acid 125 mg (Augmentin) tds
This image is from a computed tomography scan she underwent three months ago. Describe how you will assess this patient.”
“You are in the pre-anaesthetic assessment clinic. A 74-year-old man is scheduled for cataract surgery next week. His comorbidities include ischaemic heart disease, hypertension, and Parkinson’s disease. He had a deep venous thrombosis six months ago. His current medications are:
Warfarin 3 mg daily
Levodopa/benserazide 100 mg/25 mg tds
Metoprolol 50 mg daily
Irbesartan 150 mg daily
Glyceryl trinitrate 600 µg sublingually prn
What are the key issues in your preoperative assessment?”
The Emergency Department registrar, who is currently doing a term in anaesthesia, calls you to discuss a patient. The patient is a 76-year-old man who fell from his bicycle, sustaining a fracture through the stem of his previous right total hip joint replacement. He is scheduled for revision hemi-arthroplasty on your trauma list in two days. In addition to a routine preoperative assessment, the trainee would like to know what information he should obtain in this case and how it will affect your management.
“You have been asked to review a patient who is booked for surgery tomorrow. He is a 56-year-old man who is scheduled for left total hip replacement in the lateral position. He weighs 140 kg and has a body mass index (BMI) of 43. His comorbidities are hypertension, obstructive sleep apnoea, and myasthenia gravis. His current medications are:
Ramipril 5 mg daily
Pyridostigmine 60 mg four hourly
Metoprolol 50 mg bd
Omeprazole 20 mg daily
Venlafaxine 150 mg daily
Discuss your initial assessment of this man.”
“You are an anaesthetist working in a major metropolitan hospital. A 52-year-old woman, who received a left lung transplant two years ago for chronic obstructive pulmonary disease, is scheduled for surgery. She now has endometrial cancer and requires total abdominal hysterectomy and salpingo-oophorectomy.
Her current medications include:
Tacrolimus
Mycophenolate
Prednisone
How will you assess her preoperatively?”
“You have asked your registrar to review a patient who is booked for fixation of fractures on today’s trauma list. She is a 78-year-old woman with fractures to her clavicle and humerus after being hit by an electric mobility scooter, in a low velocity impact accident two days ago. Your registrar tells you that the patient has a past medical history of hypertension and chronic back pain, and that her exercise tolerance is reduced to one flight of stairs. She is 160 cm tall, weighs 120 kg, is alert and orientated, and is fasted.
Her regular medications are:
irbesartan
aspirin
transdermal fentanyl
She is currently using intravenous patient-controlled analgesia. The registrar informs you that he has found a previously unreported pansystolic heart murmur, heard loudest at the apex. What particular issues concern you in providing anaesthesia for this patient?”
“You are the anaesthetist for a urology list at a freestanding day surgery unit. The first patient is a 64-year-old man who requires circumcision for phimosis.
His past history includes:
Ischaemic heart disease (non-STEMI 18 months ago, followed by a drug-eluting stent to the
right coronary artery)
Obesity with body mass index of 41
Severe obstructive sleep apnoea
Hypertension
Type 2 diabetes
His current medications are:
aspirin 100 mg daily
metoprolol CR 95 mg daily
lisinopril 10 mg daily
metformin 850 mg tds
NovoMix 30 (biphasic insulin) 25 units morning, 10 units evening
atorvastatin 40 mg daily
glyceryl trinitrate prn
His observations are:
Blood pressure 140/90 mmHg
Heart rate 70 /minute
SpO2 97% on room air
How will you assess this patient’s suitability for surgery in the day surgical unit?”
“You are called to the emergency department to see a 38-year-old woman, a known asthmatic who has presented with breathing difficulties.
Her current medications are:
fluticasone/salmeterol inhaler bd
salbutamol inhaler prn
Her current observations are:
Blood pressure 135/81 mmHg
Heart rate 96 /minute
SpO2 88% on room air
What are the key issues in your assessment of this woman?”
“Following induction of the first patient on your afternoon list, the surgeon mentions to you that an emergency case needs to be added to the end of the list. The patient is a 78-year-old woman with an obstructed femoral hernia. The surgical registrar tells you she has a background of hypertension, diet-controlled diabetes and a murmur. The patient is currently in the Coronary Care Unit (CCU) on a heparin infusion. You send your first year trainee anaesthetic registrar to review the patient, and he returns with the patient’s medication list and 12 lead electrocardiograph. Her current medications are:
perindopril 10 mg daily
atorvastatin 40 mg daily
metoprolol 25 mg bd (since admission to CCU)
heparin infusion (activated partial thromboplastin time 60–70 seconds)
The patient’s 12 lead electrocardiograph is shown here. What further information and assessment will you seek?”
“Working alone, you have just started the last case on your all-day urology list (expected to take approximately 90 minutes). The surgeon advises you that she wants to do an extra case. A 56-year-old woman has been admitted and needs a ureteric stent. She is febrile with a temperature of 38.5°C. Her medical history includes:
Insulin-requiring diabetes mellitus. Her blood glucose level is currently 18 mmol/l (325 mg/dl).
Hypertension. Her blood pressure is 100/60 mmHg (usually higher at around 140/70 mmHg).
How will you manage her diabetes mellitus?”
“In the Pre-Anaesthetic Assessment Clinic, your next patient is a 59-year-old man for repeat ophthalmic surgery. He has had several eye surgery procedures in the past few years: cataract surgery in each eye, and a trabeculectomy in the right eye. He wears glasses for myopia (short-sightedness) and has longstanding glaucoma. Nine months ago he had an episode of syncope and was admitted to hospital. He was fully investigated and no significant cardiac or respiratory disease was found. He is scheduled to have a revision right trabeculectomy in the stand-alone private eye daysurgery centre next week.
Background History:
Hypertension
Glaucoma
Low grade ongoing fever secondary to recurrent kidney stones
Arthritis
Medications:
Losartan / hydrochlorothiazide
Eye drops: Acetazolamide
Latanoprost (prostaglandin)
Timolol (beta-blocker)
Trimethoprim
Aspirin
Paracetamol as required
How will you decide which preoperative investigations are appropriate for this man?”
“Your junior anaesthesia trainee is assessing a 78-year-old woman with end-stage renal failure in the pre-anaesthetic assessment clinic, and calls you for advice. The patient is scheduled to undergo laparoscopic insertion of a peritoneal dialysis catheter on your list in one week’s time. She has type-2 diabetes, hypertension, osteoarthritis and restless legs syndrome. The trainee wishes to optimise the patient’s condition prior to surgery and formulate an anaesthesia management plan.
Current medications:
Soluble insulin 8 units s/c t.d.s.
Long-acting insulin 28 units s/c nocte
Perindopril 8 mg p.o. mane
Aspirin 100 mg p.o. mane
Omeprazole 40 mg p.o. mane
Carbamazepine 300 mg p.o. nocte
What advice would you offer your registrar?”
You are scheduled to care for a 54-year-old man, a long-term C4–5 quadriplegic, who needs a stent inserted for treatment of a ureteric stone. His initial injury was a diving accident and required a C3–6 cervical fusion. He has had numerous urological procedures in the past. He has difficult intravenous access. What are your anaesthetic concerns in a long-term quadriplegic patient?
You are the anaesthetist for a 40-year-old man booked for excision of a mandibular cyst in a free-standing day surgery unit. You have reviewed him before and the only issue you identified was undiagnosed, untreated hypertension for which you referred him back to his general practitioner. He has been commenced on treatment for essential hypertension. You review him on the morning of surgery and identify that he has omitted to take the angiotensin-converting enzyme inhibitor he has been prescribed, and his blood pressure is 170/100 mmHg. How will you manage this man?
“A 47-year-old man fell today and has a displaced comminuted ankle fracture, which needs operative fixation in a tertiary hospital today. No other injuries were sustained in the fall. He has classic myotonic dystrophy type 1. He was diagnosed 20 years ago, and the disease progression has been gradual. He lives at home with his brother. He has bilateral ankle orthoses for foot drop and walks with a frame. He is worried about how the anaesthetic will affect him, given his myotonic dystrophy.
His normal medications are:
Ibuprofen 400 mg p.r.n.
Gabapentin 300 mg t.d.s.
How would you assess his condition?”
“A 55-year-old man weighing 160 kg is scheduled for laparoscopic Nissen fundoplication and mesh repair of hiatus hernia. He has a history of severe gastro-oesophageal reflux disease, a large hiatus hernia, type 2 diabetes and obstructive sleep apnoea.
His current medications include:
* pantoprazole 40 mg bd
* metformin 850 mg bd
* isophane insulin 10 units mane, 16 units nocte
* quinapril 20 mg daily
* simvastatin 20 mg daily
He had an uneventful general anaesthetic two years ago and was documented to have a Grade 2 view at direct laryngoscopy. Describe your preoperative assessment of the cardiovascular system in this patient, given his morbid obesity.”
“A 72-year-old female is at pre-admission clinic the day before her elective three-level posterior lumbar intervertebral fusion for spinal stenosis. She has constant lower back pain with no neurological signs. She has well-controlled type 2 diabetes mellitus, and denies other medical history. Her exercise tolerance is restricted by pain. She walks slowly and can manage most domestic chores. The patient has a body mass index of 24kg/m2.
Her current medications are:
* oxycodone 20 mg modified release (MR) bd
* diclofenac 50 mg bd
* paracetamol 1000 mg qid
* gliclazide 60 mg MR mane
This is her full blood count from yesterday:
Haemoglobin 87 g/l (115-165)
red cell count 3.2 x1012/l (3.9-5.6)
haematocrit 0.28 (0.35-0.47)
MCV 70 fl (80-100)
MCHC 310 g/l (320-355)
white cell count 5.0 x109/l (3.5-12.0)
neutrophils 3.5 x109/l (1.5-8.0)
platelets 330 x109/l (150-400)
Describe your approach to her anaemia. What is your primary concern and how will you address it?”
“You receive a call from the orthoaedic surgeon on call regarding a 55-year-old female with metastatic breast cancer, who developed a pulmonary embolus two weeks ago. She now has a pathological fracture of her proximal left femur.
Her current medications are:
* apixaban 5 mg bd
* pregabalin 75 mg daily
* ramipril 2.5 mg daily
* transdermal buprenorphine 20 mg/hour
The surgeon has called to discuss timing of surgery and management of her anticoagulation. How will you manage her anti-coagulation?”
“You are the consultant working in the preadmission clinic of a large tertiary hospital. Your next patient is a 72-year-old man with recently diagnosed transitional cell carcinoma of the bladder. He is scheduled for a radical cystectomy and ileal conduit.
His past medical history includes:
* Chronic obstructive pulmonary disease
* Paroxysmal atrial fibrillation
* Current smoker
* Hypertension
* Type 2 Diabetes Mellitus
His medications are:
* Inhaled salmeterol (Serevent)
* Inhaled tiotropium (Spiriva)
* Inhaled salbutamol (Ventolin) prn
* Dabigatran 110mg bd
* Digoxin 125mcg mane
* Perindopril 4mg mane
* Metformin 500mg bd
How will you assess this man’s respiratory function?”
“A 54-year-old man is scheduled for orthognathic surgery comprising of the following procedures for his obstructive sleep apnoea:
* Le Fort I maxillary osteotomy
* Bilateral sagittal split mandibular osteotomies
* Genioplasty
His medical history included hypertension and a myocardial infarction 12 months ago for which he was managed with a bifurcation drug eluting stent. He has continued to smoke. His medications include
Aspirin
Clopidogrel
Irbesartan
Amlodipine
Ezetimibe/simvastatin
What concerns do you have for this man undergoing his planned procedure?”
“It is 1900 hours on a Monday evening and you are the on-call anaesthetist at a small regional hospital. You are called by a locum orthopaedic surgeon to review a frail 75-year-old female who has been transferred from a nursing home with a suspected fractured neck of femur. Her son and daughter have just arrived. The ambulance and nursing home notes state: “Unwitnessed fall several hours ago, shortened externally rotated left leg with severe left hip pain”
.
Her medical history includes:
1. Mild dementia
2. Chronic kidney disease (stage 3B)
3. Hypertension
Her medications are:
* Aspirin 100 mg daily
* Hydrochlorthiazide 25mg daily
* Atenolol 50mg daily
* Rivastigmine 6mg bd
* Omeprazole 40 mg bd
How you will assess this patient?”
“A 59-year-old male is presenting for elective resection of a melanoma metastasis to the brain in the Preoperative Clinic.
His medical history is significant for Hypertension, Dyslipidaemia and Ischaemic Heart Disease.
His medications include:
Ramipril 5mg mane
Rosuvastatin 5mg nocte
Aspirin 100mg daily
Metoprolol 25mg bd
Dexamethasone 4mg mane
Levetiracetam 500mg bd
Please describe your assessment of this patient.”
“You are in the pre-admission clinic assessing a 72-year-old woman who is booked for bilateral knee replacement on your list in three weeks’ time. She weighs 55 kg and is well, apart from being treated with telmisartan for mild hypertension. She has no history of ischaemic heart disease, heart failure or cerebrovascular disease.
Results Reference Ranges
Haemoglobin 129 g/l (102–152 g/l)
Platelets 251 X 109/l (150–400 X 109/l)
White cells 7.1 X 109/l (4.0–10.0 X 109/l)
Sodium 142 mmol/l (135–145 mmol/l)
Potassium 4.5 mmol/l (3.5–5.5 mmol/l)
Urea 6.3 mmol/l (3.5–10 mmol/l)
Creatinine 70 µmol/l (45–95 µmol/l)
eGFR 79 ml/min/1.73m2 (>89 ml/min/1.73m2)
How will you estimate her perioperative cardiac risk for this surgery?”
You are the emergency anaesthetist and a 70-year-old woman with rheumatoid arthritis and limited neck movement is booked for washout of a left total knee replacement (TKR). She had her total knee joint replacement two weeks ago under spinal anaesthesia where intrathecal access was noted to be difficult. Postoperative mobilisation has been slow and she has developed a painful calf with a left deep vein thrombosis identified on ultrasound with no respiratory symptoms. She remains an inpatient and is noted to have some confusion at present. She has been anticoagulated on warfarin with an International Normalised Ratio of 2.5. She wishes to have a general anaesthetic and prefers not to have a spinal for the washout procedure. Outline your preoperative concerns in this patient?
“You are in the anaesthetic pre-admission clinic. Your next patient is an eighty-year-old Italian man with a history of recurrent transient ischaemic attacks, presenting for awake carotid endarterectomy.
His medications are listed as follows.
Atorvastatin 40 mg daily
Amlodipine 5 mg daily
Metformin 500 mg tds
Aspirin 100 mg daily
Dipyridamole 150 mg daily
How would you assess this man prior to his anaesthetic?”
“You are the anaesthetist covering the emergency list on a weekend in a tertiary hospital. You meet your next patient in the holding bay. He is a 70-year-old man booked for a laparoscopic cholecystectomy for a suspected gangrenous gall bladder. His past medical history includes hypertension and he is known to have aortic stenosis. He has been in bed for a week with worsening right upper quadrant pain, nausea, fever, and lethargy. He is diaphoretic and looks unwell.
His current observations are:
SpO2 of 93% on two litres per minute of oxygen via nasal prongs
Respiratory rate 20 /minute
Blood pressure 95/40 mmHg
Heart rate 105 /minute
There is an echocardiography report dated February 2016:
Aortic stenosis, aortic valve area 0.91 cm2, mean gradient 39 mmHg, peak velocity 3.67 m/s, left ventricular size normal, mild concentric hypertrophy with diastolic dysfunction but normal left ventricular systolic function, ejection fraction 60%.
Medication:
Quinapril 20 mg bd
How will you proceed?”
“You are seeing a 50-year-old man in the pre-anaesthetic clinic. He is scheduled for robotic-assisted laparoscopic radical prostatectomy in one week’s time. He has type 2 diabetes mellitus and hypertension, and he smokes 10–15 cigarettes a day.
Current medications include:
Metformin
Empagliflozin
Ramipril
How would you prepare this patient for surgery?”
“You are the anaesthetist for a gynaecology list at a major metropolitan hospital. You are reviewing the next patient on the list in the anaesthetic room. She is a 36-year-old woman for removal of an intrauterine device (IUD) +/- laparoscopy. The surgeon’s booking notes state:
Removal of hormone-releasing IUD (Mirena) –possible extrauterine migration requiring
laparoscopy
Tetraplegia (15 years)
Obese
Current medications:
Amitriptyline
Baclofen
Cranberry capsules
What additional specific information do you require in order to formulate your anaesthetic plan?”
“A 34-year-old patient presents at your pre-admission clinic. She is booked on your list for a diagnostic laparoscopy for infertility. She has a BMI of 45kg/m2 and a past history of severe asthma. A respiratory physician is managing her for her asthma. The patient says that her asthma has improved and a letter from the physician states that her current FEV1 ranges from 50-64% of predicted and this has improved from 40-50% of predicted in the preceding 3 months.
Her current medications include:
Symbicort
Ventolin
Discuss the key issues in your pre-op assessment of this patient.”
“An 80-year-old man sustained right-sided rib fractures after a fall and was admitted to the surgical ward of your regional hospital last night. His only other injury is a fractured right clavicle. You are asked to review his pain management.
His past medical history includes:
Hypertension
Dyslipidaemia
Chronic obstructive pulmonary disease
Chronic moderate renal impairment
Type 2 diabetes mellitus
Prostate cancer
Regular medications:
Atorvastatin 40 mg daily
Budesonide-formoterol (50 µg/3 µg) inhaler bd
Irbesartan 75 mg daily
Metformin 500 mg daily
Salbutamol inhaler prn
Despite his medical issues, he is still active, and cares for his wife in their home. Currently, his pain management is tramadol by intravenous infusion at 20 mg per hour, and oral paracetamol 1 g qid. You attend to assess him and notice that he is disorientated and confused, although cooperative. Why may he have become confused?”
“A 40-year-old obese woman (body mass index 32 kg/m2, 95 kg) with myasthenia gravis is listed for a laparoscopic total hysterectomy and bilateral salpingectomy.
Medications:
Pyridostigmine 240 mg tds
Prednisolone 25 mg daily
Buprenorphine patch 10 µg/hour
Oxycodone 5–10 mg prn
Tramadol IR prn
Oral contraceptive pill
Allergies:
Sugammadex
How will you assess the severity of her myasthenia gravis in order to formulate a perioperative plan?”
You are in the pre-admission clinic reviewing a 48-year-old woman booked for laparoscopic total hysterectomy (excision of fibroids) on your list tomorrow. Her past medical history is of type 1 diabetes mellitus, for which she is on insulin administered via a subcutaneous pump. Her electrocardiogram is below. What does it show and what will you do?
“You are asked to review a 35-year-old woman who presents for revision of a left forearm arteriovenous fistula. Her comorbidities include end-stage renal failure due to nephrotic syndrome, for which she requires haemodialysis; morbid obesity; and known pulmonary hypertension.
Current medications:
Prednisolone 15 mg daily
Cyclosporine 150 mg bd
Amlodipine 5 mg daily
Aspirin 150 mg daily
How will you assess the severity of her pulmonary hypertension?”
“You are asked to see an 84-year-old woman in the acute orthopaedic ward. She was admitted earlier in the afternoon with a peri-prosthetic femoral fracture after a mechanical fall at home. She had a right total hip replacement five years ago for osteoarthritis. She is scheduled for revision total hip arthroplasty tomorrow morning. She has a history of cardiac failure, type 2 diabetes, hypertension and atrial fibrillation.
Medications: rivaroxaban 15 mg daily
atenolol 50 mg daily
metformin 500 mg twice daily
perindopril 5 mg daily
At your pre-operative consult she is distressed, in pain and difficult to engage. How would you assess her capacity to make decisions?”
“A 58-year-old man presents to the preadmission clinic prior to thoracic decompression and sural nerve biopsy to be done in the prone position. His history includes progressive lower limb and truncal weakness. Progression has occurred over the last four months and he is now wheelchair-bound. Weight 152kg, body mass index 48 kg/ m2
His past medical history includes:
obstructive sleep apnoea - on continuous positive airway pressure (CPAP) therapy
hypertension
Current medications:
candesartan 8mg daily
enoxaparin 40mg subcutaneous injection daily
pregabalin 150mg bd
What are the key issues you will address during the pre-anaesthetic assessment?”
“A 47-year-old man is on your list today for an open reduction and internal fixation of a right proximal humerus fracture. His injury was sustained in a motorbike accident three days ago. Following his accident he was assessed in the emergency department and then discharged home.
Medications:
Metformin 1 g bd
Atorvastatin 40 mg daily
Perindopril 8 mg daily
Paracetamol 1g q4h PRN (last three days)
Oxycodone 10mg q4h PRN (last three days)
weight 146 kg
height 180 cm
body mass index 45 kg/ m2
heart rate 78 beats per minute
blood pressure 148/86 mmHg
SpO2 96% on air
respiratory rate 16 breaths per minute
On initial inspection he has a full beard, a graze on the right side of his face, and bruising on the right side of his chest and right hip. His right arm is in a collar and cuff sling. What issues would you focus on assessing pre-operatively?”
“You have been asked to assess a 26-year-old woman with ulcerative colitis for an inpatient semi-urgent laparoscopic-assisted total proctocolectomy with ileoanal anastomosis.
Medications
mesalazine 2g orally bd
prednisolone 40mg orally daily
infliximab 300mg intravenously fortnightly
Reference ranges
sodium 135 mmol/L (135-145 mmol/L)
potassium 4.0 mmol/L (3.5-5.2 mmol/L)
creatinine 75 µmol/L (<105 mmol/L)
haemoglobin 105 g/L (115-155 g/L)
MCV 71 fL (80-100 fL)
MCH 23 pg (27.5-33.2 pg)
white cell count 6.4 x103 /µL (3.5-9.8 x103 /µL)
platelets 400 x103 /µL (150-450 x103 /µL)
iron 8 µmol/L (5.0-30 µmol/L)
transferrin 3.2 g/L (2.0-3.2 g/L)
transferrin saturation 12% (10-45%)
ferritin 40 ng/mL (20-250 ng/mL)
CRP 150 mg/L (<5 mg/L)
albumin 30 g/L (38-52 g/L)
What are the key points to consider when deciding on the best timing for her surgery?”
“You are the anaesthetist for the emergency theatre at a tertiary hospital. The surgeons have booked an 81-year-old male patient for repair of an incarcerated upper abdominal incisional hernia. You review him in the Emergency department.
81-year-old male
Examination:
On 2L/min oxygen, tachypnoeic
Pulse rate120 bpm
BP 105/65
Past medical history
CABG / MVR (St Jude mechanical valve) 2002
Chronic obstructive pulmonary disease with bullous disease – home oxygen 18 hours per day
Bowel cancer - laparotomy 2018
Medications:
metoprolol 50 mg daily
digoxin 62.5 microg daily
irbesartan / hydrochlorothiazide (150/12.5mg) daily
warfarin 5mg daily (INR target 2.5-3.5)
tiotropium inhaler 2.5microg daily (Spiriva)
fluticasone / salmeterol (250/25microg) 2 puffs twice daily (Seretide)
salbutamol inhaler prn
How do you plan to assess and optimise this patient prior to surgery?”
“You are the anaesthetist in the Anaesthetic Assessment Clinic. The spinal surgeon has referred an 82-year old woman for an anaesthetic assessment. The proposed surgery is L4/5 spinal decompression and instrumented fusion for lumbar spinal stenosis. The surgeon’s booking notes state:
hypertension
diabetes mellitus
myocardial infarction 10 years ago
Medications:
aspirin
metformin
quinapril
vildagliptin
This is today’s ECG (rate controlled AF). What are your management priorities for this patient based on this ECG?”
You are midway through an ENT list at a large metropolitan private hospital. The nurse in the admissions unit brings in an ECG for you to review, along with a GP Health Summary. The patient is scheduled for balloon dilatation of subglottic stenosis later on the list. Interpret the ECG. (see PDF)
“You are a consultant anaesthetist at a metropolitan tertiary hospital. On the day’s emergency list is an 82-year-old man for the debridement of his right forefoot. He is a current inpatient having been admitted from home, unwell with sepsis, the previous day. From the intern admission note you see that he has a significant past medical history including:
Hypertension
CCF – medically managed
Chronic renal impairment – eGFR 42 mL/min/1.73m2
Aortic stenosis managed by TAVI in 2019 and complicated by CVA with mild residual dysphasia
Dual antiplatelet therapy
Progressive dementia – still lives at home with support from his family and aged care service providers who visit daily You go to the ward to pre-operatively assess him. He is the patriarch of a large family and his son (who has medical power of attorney) is in attendance. How would you assess his peri-operative risk of both morbidity and mortality?”
“A 55-year-old male has been added to your general surgical list for the laparoscopic insertion of a peritoneal dialysis catheter. Their past medical history is of end stage renal failure, chronic obstructive pulmonary disease and hypertension.
His medications include:
amlodipine 10 mg po daily
frusemide 120 mg po mane
fluticasone/salmeterol inhaler two doses bd
He has had worsening shortness of breath on exertion over the past month. He tells you that at his last consultation with his respiratory physician he was told that he has “high blood pressure in his lungs”. How will you assess the severity of this patient’s recently diagnosed pulmonary hypertension?”
A 65-year-old female with long standing laryngeal papillomatosis, presents for laser ablation. She has dyspnoea and hoarseness that has worsened significantly in the last month. She is on no medications. Below is her preoperative ECG (see PDF). Describe this ECG
“You are in pre-assessment clinic reviewing a 68-year-old female. She is booked for bilateral total knee joint replacement surgery. Her initial details and medications have been documented by the clinic nurse:
Height 162 cm
Weight 100 kg
Body Mass Index 38.1 kg/m2
Medications:
insulin via pump
fentanyl patch 12 µg/hr TD q72hrly
pregabalin 150 mg PO BD
paracetamol 667 mg PO TDS
quinapril/hydrochlorthiazide 10 mg/12.5 mg PO daily
atorvastatin 40 mg PO daily
aspirin 100 mg PO daily (stopped by surgeon 2 days ago)
clopidogrel 75 mg PO daily (stopped by surgeon 2 days ago)
pantoprazole 40 mg PO daily
Allergies:
penicillin (rash)
Given this information, what medical problems are indicated that you will need to assess further?”
“You attend the subacute Coronary Care Unit to review a 52-year-old man with a six week history of increasing dyspnoea who has been scheduled for revision aortic valve replacement +/- mitral annuloplasty tomorrow. His past history includes a bioprosthetic aortic valve replacement eight years ago. On admission his echocardiogram showed severe aortic regurgitation and moderate mitral regurgitation, with an estimated left ventricular ejection fraction of 35% and a moderately dilated left ventricle. His only regular medication prior to this admission was aspirin 100 mg daily, but during this admission he has been commenced on:
* furosemide (frusemide) 80 mg bd orally
* perindopril 6 mg mane orally
* dobutamine infusion 2.5 mcg/kg/min intravenously.
When you attend the ward you find him walking around with his IV pole. He weighs 80 kg. Outline how you would determine if this patient is optimised for his surgery.”
“Your next patient on the emergency list is a 24-year-old man scheduled for a laparoscopic appendicectomy. His only past medical history is that of occasional self-limiting palpitations on exertion. He has been sick for three days with severe abdominal pain, nausea and vomiting. He weighs 70 kg.
His vital signs are:
* temperature 38.5° C
* heart rate 106 bpm
* blood pressure 100/71 mmHg
The surgeon suspects a perforated appendix. He has an electrocardiogram (ECG) in his notes. Please describe this ECG (see PDF)”
“You are assessing a patient on the neurosurgical ward who is booked on your list tomorrow for coiling of cerebral aneurysms in the hospital’s interventional radiology suite. The 56-year-old woman presented earlier in the day with a two day history of severe headache, vomiting and malaise that was unresponsive to paracetamol. There has been no change to her level of consciousness or focal neurological deficits. Cranial CT revealed five intracerebral aneurysms, the largest in the anterior communicating artery. There is evidence of diffuse subarachnoid haemorrhage, Fisher grade 2.
Her vital signs are:
* blood pressure 145/75 mmHg MAP 88 mmHg
* pulse 85/min sinus rhythm
Past medical history
Polycystic kidney disease
* renal transplant 10 years ago
* end-stage renal disease treated with haemodialysis for two years prior to
transplantation
Hypertension
Medications
atorvastatin 20mg daily
enalapril 5mg BD
mycophenolate 1g BD
prednisone 5mg daily
tacrolimus 5mg BD
trimethoprim / sulphamethoxazole one tab BD
What specific information do you require about this patient’s medical problems to ensure optimisation for the coiling procedure?”
“A 45-year-old man presents to your Preadmission Clinic for a review ahead of an open left adrenalectomy for phaeochromocytoma in four weeks time.
He currently takes the following medications:
* Bio Magnesium supplements 2 capsules daily
* dapagliflozin 10 mg mane
* enalapril 40 mg mane
* frusemide 20 mg mane
* metoprolol 100 mg BD
How would you assess a patient with a phaeochromocytoma who is to undergo surgical resection?”
“A 47-year-old woman with acromegaly attends your preoperative assessment clinic one week prior to undergoing a transsphenoidal hypophysectomy for pituitary adenoma. She has a history of hypertension and type II diabetes mellitus. She is a heavy smoker, has a hoarse voice and reports a recent hospital admission for investigation of shortness of breath on exertion.
Medications:
* lisinopril 20 mg daily
* metformin 1000 mg nocte
* metoprolol 100 mg mane
* octreotide 75 mcg subcutaneously x 3 daily
What features of the history will help you determine the severity of her condition?”
“A 30-year old man booked for an emergency laparoscopic cholecystectomy for a gangrenous gallbladder. He has a past medical history of juvenile idiopathic arthritis (formerly juvenile rheumatoid arthritis).
He appears unwell and is vomiting.
Vital signs
Blood pressure 100/60 mmHg
Heart rate 120 bpm
SpO2 93% (room air)
Respiratory rate 20 per minute
T 38.5°C
Based on the provided history and findings, describe your initial assessment of this patient.”
“A 30-year old man booked for an emergency laparoscopic cholecystectomy for a gangrenous gallbladder. He has a past medical history of juvenile idiopathic arthritis (formerly juvenile rheumatoid arthritis).
He appears unwell and is vomiting.
Vital signs
Blood pressure 100/60 mmHg
Heart rate 120 bpm
SpO2 93% (room air)
Respiratory rate 20 per minute
T 38.5°C
Based on the provided history and findings, describe your initial assessment of this patient.”
“A 75-year-old woman presented to the emergency department following a fall in which she sustained a fractured right hip. She has no other injuries and is cognitively intact. She has been scheduled for a right hemiarthroplasty as the first case on the operating list tomorrow morning.
Past medical history
Polymyalgia rheumatica, Chronic alcohol consumption (60 g per day), Previous deep vein thrombosis with pulmonary embolism, Chronic kidney disease
Height: 162cm Weight: 61kg
Medications
Fluoxetine 40 mg mane, Perindopril/indapamide 5 mg/1.25 mg mane, Prednisolone 15 mg mane, Rivaroxaban 15 mg mane, Rosuvastatin 20 mg mane
Blood test results on admission to the emergency department:
Haemoglobin 82 g/L (115-160)
Platelets 117 x10^9/L (150-450)
Na+ 127 mmol/L (135 - 145)
K+ 4.2 mmol/L (3.5 - 5.5)
Cl- 103 mmol/L (95 - 110)
HCO3- 20 mmol/L (20 - 32)
Urea 12 mmol/L (2.5 - 7.0)
Creatinine 145 mmol/L (45 - 95)
eGFR 30 mL/min/1.73m2 (>90)
Albumin 26 g/L (33 - 46)
A cardiac rhythm strip was printed in the Emergency Department, as shown below (long pauses). How will you assess if this patient is suitable for surgery tomorrow morning?”
A 74-year-old woman presents to the preanaesthesia clinic for review ahead of a laparoscopic right hemicolectomy for colorectal cancer. She is a smoker and has a background of longstanding bronchial carcinoid tumours. How would you assess this woman?
A 28-year-old male is booked on your list for resection of a large right-sided posterior mediastinal tumour. This was diagnosed after he presented with increased breathlessness and cough. He has no other significant comorbidities. How will you assess this patient preoperatively?
“You are the on-call consultant anaesthetist at a small regional hospital. At 09:00 hours on a Saturday morning you receive a call from the surgical registrar regarding a 69-year-old man who presented to the emergency department with a 24-hour history of worsening abdominal pain. An erect chest X-ray demonstrates free gas under the diaphragm. The surgical registrar would like to bring the patient to theatre for a laparotomy.
Past medical history
Current smoker – 50 pack-years, Ischaemic heart disease – non-obstructive, medical management, Abdominal aortic aneurysm – 4 cm diameter, currently under surveillance
Medications
Aspirin 100 mg daily
Atenolol 50 mg daily
Ibuprofen 400 mg tds PRN
Perindopril 5 mg daily
Rosuvastatin 20 mg daily
Height 175 cm
Weight 80 kg
BMI 26.1 kg/m2
(ideal body weight 70 kg)
Outline how you would respond to this request.”
“You are seeing a 72-year-old man in the preanaesthesia clinic of your tertiary hospital. He is booked for a wide local excision of a sarcoma of the right latissimus dorsi muscle in ten days time. The surgeon requests that he is positioned in the left lateral decubitus position. The surgery is anticipated to take two to three hours.
Past medical history
Coronary artery stent
Type 2 diabetes mellitus
Hypertension
Transient ischaemic attack two years ago
Medications
Clopidogrel 75 mg daily
Metformin 500 mg twice daily
Perindopril 8 mg daily
Rosuvastatin 20 mg daily
Observations performed in clinic
Blood pressure 165/95 mmHg
Heart rate 85 bpm
SpO2 98% on room air
Random blood glucose 8.5 mmol/L
Height 1.78m Weight 95 kg BMI 30
How will you assess this patient’s cardiovascular system?”
“You are the on-call anaesthetist at a private hospital. One of your regular colorectal surgeons has asked you to assess a 65-year-old man for an urgent laparoscopic high anterior resection for an obstructing tumour of his sigmoid colon. The surgeon would like to proceed later today and has booked an intensive care bed postoperatively in preparation.
Past medical history:
* Hypertension
* Ischaemic heart disease – percutaneous coronary intervention (PCI) six months prior after an acute coronary syndrome (ACS). A drug-eluting stent was placed in the left anterior descending coronary artery
o Most recent echocardiogram at time of stent insertion – unremarkable with no regional wall motion abnormalities
Medications:
* aspirin 100 mg daily
* atorvastatin 40 mg daily
* clopidogrel 75 mg daily
* oxycodone 5 mg PRN
* telmisartan 40 mg daily
The surgeon has asked for your advice regarding the perioperative management of his coronary stent and antiplatelet therapy. What is your advice?”
“You are an anaesthetist at a metropolitan private hospital. A 22-year-old woman requires surgery for an acute abdomen. She is day 4 post-laparoscopic sleeve gastrectomy that was performed for obesity. The ICU specialist has called you with her blood results. (See PDF). Usual medications:
* empagliflozin 10 mg daily
* escitalopram 20 mg daily
* lansoprazole 30 mg bd
* metformin 500 mg bd
* vitamin D 1000 IU daily
Outline your preoperative assessment and preparation of this patient for surgery.”
You are asked to see a 32-year-old primipara in early labour at 38 weeks gestation. She has had extensive spinal surgery for scoliosis. She would like to ask you about analgesia in labour. Her pregnancy has been uneventful and the obstetric plan is for a vaginal delivery. Her X-ray is shown below. Is regional anaesthesia feasible in this woman?
A 54-year-old man with renal failure requires surgery for formation of an arteriovenous fistula. He is keen to be awake during the procedure. What are the key aspects of your preoperative assessment and preparation?
Two days before surgery you are consulted to assess a hypertensive 81-year old man who is an inpatient with severe ischaemia of his right leg. The vascular surgeon is planning to perform a below knee amputation. How do you assess the suitability and timing, for a regional technique for this man?
“You are the anaesthetist on an orthopaedic list in a private hospital and the first patient on your list is a 40-year-old woman, booked to undergo a right arthroscopic acromioplasty and rotator cuff repair. The surgeon does this in the high beach chair position (at 80 degrees) and requires an interscalene catheter with a local anaesthetic infuser in situ to facilitate same-day discharge home. On seeing her in the pre-admission clinic, you note that her only medication is the oral contraceptive pill, and that she is a current smoker with a 10 pack-year history, with no significant past medical history apart from occasional gastro-oesophageal reflux.
Her admission vital signs are:
Heart rate 80 /minute
Blood pressure 135/70 mmHg (mean arterial pressure 100 mmHg),
Temperature 36.2°C
Respiratory rate 16 /minute
Weight 110 kg, height 170 cm, body mass index 38 kg/m2
What information will you give the patient about the interscalene block and local anaesthetic infuser, to facilitate informed consent?”
It is Saturday morning in a private hospital. You are reviewing a 50-year-old male patient who had an open anterior resection yesterday, on behalf of a colleague. He has an epidural in situ and on examination you find he has right leg weakness. His past history includes an automated implantable cardioverter-defibrillator for an out-of- hospital cardiac arrest. A recent echocardiogram shows normal ventricular systolic and diastolic function, and normal valvular function. What are the possible causes of his leg weakness?
“On your morning vascular list today is a 78-year-old woman who has been scheduled for an elective carotid endarterectomy. She has been extensively reviewed in the preanaesthesia clinic. Her underlying cardiovascular disease is considered mild, stable and optimised. Her renal function is normal. The patient has provided consent to receive either general anaesthesia or regional anaesthesia with conscious sedation. The surgeon has requested regional anaesthesia with conscious sedation.
Today’s observations:
* SpO2 – 97% (room air)
* HR 80 bpm, regular
* BP 160/90 mmHg
* RR 14 breaths/min
* Height 166 cm
* Weight 88 kg
* BMI 31.9 kg/m2
Outline the key points you would explore before proceeding with a regional technique.”
You are in the Emergency Department with a 30 year old male who was brought in by ambulance following a car accident. You are advised by the ambulance officers that he has a suspected intra- abdominal injury, a fractured humerus, facial injuries and a Glasgow Coma Score of 12. How will you assess this patient?
You are called to the pre-operative section of the day ward of your tertiary hospital. A 62 year old man has been admitted for knee arthroscopy and the nurse caring for the patient informs you that the patient complains of severe chest pain. The patient has a past history of stable angina on exertion and hypertension. The patient’s current medications include ramipril, metoprolol and frusemide. How would you assess this patient?
“As the anaesthetist in charge of the trauma theatre you are awaiting the imminent arrival of a 25 year old male climber, found at the base of a cliff this morning. It is unknown how long he was there.
He has the following injuries:
1. Bilateral compound tibial fractures,
2. Closed left femoral fracture,
3. Central dislocation of the left femur,
4. Distended abdomen with a FAST (focussed abdominal sonography in trauma) ultrasound scan demonstrating large amounts of free fluid.
His observations include:
BP: 60/35 mmHg
HR: 140 beats per minute and regular
RR: 38 breaths/min
SaO2 unrecordable
Temperature: 33.8o C
He has been intubated and initially resuscitated by the trauma team. He has received 4 litres of crystalloid and 4 units of O Neg blood intravenously. Despite this he remains hemodynamically unstable and is being urgently transferred to operating theatre for a trauma laparotomy. Outline your priorities in preparing the operating room prior to his arrival?”
You are the consultant covering a registrar (Level 3 supervision) who is conducting a gynaecology list. He rings you urgently to advise that the 41 year old woman who is undergoing a laparotomy for removal of an ovarian malignancy is bleeding and he wishes you to attend immediately. You walk into the operating theatre and notice 1.5L of blood in the suction drain reservoir and the monitors indicate a BP of 80/40 mmHg with a heart rate of 125 bpm. Outline your priority of actions on arrival to theatre
“You are the on-duty anaesthetist at a major metropolitan hospital. You are telephoned by an ambulance crew who are at the scene of an accident. A 42 year old man has been crushed between a truck reversing at low speed and a brick wall, sustaining significant pelvic injuries. He has been extricated and emergency services crews are in attendance. He is fully conscious, with the following observations:
-Glasgow Coma Score (GCS): 15
-Systolic BP: 80mmHg
-Heart rate: 110 beats/minute
-Respiratory rate: 18 breaths/minute.
The ambulance crew requests your advice on how rapidly to transfer the patient to your hospital. What is your advice to the ambulance crew?”
You are called to the Emergency Department to assist in the management of a 24 year old man, who is brought in with a knife handle protruding from his left flank. The patient is sitting up in bed, uncooperative and aggressive. He smells of alcohol and is complaining loudly of pain. He refuses attempts to put on a pulse oximeter probe and blood pressure cuff, but looks pink, and has a strong radial pulse at a rate of 90/min.
A 19-year-old intoxicated male has been stabbed multiple times in the upper left abdomen. He is drowsy and pale but talking. A FAST scan (focussed assessment with sonography for trauma) is positive for free intraperitoneal fluid. A CT scan and an arterial blood gas including some electrolytes are presented. Outline your initial assessment and management in the emergency department.
“A 28 year old male is trapped by his legs in his truck cabin after a collision between his truck and a petrol tanker. The other driver is dead.
o The fire brigade is on site, the truck cabin appears to be unstable and the surrounding area has been evacuated as there is a major risk of fire with petrol fumes evident.
o The patient is awake, clammy and in considerable pain.
o He is pale, his pulse rate is 140 beats per min and his BP is 190/110 mmHg.
o His legs are trapped under the steering wheel but his chest and abdomen appear free. He has normal chest movements.
o You have been called to the scene to help retrieve this man. Due to the explosive risk ordinary extrication cannot be used and surgical amputation is likely to be required.
o There is a surgeon in the retrieval team.
Describe your immediate plan of action.”
A 22-year-old male construction worker is brought to the Emergency Department after an industrial accident at a building site near your hospital. He has a partially amputated right arm at the mid-humeral level. The operative plan is to proceed with immediate reimplantation. What are the anaesthetic considerations for such a case?
“You are the anaesthetist on duty and leader of the trauma response team at a regional hospital, one hour away by air from a major trauma centre. A 25-year-old front seat passenger is brought to the Emergency Department twenty minutes after a 60 kilometre per hour side-impact collision with another vehicle. The patient is sedated and intubated with bruising over the left chest and has sustained bilateral femoral fractures.
His vital signs are
BP 70/40 mmHg
HR 140 /minute
SpO2 96% on FiO2 1.0
Ventilator settings: tidal volume 600mls, rate 10 /minute
Outline your initial priorities for this patient.”
You are the anaesthestist in charge of the theatre suite, and you are called urgently to attend a theatre where a senior colleague is attempting fibreoptic intubation. The patient is a 60-year-old with a history of alcohol abuse, obstructive sleep apnoea and severe refractory reflux, scheduled for endoscopic sinus surgery. When you arrive, the pulse oximeter reads 84%, blood pressure is 190/110 mmHg and the heart rate is 130 per minute. Your colleague informs you that he has just induced the patient with propofol 200 mg and rocuronium 120 mg. He states that he “can’t see anything” with direct laryngoscopy or with videolaryngoscopy. A nurse is applying cricoid pressure. Your colleague asks you to administer “some propofol” so that he can finish the oral fibreoptic intubation. How will you manage this situation?
You are called to assist an anaesthetic registrar in theatre. A 50-year-old male with chronic liver disease is having variceal banding for haematemesis. The first year trainee has given midazolam 5 mg, fentanyl 50 μg and propofol 100 mg. The patient’s blood pressure is 80/35 mmHg and oxygen saturation is 88%. The endoscope is in the stomach where there is a mixture of fresh and old blood. What will you do?
“You are called to the Emergency Department where a 20-year-old man has been brought in by ambulance with burns. He has burns to his chest, abdomen and legs, sustained when he threw petrol onto a fire two hours ago. He is screaming and unco-operative. Ambulance officers report the man had been picked up from a campsite where he and his friends had been drinking. His friends report that he is usually fit and healthy. No other injuries have been identified.
His observations on arrival are: Heart rate 130/minute
Blood pressure 140/80 mmHg
Temperature 37.5⁰ C
An arterial blood gas taken on arrival shows the following:
pH 7.2
pCO₂ 32 mmHg
pO₂ 95 mmHg
HCO₃⁻ 20 mmol/L
SpO₂ 100%
Lactate 5 mmol/L
Base excess -9
Haematocrit 55
How would you assess his volume status at presentation?”
You are a new consultant at your hospital and have been called down to help in the Emergency Department one evening as the staff are extremely busy dealing with multiple victims from an accident. You are expected to receive an 18-year-old male who was found to be unresponsive, lying on his back, just outside his front door. The patient appears to have fallen down the front steps and hit his head on the footpath. He was intubated at the scene by the paramedics. What do you want to do in managing this situation?
You are asked to assess an 80-year-old woman with a traumatic hip dislocation following a motor vehicle accident. She is booked for an urgent closed (possibly open) reduction of her left hip in Theatre. She has a background history of hypertension. Her primary and secondary trauma surveys and trauma X-rays are complete. Her blood pressure on admission is 240/130 mmHg. What are the possible causes for severe hypertension in this patient?
A previously healthy 28-year-old male electrician was electrocuted at a construction site and fell four metres. He was initially unresponsive and was defibrillated by an on-site first aid officer with an automated external defibrillator, after which he regained consciousness. He has just been brought into the Emergency Department by ambulance. He is alert and complaining of pain. He has obvious closed fractures of his left ankle and elbow, and an electrical burn entry wound to his right palm. He has a hard cervical collar fitted. You are present to assist in his management as a member of the trauma team. What potential injuries are you concerned about in this man?
A 57-year-old man presents to the Emergency Department with severe chest pain and acute onset paraplegia. He is hypertensive. His chest X-ray is shown. Describe the findings on this chest X-ray.
You are the anaesthetist designated to the Medical Emergency Team (MET) for the day. You respond to a MET call to the medical ward, which was prompted by deteriorating respiratory parameters in a 54-year-old man who was admitted during the night with pancreatitis. When you arrive you find the patient has obvious respiratory distress and is tachypnoeic at 30/minute. He is cyanosed and his Sp02 is 85% on oxygen via facemask at 8 litres/minute. He is drowsy but rousable to command, and appears to be in considerable abdominal pain. His abdomen is mildly distended. He looks to be about 80 kg. You note that his pulse rate is 135/minute and his blood pressure is 110/40 mmHg. He hasreceived two litres of crystalloid since his admission. He has not passed urine since he has been in hospital. What is your immediate management priority?
You are the VMO Anaesthetist on call at a provincial hospital. It is 10pm at night and you have just finished a Caesarian Section. You receive a call from the Emergency Department regarding an agitated 24 year old male. He has been involved in an altercation on the way home from the pub and has injuries to his head and neck from a baseball bat. When you arrive in the ED the patient is on a guerney at 45 degrees head up with a hard collar on. He has facial contusions and missing teeth. He is distressed with noisy breathing. HIS PR IS 100. BP 110/70 and SpO2 95% with a Hudson mask on. What do you think are the important issues to assess here?
“As the duty Anaesthetist at a large teaching hospital you respond to an emergency call for assistance in the Post Anaesthesia Care Unit (PACU). On arrival you notice a young woman being hand ventilated via Ambu-Bag by a junior registrar with two PACU nurses in attendance.Monitor scan shows:
HR 115bpm
SpO2 84%
NIBP 160/105 mmHg
The patient is clearly cyanosed What are your priorities?”
“A 20-year-old male has been brought to the Emergency Department with a stab wound to his anterior chest that occurred during a fight. On your arrival you find him agitated with the following observations:
SaO2 94%
HR 130
BP 95/50 mmHg
RR 20
Outline your immediate management of this patient who is uncooperative and aggressive?”
You attend a trauma call in the Emergency Department; you are told the ambulance has just arrived with an elderly pedestrian who has been hit by a car. What information do you need to obtain, during handover, from the ambulance staff?
You are the on-call consultant anaesthetist on Sunday morning at a district general hospital. You receive a telephone call from the emergency department registrar, who says: “We have a man down here with an injury to his face from an angle grinder. We’re worried about his airway. He’s bleeding profusely and we need you straight away.” You are currently anaesthetising an ASA physical status 3 patient for laparotomy, with a first year trainee anaesthetic registrar. What will you do?
“It is 0900 hours on Saturday morning; you are working in a large regional hospital and respond to a trauma call to the emergency department. A young man has sustained injuries from an accident when his motorbike hit a tree. He was initially found confused at the scene by a passing motorist. On arrival in the emergency department he is uncooperative and smells of alcohol. He has obvious large lacerations to his head and lower limbs.
His observations are:
Blood pressure 90/50 mmHg
Heart rate 120 /minute
Respiratory rate 22 /minute
SpO2 95%
Temperature 35.3°C
How will you approach this uncooperative patient?”
“You are the anaesthetist on trauma call at a tertiary hospital. The retrieval service has just brought a 26-year-old man to the emergency department following an industrial blast injury. He has sustained burns to his anterior chest, abdomen, and legs. He is already intubated.
His vital signs are:
Blood pressure 80/45 mmHg
Heart rate 110 /minute
SpO2 80% (FiO2 1.0)
Temperature 36.5°C
Describe your initial management of this patient.”
You are on the pain round with the acute pain nurse when you attend an emergency buzzer call on the ward. An elderly patient has collapsed in the bathroom and is unconscious. You are the most senior doctor in attendance. How will you manage this situation?
You receive a trauma call to the emergency department of the regional hospital where you are working. The paramedics have brought in a 30-year-old male who fell from a quad bike while intoxicated. He was dragged some distance before the quad bike rolled onto his legs, pinning him. On arrival in the emergency department, he is conscious, screaming and has mangling injuries to his lower limbs. Bilateral arterial tourniquets are in situ. He has tyre marks across his abdomen and an open abdominal wound contaminated with dirt and grass. Outline your priorities in the initial management of this patient.
You are the duty anaesthetist at a regional centre, and are urgently called to the emergency department. Following a drunken fight over dinner, a 38-year-old man has been stabbed in the neck with a kitchen knife, which is still deeply embedded. He is being cooperative with the emergency department staff. How will you respond?
You are the on-call consultant anaesthetist in a tertiary hospital. You are called to the emergency department to assist in the management of a 52-year-old male pedestrian hit by a car. Preliminary information from the ambulance service is as follows: Four fractured limbs, pelvic binding, open leg fractures. Cervical collar in situ. Intubated for low Glasgow Coma Scale (GCS) score. Past history of cirrhosis and portal hypertension. What will you do before the patient arrives?
It is 2000 hours. You are the anaesthetist on-site at a large regional hospital and have been called to the Emergency Department. On your arrival, you are informed that a major road traffic accident has just occurred. There were two fatalities at the scene. Three surviving patients are being transported to the hospital, and are due to arrive in 15 minutes. The first patient has been intubated and cardiopulmonary resuscitation is in progress. The second patient is now conscious. He has right-sided chest and abdominal injuries. The third patient is conscious. He has obvious left leg fractures with absent distal pulses.The only Emergency Department consultant is currently managing a patient in cardiac arrest. She asks you to organise the distribution of personnel for the impending arrivals. What are your management priorities?
You are a consultant in a large private hospital and have just handed over your last patient in the post-anaesthesia care unit (PACU). As you leave the PACU, an anaesthetic nurse calls you to assist an anaesthetist in another theatre. On entering the theatre, you immediately observe a conscious patient undergoing surgery on his forearm, the monitor showing ventricular tachycardia, and your anaesthetic colleague hurriedly drawing up medications. How will you proceed?
“You are the duty anaesthetist in a major teaching hospital and are called by the anaesthetic fellow to provide him with assistance in theatre. He tells you his patient is a 56-year-old woman who was added to the list for laparoscopic removal of an intra-abdominal mass (possibly a splenic lymphoma) and insertion of vascular access for chemotherapy. The case has now been underway for 45 minutes. On arrival you note the following vital signs:
Blood pressure 85/40 mmHg
Heart rate 133 /minute
How will you manage this situation?”
A 30-year-old man has been injured in an explosion whilst manufacturing illicit drugs (methamphetamines). He has been retrieved from the scene by a medical team and is expected to arrive at your tertiary hospital in about ten minutes. Your registrar phones you for advice. What information do you want from the registrar?
“It is 0100 hours. You are the on-call consultant anaesthetist at a private hospital. The on-duty consultant emergency physician calls for urgent assistance and you attend. A few minutes ago, a 28-year-old man staggered into the emergency department smelling of alcohol. He was hit in a fight, has bruising to his neck and face and has inspiratory stridor.
His vital signs are:
Respiratory rate 24 /minute
SpO2 95% on room air
Blood pressure 160/90 mmHg
Heart rate 100 /minute
Temperature 36.7°C
What is your primary concern and how will you address it?”
“It is 0300 hours on a Sunday morning in a central city teaching hospital. A 24-year-old male has been involved in an altercation in a nightclub and has been attacked with a knife. The patient is distressed and agitated on arrival in the emergency department where you have been called to assist in resuscitation. He is pulling off his oxygen mask. There are multiple puncture sites in the upper abdomen. His initial observations are below.
Glasgow Coma Scale score 14/15
heart rate 126 /minute
blood pressure 95/60 mmHg
SaO2 99%
respiratory rate 28 /minute
Describe your initial assessment and management.”
A 28-year-old male surfer sustains a degloving injury of his right thigh from a shark bite. He arrives at your regional hospital’s emergency department in extremis. You are called to assist. He is conscious and a pressure bandage has just been removed from his leg. His heart rate is 140/minute, blood pressure is 60/30 mmHg, and temperature is 34.4°C. How much blood do you think he might have lost? How will you manage this?
A 120kg 25-year-old rugby prop has suffered an injury during a game. He has arrived in the emergency department with signs of an incomplete C4 quadriplegia. The emergency department staff ask for your assistance in transferring the patient to the computed tomography (CT) scanner. When you arrive in the emergency department, he is on a rigid spine board with a hard cervical collar in place and his head taped to the board. He has a Glasgow Coma Score of 15. He is breathing room air. Your emergency colleague tells you he has some movement of his right arm but none in his other three limbs. A lateral cervical X-ray has been obtained and CT are waiting to scan his cervical spine. Is there anything you would like to do before transporting the patient to the scanner?
“You are the helicopter retrieval consultant, tasked to retrieve a 30-year-old woman with decreased level of consciousness from a regional hospital. You are informed that it is suspected that the patient has overdosed. When you arrive, the patient is in the resuscitation area of the emergency department. Her observations are
SpO2 98%
Respiratory rate 20 /minute
Heart rate 110 /minute
Blood pressure 80/50 mmHg
Temperature 38°C
Glasgow Coma Scale score 11 (E3, V3, M5)
Describe your initial assessment and management of this patient.”
“You are the anaesthetist on the trauma team at a tertiary referral hospital. The following pre-hospital information has been received from the rescue helicopter: Mechanism 55-year-old male rural firefighter trapped under a fallen tree whilst fighting a bushfire
Injuries Burns on face and over lower abdomen and legs
Signs Confused
Heart rate 140 /minute
Systolic blood pressure 90 mmHg
SpO2 98%
Treatment Oxygen mask
Morphine 10 mg
What are the immediate priorities of the trauma team upon receiving the patient?”
“You are an anaesthetist in a regional hospital and your duties include covering their (small) high dependency unit. The emergency department calls you to assist with an unwell 60-year- old man who has presented with severe abdominal pain and some respiratory distress. He has been vomiting. He is well known to the emergency department on account of frequent presentations with alcohol-related problems.
His blood results are:
Reference Ranges
Haemoglobin 185 g/l (125–180 g/l)
White blood cells 16.1 x 109 /l (4–11 x 109 /l)
Platelets 166 x 109 /l (150–400 x 109 /l)
Haematocrit 0.61 (0.4–0.54)
Na+ 141 mmol/l (135–145 mmol/l)
K+ 4.5 mmol/l (3.5–5.2 mmol/l)
Urea 9.1 mmol/l (3.0–8.0 mmol/l)
Creatinine 108 µmol/l (60–110 µmol/l)
ALT 68 U/l (<40 U/l)
AST 38 U/l (<45 U/l)
GGT 171 U/l (<60 U/l)
Bilirubin 20 µmol/l (<20 µmol/l)
Albumin 35 g/l (35–50 g/l)
Glucose 8.1 mmol/l (3.5–6.1 mmol/l)
Lipase 481 U/l (<60 U/l)
Medications:
Metformin
Perindopril
Spironolactone
What do you think might be going on and what will you do on arrival in the emergency department?”
A 68-year-old woman is undergoing a Whipple’s procedure for an early pancreatic cancer. You are administering a volatile anaesthetic, with an epidural, arterial line and central venous line. She has been stable during the surgery. She has a past history of anxiety and depression, but is otherwise well. The surgeons are now about an hour away from finishing. Your monitor alarms and you look at your screen (below). (ECG provided in pdf). What is your initial management?
A 27-year-old man is dropped off by an unknown person at the emergency department of a major trauma centre. He is pale and diaphoretic and collapses in the doorway, so he is carried into the resuscitation bay. You are nearby and rush to assist as part of a trauma team. You see he has a gunshot entry wound in his left back (mid-chest level) and an exit wound in his abdomen (left upper quadrant). He loses consciousness in front of you. His heart rate is 140 /minute. You are unable to record a blood pressure or feel a peripheral pulse, but he has a weak carotid pulse. How will you assist with this patient’s management?
“You are called by the emergency department consultant in your tertiary hospital to see a 75- year-old man who has just returned from the radiology department. He was the driver in a single-vehicle accident. He was wearing a seatbelt and the airbags deployed. No one else was injured. The primary and secondary surveys reveal no cervical spine, pelvic or abdominal injuries. He reports chest pain and difficulty breathing.
His observations are:
SpO2 93% on non-rebreather bag
Respiratory rate 32 /minute
Heart rate 115 /minute
Blood pressure 110/60 mmHg
Glasgow Coma Scale score 14
This is his chest X-ray. How will you assess and manage this patient’s respiratory function?”
A 25-year-old pedestrian is hit by a car and presents to your large rural hospital which commonly manages major trauma. You attend the emergency department on his arrival. His oxygen saturation is 96% on a non-rebreathing Hudson mask delivering 15 litres per minute of oxygen. Heart rate and blood pressure are recorded at 150 /minute and 90/60 mmHg respectively. His Glasgow Coma Scale score is 15. He weighs 70 kg. His abdomen is painful and distended. Both his thighs are swollen and both legs are in splints. There are no other injuries on primary survey. How would you assess his degree of shock? How much blood do you estimate he has lost?
“A 56-year-old man presents to the emergency department after a motor vehicle accident involving a fatality in the other vehicle. You have been asked to escort this patient for a computed tomography scan. He complains of bilateral upper limb paraesthesia. His past medical history includes mechanical aortic valve replacement.
Medications include:
Warfarin 3 mg daily
He appears oriented with a blood pressure of 90/40 mmHg and a heart rate of 50 /minute. What are your concerns?”
“You are the emergency anaesthetist working in a major hospital. You receive a call alerting you to the arrival of a patient in ten minutes. The ambulance service is bringing a 27-year-old male who has fallen from a height of 5-10m.
He is reported to have:
GCS 7
SBP 80 mm/hg
HR 125 bpm
Oxygen saturation 89% on Oxygen 6l/min
Pupils fixed at 4mm
En route the ambulance performed a right-sided needle thoracostomy. The patient has IV access x2, a pelvic binder and cervical collar and has not been intubated. Theatre will not be available for at least 45 minutes Describe your initial assessment and management”
“A 47-year-old male is brought into the emergency department of a tertiary hospital where you are the anaesthetist on call. He has sustained abdominal gunshot wounds about 2 hours ago with only minor revealed bleeding obvious. He is diaphoretic and agitated requiring restraint. Your help is required to assist keeping him still enough for larger bore intravenous access (he currently has a 20 gauge cannula) and CT angiogram. He is believed to have recently used methamphetamine.
BP 100/56
HR 70bpm
SaO2 96%
He has a background of illicit drug use and a known methamphetamine induced cardiomyopathy (ejection fraction 20-30% on recent echocardiogram). He has extremely poor dentition (“Meth Mouth”).
Medications:
telmisartan 40mg daily
carvedilol 25mg (slow release) daily
frusemide (furosemide) 20mg daily
How would you assess his volume status for anaesthetic intervention prior to imaging?”
You are the day time anaesthetist on for trauma calls at a major trauma hospital. You have been asked to attend a trauma call in the Emergency Department for a 68-year-old woman after a motor vehicle accident earlier today. The patient was driving on the highway and her car crashed into a tree. She has a past history of hypertension, previous TIA and previous DVT. Currently she is alert and talking with a GCS of 15. Her BP is 130/80, HR 110bpm and SO2 95% on 6L via Hudson mask. This is her Chest X-ray on admission: (displaced rib fractures, no pneumothorax). What are your concerns?
You are the on-call anaesthetist for a regional base hospital. It is 2030 hours and you have just arrived in the carpark to review a patient for tomorrow’s elective operating list, when you receive a request for assistance from the ED consultant. He is busy resuscitating a sick patient and cannot attend a new category 1 trauma patient that has just arrived by ambulance. The new arrival is a 45-year-old man who has been assaulted at a local hotel. He has been struck in the neck with a broken beer bottle and was found by paramedics lying on the floor of the public bar. They noticed profuse bleeding from an anterior neck wound before applying some gauze. He is restless, irritable and combative. How are you going to manage this situation?
“You are the on-site anaesthetist in a large regional centre and have been called to the Emergency Department to assist with the management of an 70 year-old woman who was brought in by ambulance ten minutes ago after falling down the stairs at home. The patient lives independently and was discovered at the base of the stairs by her visiting daughter this morning after having fallen down the stairs last night. The patient was unable to reach the telephone to call for help and was not wearing her personal alarm button. The patient is sitting up on a trolley and appears short of breath.
Observations:
HR - 113 bpm
BP – 148/92 mmHg non-invasively
SpO2 - 92 % on oxygen 15 l/min via non-rebreather mask
GCS - 12 (E3 V4 M5)
Past medical history:
* atrial fibrillation
* chronic back pain
Regular medications:
* apixaban 2.5 mg bd
* buprenorphine transdermal patch 15mcg/hour
* digoxin 125 mcg daily
* perindopril 2 mg daily
Describe your initial assessment and management of this patient.”
You are the duty anaesthetist in a small regional hospital and have been called urgently to assist the anaesthesia registrar in the emergency theatre. The registrar is anaesthetising a previously well 25-year-old man (75 kg, ASA I) who is undergoing open reduction and internal fixation of a fractured tibia and fibula sustained in a skiing accident two days ago. The registrar has noted progressive desaturation and increasing tachycardia over last 15 minutes. The surgeons have deflated the torniquet and are closing the wound. What will you do when you enter the operating room?
“You are the anaesthetist on-call in a major trauma centre. The orthopaedic registrar has booked a 45-year-old woman for an urgent C3 – C7 decompression and fusion for C5/6 bilateral facet joint fracture-dislocations. She has an incomplete spinal cord injury. The patient sustained the injury jumping off a cliff in an act of deliberate self-harm. She has had a prolonged extraction time due to difficult terrain.
Past medical history
- anxiety and depression with multiple previous suicide attempts
- polysubstance use disorder with previous intravenous drug use
Medications
clonazepam 0.5 – 1.0 mg PRN for anxiety
methadone 100mg mane
quetiapine 300mg nocte
Please comment on her chest X-ray.”
“As the duty anaesthetist in a tertiary hospital, you are asked to attend the emergency department for the impending arrival of a 38-year-old male who has been struck on the head while working on a building site. On arrival with the paramedics he is unconscious with a laryngeal mask airway in situ. He has a compression bandage applied to his head and severe right periorbital and midface swelling The initial observations from the paramedics are as follows:
Heart rate 76 bpm
Blood Pressure 167/90 mmHg
SpO2 90% spontaneous ventilating on a T-piece with oxygen at 15 L/min
Respiratory rate 28/min
Left pupil size 4 and non-reactive
Right pupil size 2 and sluggishly reactive
What are your priorities in the management of this patient?”
“You arrive at a trauma call in a regional hospital just as the emergency doctor has intubated the patient. As a self-inflating bag is being connected, a team member announces that they cannot feel a pulse. You are told the patient is a 25-year-old male who fell approximately three metres as a result of collapsed scaffolding and has just been intubated because of increasing restlessness and a falling GCS (Glasgow Coma Scale). You agree to lead the ongoing resuscitation. The bedside monitor shows:
ECG HR: 120 bpm
Last recorded SBP: 90 mmHg
Blood pressure monitor is cycling and not recording a pressure
Pulse oximeter: ‘searching’
How will you manage the resuscitation?”
“A 62-year-old woman has just arrived by ambulance to your burns centre emergency department and you have been called to assist as part of the trauma team. She was caught in a house fire where she was trapped indoors for ten minutes before firefighters were able to free her. Forty-five minutes have lapsed since rescue.
Past medical history:
* Smoking 40 pack-year history
* Mechanical mitral valve replacement
* Chronic kidney disease
Medications:
* warfarin 5 mg daily
Examination findings:
Approximate weight 70 kg
SpO2 99% (oxygen via Hudson mask at 10 L/min)
HR 120 bpm (sinus rhythm)
BP 100/70 mmHg
RR 21/min, shallow breathing and coughing intermittently
Chest auscultation: mild diffuse wheeze.
Her burn injuries have been documented by the paramedic in the diagram overleaf (hashed areas).
The burnt skin is a blotchy red or dark pink colour and has large blisters. Capillary refill is sluggish (> 2 seconds) and there is loss of sensation in some areas. She is confused about the events, visibly distressed and in pain. She has been administered 10 mg of morphine intravenously. How do you assess her burn injuries?”
You are to anaesthetise a 52 year old man. He is scheduled for a decompressive L4-S1 laminectomy and pedicle screw stabilisation. How do you prepare for the prone position?
“You are called to the Emergency Department (ED) to assist with a 64-year-old patient. The ED triage nurse knows the patient and advises that he has alcoholic liver disease. He had a witnessed fall and knock to the head while drinking at a local hotel. The Emergency Department is extremely busy and a junior doctor needs assistance in transferring the patient to the CT scanner. The patient smells of alcohol, is covered in old bruises, looks malnourished and has a slightly distended abdomen. Vital signs include: oxygen saturation 94%
blood pressure 85/50 mmHg
heart rate 100/minute
Glasgow Coma Score 11 (pupils are equal and reactive to light, he is confused and localises when roused)
What will you do to help get the patient through the CT scanner?”
You are working in a stand-alone day surgery facility and are providing sedation to a 75-year- old man for excision of multiple facial skin lesions. The patient is otherwise healthy, with no previous anaesthetic issues. You have administered clonidine 30 µg and fentanyl 100 µg, and are giving oxygen via nasal prongs. During removal of a lesion near the nose, diathermy is used and a flame is noticed which the surgeon extinguishes. What will you do?
You are the duty anaesthetist in a teaching hospital. The anaesthetist in the emergency theatre is a senior registrar. His next patient is an obese 56-year-old man for laparoscopic cholecystectomy, following a recent episode of pancreatitis. As you pass the anaesthetic holding area the registrar is finishing his preoperative assessment of the patient. You notice that the patient has a hoarse voice and appears somewhat short of breath. How will you proceed?
You are asked to see a 20-year-old female intravenous drug user who requires tricuspid valve replacement due to endocarditis. She has been receiving subcutaneous heparin for two weeks. Her platelet count is 70 x 109 /L. What is the significance of the platelet count in this clinical situation?
A 50-year-old man with acute coronary syndrome is scheduled for urgent cardiac surgery in three hours. He has triple vessel disease with 80% left main stenosis, 75% left circumflex stenosis and 70% right coronary stenosis. His ejection fraction is 45%. His medications include prasugrel, aspirin, tirofiban and a heparin infusion. Discuss your preoperative assessment of this man.
“A mildly confused 86-year-old woman with known rheumatic mitral valve disease has presented to your hospital for the third time in the last four months. Her major symptoms are breathlessness & fatigue. This woman has a medical history of Hypertension, Hypercholesterolaemia, Obesity (Height 160cm, Weight 101 kg, Body Mass Index 39.45 kg/m²)
She takes the following medications: Irbesartan 150mg daily
Atorvastatin 40mg daily
Esomeprazole 40mg daily
Frusemide 40mg twice daily
Sotalol 40mg twice daily
The treating cardiologist is considering performing percutaneous mitral balloon valvotomy. The cardiologist requests an opinion regarding this patient’s ability to tolerate general anaesthesia for the procedure. A recent transthoracic echocardiogram is provided. Please outline what further information you might require to form an opinion and what your initial response to the cardiologist will be.”
It is 10pm and you are asked to review a 56-year-old man scheduled for surgery for an irreducible inguinal hernia. He has an Automatic Implantable Cardioverter Defibrillator (AICD), inserted 2 years ago following a VF arrest. He has long QT syndrome. How will the AICD influence your management of this man?
“A 75-year-old sedentary man with a history of aortic stenosis is re-booked for cardiac catheterization prior to aortic valve surgery. The cardiologist has requested an anaesthetist to assist as the procedure was cancelled the day before because the patient was agitated and unable to keep still. The patient stopped smoking 2 weeks ago and rarely goes to the doctor. His known current medications are:
Aspirin 100mg daily
Esomeprazole 40mg bd
How will you manage this patient for his cardiac catheterization?”
You are the off-floor anaesthetist who is called to the interventional cardiology unit as part of a MET call. As you arrive, staff are helping an 85 year old man back to his bed. You are told he collapsed whilst getting dressed in readiness for discharge home. The cardiology fellow who is there tells you that he performed a coronary angiogram 2 hrs earlier and found extensive calcification throughout his arteries unsuitable to stenting. He mentions that the patient has a history of peripheral vascular disease, insulin-requiring diabetes and currently takes anti-hypertensives and aspirin. What further information would you seek from the cardiology fellow as you assess this man?
You are called to the cardiac catheter lab to provide assistance with a 54-year-old man. On arrival you find the patient is agitated and has severe central chest pain. The cardiology professor wants you to “put the patient to sleep” immediately. This is the patient’s 12 lead ECG. What does this ECG show? How will you manage this situation?
“You are asked to anaesthetise a 62-year-old man for left shoulder arthroscopy and rotator cuff repair. His past medical history includes a community cardiac arrest with a full recovery. He now has an automated implantable cardioverter defibrillator.
His current medications are:
bisoprolol 5 mg bd
losartan 50 mg daily
atorvastatin 40 mg daily
aspirin 100 mg daily
His chest X-ray is shown here. What is your perioperative management plan for this patient’s cardiac device?”
“The first patient on your list tomorrow is a 49-year-old man with dilated cardiomyopathy, scheduled for insertion of a biventricular pacemaker/implantable defibrillator. The patient has been in hospital for two weeks for treatment of biventricular failure.
His medications include:
* frusemide 80 mg mane and midi
* Slow K 600 mg bd
* perindopril 10 mg mane
* aspirin 100 mg mane
* spironolactone 50 mg mane
* thiamine 100 mg mane
The cardiology team have requested a general anaesthetic for the procedure. He has the following chest x-ray. Please discuss the major issues concerning this case.”
“You are the Duty Anaesthetist onsite at a tertiary hospital which facilitates interventional cardiology and cardiothoracic surgery. You receive a phone call from the Interventional Cardiology Suite this evening, requesting immediate assistance for complications during percutaneous coronary intervention (PCI). A 65 year old man has presented with acute chest pain and is now undergoing angioplasty and stenting of the left anterior descending (LAD) coronary artery. He had been stable in the ambulance. He smokes 15 cigarettes a day, has a BMI of 35, hypertension and type 2 diabetes mellitus.
Usual medications: Quinapril
Atorvastatin
Metformin
What are you considering as you make your way immediately to the Interventional Cardiology Suite?”
A 76-year-old man is admitted the night before elective coronary artery bypass grafts and aortic valve replacement for aortic stenosis. As you arrive for your preoperative visit the nurse tells you the patient was well on admission but is now complaining of shortness of breath, and she is about to perform an electrocardiogram. Describe your management of the situation.
“You are the anaesthetist in the pre-anaesthesia assessment clinic seeing a 78-year-old man for insertion of an automated implantable cardioverter-defibrillator (AICD) the following day.
His medications include:
Bisoprolol
Aspirin
Perindopril
Atorvastatin
Spironolactone
Thiamine
His clinic observations are:
Blood pressure 95/58 mmHg
Heart rate 72 /minute
SpO2 98% on room air
What features of his history indicate the severity of his cardiovascular disease?”
You are assessing an 80-year-old man in the coronary care unit just prior to a planned permanent pacemaker insertion for persistent bradyarrhythmias. The cardiologist has requested anaesthesia assistance as the patient has been having difficulty lying flat due to back pain. He had an elective proximal right coronary artery stent seven days ago, and a TAVI performed under general anaesthesia five days ago, for treatment of his coronary artery disease and severe aortic stenosis. How would you assess this patient prior to his procedure?
You are the duty anesthetist for your cardiovascular theatres. You are called to the cardiac catheter lab recovery where they have a hypotensive patient following coronary angiography. The medical emergency team (MET) have been called. The patient is a 55-year-old 85 kg male. He has no allergies or anaesthetic issues.He has a new presentation of heart failure and is being assessed for coronary artery disease. His pre procedure echocardiogram showed a mildly dilated left ventricle with moderate dysfunction and an ejection fraction of 40%. On entering the recovery room you notice a number of people around the patient with more arriving. The monitor shows (see PDF). What will you do?
“A 76-year-old man with infected pacing wires has been scheduled for removal of his entire pacing system under general anaesthesia. The procedure is to be performed in a hybrid theatre in the cardiology department.
Current medications
aspirin 100 mg daily
atorvastatin 40 mg daily
flucloxacillin 2g qid IV
perindopril 1 mg daily
His chest X-ray is displayed. Considering your clinical assessment of this man, what are your main areas of concern?”
“You are currently providing anaesthesia for the transoesophageal echocardiogram and cardioversion list. The cardiology registrar informs you that he has added a 65-year-old man to the end of the list. The patient presented to the emergency department two hours ago with severe shortness of breath. He has a history of multiple admissions to the Coronary Care Unit for management for his cardiac amyloid disease. (ECG = rapid AF)
Medications on admission:
* atorvastatin 40 mg daily
* bisoprolol 2.5 mg daily
* furosemide (frusemide) 40 mg twice daily
* potassium chloride 1500 mg twice daily
* prednisone 10 mg daily
* rivaroxaban 20 mg daily
* spironolactone 25 mg daily
What are the key issues concerning your management of this patient?”
“A 63-year-old woman is booked for a pulmonary vein isolation for atrial fibrillation today. She is scheduled to be the first case on your afternoon list in the cardiac catheter laboratory. She describes episodic palpitations lasting 30 to 40 minutes up to five times a day with associated fatigue and exertional limitation.
Past medical history:
* Ischaemic heart disease – drug-eluting stent (DES) to proximal LAD 12 weeks ago for stable angina symptoms
* Hypertension
* Type 2 diabetes mellitus
* Obstructive sleep apnoea requiring nocturnal CPAP
* Elevated BMI (41 kg/m2)
* Chronic kidney disease - baseline eGFR 65 mL/min/1.73m2
Current medications:
* apixaban 5 mg bd
* aspirin 100 mg daily
* clopidogrel 75 mg daily
* dapagliflozin 5 mg daily
* bisoprolol 2.5 mg bd
* insulin glargine 20 units nocte subcutaneously
* irbesartan/hydrochlorothiazide 150 mg/12.5 mg daily
* pantoprazole 40 mg daily
* rosuvastatin 10 mg nocte
An ECG has been taken this morning. Her blood pressure is 118/79 mmHg and SpO2 98% (room air). What additional information would you like to obtain from the patient prior to commencing anaesthesia?”
A 57 year old man presents to your preanaesthetic admission clinic in a tertiary hospital. He is booked for an open extended right hemihepatectomy. He has had a previous anterior resection and has hypertension treated with 50mg atenolol daily. He has good exercise tolerance and there are no other significant clinical findings.
A 55-year-old woman with multinodular goitre presents for right hemithyroidectomy. She has a history of hypertension. Her current medications are propylthiouracil and enalapril. What are the clinical issues for anaesthesia in this patient?
A 38-year-old woman is booked for a total thyroidectomy. She has a large multinodular goitre with possible retrosternal extension. The patient is otherwise well and weighs 54 kilograms. You are to provide level 3 supervision for an Anaesthetic Provisional Fellow during this case. The Anaesthetic Provisional Fellow contacts you the day before surgery to discuss the patient. What are the key issues of the preoperative assessment with regard to the anaesthetic management of this patient?
You are the consultant for the Monday morning emergency list and are asked to anaesthetise a 60-year-old man for a laparotomy. He was admitted on Friday evening, having presented with vomiting and abdominal distension. He now has a tense abdomen causing him pain. The patient mentioned that he has stopped exercising for the past fortnight because of occasional chest pain and breathlessness on exertion. A preoperative electrocardiogram and abdominal X-ray have been performed. How will you manage this patient?
“A 35-year-old woman is booked on tomorrow’s gynaecology list in your hospital’s day surgeryunit. A copy of the operating theatre list is shown below.
DAY SURGERY UNIT LIST, Monday 23rd October 2017, Theatre 2
Start 08:00am
Patient Details Procedure Alert
Ms XX, 35 years
Hysteroscopic myomectomy and endometrial
polypectomy
Equipment: Bipolar resectoscope
BMI 33
Admit: Day Surgery Estimated time: 150 mins
Referral: Gynaecology Fertility Clinic
(Pre admission clinic 16/10/17)
What are your anaesthetic considerations in preparing for this case?”
“You are the Consultant Anaesthetist on duty for a regional hospital and receive a call from the General Surgeon at 10pm. She has a 16y.o. male with appendicitis who needs a laparoscopic appendicectomy as soon as possible. The patient has no past medical history but has had abdominal pain and been unwell for the past 3 days.
Observations Weight- 60kg; BP 110/80; HR 110; Temp 37.3; O2 sat 98% on air; RR 25
How will you decide when the patient is ready?”
“A previously well 45-year-old woman presents for pre-op assessment to your anaesthetic clinic, for an urgent Total Thyroidectomy booked for the following day on your list. She gives a 6-month history of unintentional weight loss of 18 kg, tremors and anxiety. She was diagnosed with Graves’ disease 3 months ago. Carbimazole was started but was discontinued after 2 months, secondary to bone marrow suppression. She refused to have Radioiodine 131I treatment due to concerns about radiation. She now complains of palpitations and finds it increasingly difficult to get out of bed due to weakness. Her pulse is fast and irregular, BP is 140/94 mmHg and SpO2 is 98% on air.
Her current medications are:
Metoprolol 12.5 mg ………………….. twice a day
Prednisolone 20 mg …………………. once a day
Cholestyramine 4 g ………………….. thrice a day
With respect to assessment, what are your priorities specific to this patient?”
“You are seeing a 40-year-old woman in the pre-anaesthesia assessment clinic for semi- urgent thyroidectomy. She presents with failed medical management of thyrotoxicosis.
Current medications:
Carbimazole 15 mg tds
Metoprolol 100 mg bd
Prednisone 10 mg mane
Temazepam 10 mg nocte
How will you assess this patient?”
“You are working in the day surgery unit of a large regional hospital and allocated to a gynaecology list with a registrar. The next patient on the list is a 23-year-old woman booked for a hysteroscopic myomectomy.
Past Medical History
Menorrhagia secondary to uterine fibroids
Anxiety
Medications
Ferrous sulphate
Fluoxetine
Past Surgical History
Hysteroscopic myomectomy 2019
What additional information would you like to obtain to assist in formulating an anaesthetic plan?”
An otherwise well 50 year old male presents for functional endoscopic sinus surgery (FESS). He has a history of a previous failed intubation under anaesthesia.
You are scheduled for an ENT list with an anaesthetic registrar. A 38-year-old male with a recent change in phonation was added to the list the day before. He has no significant history apart from heavy smoking. He is booked for microlaryngoscopy and biopsy and possible laser treatment of a vocal cord tumour (pictured). How will you assess this man’s airway?
You are asked to take over a case from a colleague who has to urgently leave theatre. The patient is a 45-year-old woman having a septorhinoplasty. As he leaves, he hurriedly informs you that the patient was intubated with a videolaryngoscope after three attempts with direct laryngoscopy. A few minutes after taking over the case, the surgeon requests that you drop the blood pressure to reduce bleeding. How do you respond to this request?
“You are the anaesthetist for the emergency list and your next case is a 50-year-old man with a diagnosis of nasopharyngeal carcinoma who has been undergoing radio and chemotherapy for his cancer. Over the last few weeks he has been having recurrent epsiodes of epistaxis and was brought to hospital last night after having a large epistaxis that has been controlled using a Foley’s catheter in the nasopharynx. The patient is in your anaesthetic room and the surgeon wants to perform urgent examination under anaesthesia and cautery of his nose. The following results are available (on wall and on desk):
Hb: 99 g/L MCV: 80 fl MCHC: 260 g/L WCC: 2.3 x109/L Plt: 97 x109/L
INR: 1.4
How will you proceed?”
You are working in a stand-alone private hospital. You are anaesthetising a urology patient for insertion of a ureteric stent, an urgent case which was added to the end of your ear, nose and throat list that afternoon. The Post Anaesthesia Care Unit nurse calls to say that your patient, who arrived in the unit 30 minutes ago, is now bleeding heavily from the nose. You recall that the case was an uneventful endoscopic sinus procedure performed under general anaesthesia with a flexible laryngeal mask. The patient had a history of ischaemic heart disease and was taking aspirin for a drug-eluting stent that was inserted two years ago. What is your initial response to this call?
“A 57-year-old farmer with a 30 pack year smoking history has been added to your preadmission clinic. He is booked for microlaryngoscopy and biopsy of a base of tongue lesion on your afternoon list. The nasendoscopy picture from the surgeon’s referral letter is shown below. The patient gives you an anaesthetist’s letter from 1997, when he underwent appendicectomy, which states “Unable to view glottis. Suggest use straight blade.” His only other prior surgery was a carpal tunnel release under regional anaesthesia two years ago.
His current medications are:
* fluticasone/salmeterol
* tiotropium inhaler
* salbutamol inhaler
* omeprazole
* aspirin
What issues concern you?”
“A 58-year-old male presents to the pre-anaesthetic clinic prior to a functional endoscopic sinus surgery (FESS) procedure.
His past medical history includes:
Obstructive sleep apnoea
Poorly controlled gastro-oesophageal reflux
Ischaemic heart disease with coronary stenting ten years ago
Ex-smoker (ceased ten years ago)
Weight 145 kg, body mass index 45 kg/m2
Current medications:
Metoprolol
Ramipril
Aspirin
Ranitidine prn
Please describe key areas of concern in your pre-anaesthetic management plan.”
“You are doing an ECT list at a private stand-alone facility. Your 2nd patient of the morning is a 55-year-old male having his first ECT titration for schizophrenia with bipolar disorder. He has a history of hypertension, hypercholesterolaemia, and is a current smoker. His medications are as follows:
amlodipine
lithium
olanzapine
rosuvastain
Discuss your preoperative approach to this patient?”
You are the Consultant Anaesthetist on duty, including responsibility for the Intensive Care Unit (ICU). In the evening, you receive a telephone call from the Emergency Department (ED) requesting your presence (your registrar is already occupied elsewhere). A 46-year-old man has been brought in by friends, having collapsed after feeling unwell for a few days. He now appears confused and is vomiting. Before you make your way to the ED, what key information do you need over the phone?
“A previously well 68-year-old man has been booked for percutaneous drainage of a pelvic collection, which has developed following radical prostatectomy three days ago. You have been asked to provide sedation for the procedure, in the interventional radiology suite, on the after-hours list. When you review him on the ward preoperatively he looks pale and unwell. His only complaint to you is of some suprapubic discomfort. His observations are:
Heart rate 100 /minute
Blood pressure 95/60 mmHg
SpO2 94% on room air
Temperature 38.6°C
What will you do whilst awaiting the procedure?”
“A 49-year-old woman presents in the evening with presumed ischaemic bowel for laparotomy. An arterial blood gas (ABG) has recently been performed in the emergency department.
Medical history:
* Ehlers-Danlos syndrome
* Chronic pain
Medications:
* Pregabalin SR 150 mg bd
* Paracetamol 660 mg tds
* Oxycodone/Naloxone 10/5 mg bd
* Oxycodone 5 mg prn
* Meloxicam 15 mg daily
* Pantoprazole 40 mg daily
ABG: Reference Ranges
pH 7.22 (7.35–7.45)
PCO2 30 mmHg (35–45 mmHg)
PO2 72 mmHg (80–100 mmHg)
HCO3− 12 mmol/l (22–26 mmol/l)
Base excess -15 mmol/l (-2–2 mmol/l)
Na+ 141 mmol/l (135–145 mmol/l)
K+ 3.1 mmol/l (3.5–5.0 mmol/l)
Cl- 105 mmol/l (98–06 mmol/l)
Lactate 8.2 mmol/l (0.0–2.0 mmol/l)
What are the likely differential diagnoses for this patient’s abnormal ABG?”
“An 80-year-old man sustained right-sided rib fractures after a fall and was admitted to the surgical ward of your regional hospital last night. His only other injury is a fractured right clavicle. You are asked to review his pain management.
His past medical history includes:
Hypertension
Dyslipidaemia
Chronic obstructive pulmonary disease
Chronic moderate renal impairment
Type 2 diabetes mellitus
Prostate cancer
Regular medications:
Atorvastatin 40 mg daily
Budesonide-formoterol (50 µg/3 µg) inhaler bd
Irbesartan 75 mg daily
Metformin 500 mg daily
Salbutamol inhaler prn
Despite his medical issues, he is still active, and cares for his wife in their home. Currently, his pain management is tramadol by intravenous infusion at 20 mg per hour, and oral paracetamol 1 g qid. You attend to assess him and notice that he is disorientated and confused, although cooperative. Why may he have become confused?”
“You are the anaesthetist covering the Intensive Care Unit and operating theatres for a rural hospital on a weekend. You are asked to assist in the management of a 55 year old female brought in by ambulance to the Emergency Department with nausea, vomiting, rigors and fever.
Medications: rivaroxaban, sotalol
Vital Signs:
HR 108, BP 90/40, RR 18, Temp 38.8, SaO2 96% room air, Weight 80 kg
Venous Blood Gas room air:
Reference Range
FiO2 0.21
pH 7.31 7.35-7.45
PCO2 30 (4) 41-51 mmHg (5.5-7.6 kPa)
PO2 45 (6) > 40 mmHg (> 5.3 kPa)
Hb 150 115-165 g/L
K 4.8 3.5-5.2 mmol/L
Na 144 135-145 mmol
Ca 1.1 1.12-1.32 mmol/L
Cl 105 95-115 mmol/L
HCO3 15 23-29 mmol/L
Base excess -10 -2 - + 2 mmol/L
Lactate 3.6 <1.3 mmol/L
What will you do on arrival in the Emergency Dept?”
“A 30 year old 50 kg male with severe uncontrolled epilepsy is booked for an awake craniotomy for mapping and treatment of epilepsy. He is on the following antiepileptic medications:
Phenytoin (Dilantin)
Clonazepam (Rivotril)
Levetiracetam (Keppra)
Whilst conducting your preoperative assessment on the neurosurgical ward, he suddenly has a grand mal seizure.”
A 58-year-old female presents to the Emergency Department with a severe headache and a Glasgow Coma Scale score of 13 (E3V4M6). She is diagnosed with subarachnoid haemorrhage and is booked for emergency coiling of her cerebral aneurysm in the radiology suite. You have been asked to assess her and noticed the following findings on her ECG. What preoperative instructions would you give the Emergency Department staff in the management of this patient?
A 28-year-old woman with Grade 2 subarachnoid haemorrhage has undergone clipping of a middle cerebral artery aneurysm. She was two days post-bleed and had been commenced on nimodipine preoperatively. She had been previously well. Intraoperatively, the course of the anaesthetic and clipping was unremarkable. Temporary clipping was used to facilitate definitive clipping of the aneurysm. Hypertension was induced with a metaraminol infusion at the time of temporary clipping. You wake the patient at the end of the procedure and proceed to the recovery room. Describe your handover and instructions to the post-anaesthesia care team.
You are the on-call anaesthetist in a tertiary hospital and called to anaesthetise a previously well 40-year-old female who presented four days previously to a rural hospital with sudden severe headache and nausea. She is now monitored in Intensive Care with a GCS of 13 and persisting headache. Investigations have revealed a cerebral aneurysm and the report states it is an anterior communicating artery aneurysm, Fisher Grade 2. The neuro-interventional radiologist wishes to coil the aneurysm. Outline your preoperative anaesthetic plan.
You are a newly appointed consultant at a hospital where you have not worked previously. You have been assigned to the Angiography Suite for a case tomorrow. A thirty-year-old male is scheduled for elective embolisation of an intracranial arteriovenous malformation prior to definitive surgery planned for the following day. He is otherwise well and on no regular medication. What are the important issues to consider in preparing for this case?
You are the on-call anaesthetist, and you receive a call from your registrar regarding an emergency case. A 58-year-old with metastatic lung cancer is scheduled for thoracic decompression-laminectomy for acute spinal cord decpmpression. He has back pain, diffculty voiding and bilateral sensory and motor loss in his lower limbs. Please outline the main concerns you have about this patient.
You are asked by the neurosurgical team to review a 45-year-old man in the Emergency Department who requires an urgent craniotomy. A neighbour found him unconscious at the base of a ladder. He is now conscious with a Glasgow Coma Score (GCS) of 13 (Eyes=3 Verbal=4 Motor=6) and has informed the emergency staff that he is a Jehovah’s Witness. The primary and secondary surveys are otherwise clear and a CT head has been performed. What do you see in this slice of the CT brain and bone window?
A 78-year-old woman is brought into the Emergency Department by ambulance having collapsed at home. On arrival she has a Glasgow Coma Score of 5 with a left sided hemiparesis. She is intubated in the emergency department. An urgent non-contrast CT is suggestive of a ruptured aneurysm. The aneurysm is unsuitable for coiling. Below is her CT scan. What clinical and radiological features may indicate a poor prognosis in this patient?
After taking your patient to the post-anaesthesia care unit (PACU), you are asked by nursing staff to review another patient. He is a 28-year-old man who has had a left craniotomy for excision of a tumour. The nurse has been unable to contact the anaesthetist who was involved with the case. The patient has been in the PACU for one hour. In the last ten minutes he has become increasingly restless, agitated, and hypertensive. His blood pressure, measured from a radial arterial line, is 200/100 mmHg. Outline your initial assessment and treatment in the PACU.
You are in the pre-admission clinic with a 55 year old woman who is scheduled to have a transphenoidal hypophysectomy. She has features of acromegaly, but apart from hypertension and anxiety, she is otherwise fit and well. BP: 160/90 Weight: 70 kgs Medications: Enalapril and Fluoxetine What specific preoperative assessment is required for this patient?
“Your first patient on the next day’s neurosurgical list is a 55-year-old female inpatient for excision of a meningioma. Prior to her admission with this problem she had been fit and well. She is to undergo a stereotactically-guided retrosigmoid craniotomy. Current medications:
Levetiracetam 500 mg PO bd
Dexamethasone 4 mg IV tds
Ondansetron 4 mg IV tds
Please describe the main focus of your preoperative assessment in this case.”
At the beginning of your day shift, a 54-year-old woman presents to the emergency department of your large tertiary referral hospital with an initial Glasgow Coma Scale (GCS) score of 12 (E3, M5, V4). A computed tomography scan demonstrates a subarachnoid haemorrhage. You have been asked to review the patient in preparation for planned coiling of her ruptured aneurysm in the interventional radiology suite. When you arrive in the emergency department her GCS is now 9. Her blood pressure is 280/160 mmHg. What will you do when you arrive in the emergency department?
“You are called to the Emergency Department at 12pm to assess a 75-year-old female with an acute anterior cerebral circulation ischaemic stroke. She has facial paresis, arm weakness and abnormal speech.
Her past history includes atrial fibrillation, type 2 diabetes and hypertension.
Her medications are:
irbesartan 150 mg daily
dabigatran 150 mg bd
dapagliflozin 10 mg daily
The onset of symptoms was 2 hours prior to admission. Her GCS is 15 and she is compliant with instructions. She has been scheduled for urgent clot retrieval. What will you do when you arrive in the Emergency Department?”
“You are reviewing a 44-year-old man for revision ventriculoperitoneal shunt for congenital aqueductal stenosis which was first diagnosed at age 13. This operation will be his fourth revision. He is an inpatient on the neurosurgical ward.
His current medications are:
* dexamethasone 4 mg bd orally
* omeprazole 20 mg mane orally
* levetiracetam (Keppra) 250 mg bd orally
Please comment on the CT scan and explain what specific information you would like in your assessment of this patient.”
“You are an anaesthetist working in a major hospital which includes neurosurgery. You are currently in the preanaesthetic assessment clinic reviewing a 52-year-old man for his suitability to undergo awake craniotomy for tumour resection. He presented with a headache and right-sided weakness two weeks ago. Since diagnosis, his oral medications are:
* dexamethasone 4 mg TDS
* levetiracetam 500 mg BD
Weight 110 kg, height 182 cm (BMI 33kg/m2).
What does this scan show and how will you assess this patient’s suitability for awake craniotomy?”
“A 34 year old primigravida is in need of a lower uterine segment Caesarean section (LUSCS) for obstructed labour. The patient has been keen to have a natural birth. She has been labouring for 18 hours. She was assessed to be fully dilated 2 hours ago with the baby in an occiput posterior (OP) position. On examination, you found the patient to be exhausted and distressed with contractions. The analgesia so far has been inhaled nitrous oxide (50% in oxygen). She is obese: her BMI is 42kg/m2
Vital signs:
Heart rate 120 beats per minute
BP 110/60 mmHg
Temperature 37.8°C.
There is macroscopic haematuria. The foetal heart rate is 170 beats per minute with acceptable variability. The patient remarked that she is very keen to stay awake for her surgery. What would be your choice of anaesthesia for this patient and why?”
A 22 year old pregnant woman (G1P0) living in a remote indigenous community is referred to a tertiary obstetric hospital for increasing shortness of breath and an arrhythmia discovered by the community nurse at her first antenatal visit at about 10 weeks of pregnancy. The obstetrician refers her to the pre-anaesthetic clinic for assessment and preliminary planning for her mode of delivery. How would you assess this woman? Her initial 12-lead ECG is available below (atrial fibrillation)
You are on call for obstetric anaesthesia at a rural hospital. You are rung by the obstetrician who asks you to come in urgently for a caesarean section for foetal distress. The patient is a 30 year old term primigravada with an epidural catheter in situ.
You are working in the maternity preadmission clinic. The obstetric team refers the following patient to you to formulate a delivery plan. She is a 20 year old at 20 weeks gestation with Down Syndrome and mild intellectual impairment. She also has a loud systolic murmur. She is accompanied by her mother who is concerned about her daughter’s labour pain. The mother would like her to have an epidural for her labour and delivery.
You are the consultant anaesthetist attending a High Risk Antejnatal Clinic. You have been asked to see a 24-year-old female with myotonic dystrophy who is 28 weeks pregnant (first pregnancy). She is currently on enoxaparin in 60 mg per day subcutaneously, due to a past history of pulmonary embolus (prior to this pregnancy).
You are asked to come to labour ward by your registrar who has had several failed attempts to perform an epidural for a 22-year-old primigravida. The patient has previously had spinal surgery. What will you do?
A 150 kg, 25-year-old primigravida with cord prolapse is wheeled into theatre without warning. The obstetrician wants to perform immediate caesarean section. How will you assess this woman for anaesthesia in this situation?
You are called to the Delivery Suite to review a 30-year-old G3P2 woman who is planning vaginal birth. Her first baby was delivered by caesarean section for foetal distress, and her second baby was born via an uneventful vaginal delivery. She is in severe pain, despite already having an epidural catheter in place. How are you going to approach the problem of controlling her severe pain?
You have been called urgently to the emergency department of a metropolitan general hospital. A 38-year-old pregnant woman at 36 weeks gestation has presented with a two day history of headache, and feeling unwell. While she was waiting to be seen, she had a convulsion. She is still fitting when you arrive, approximately ten minutes later. What are the key issues in this situation?
You are called to an emergency in the labour ward for a parturient at term who has collapsed. She is unconscious and pulseless on a labour ward bed with an intravenous line in place. What is your initial response?
“A 30-year-old primigravida at 33 weeks gestation has been referred to the high risk antenatal assessment clinic at your tertiary hospital. She is a recent migrant and speaks no English. She has mitral stenosis and pulmonary hypertension, and is in atrial fibrillation for which she is anticoagulated.Her medications include
- enoxaparin 80 mg twice daily 10am and 10pm
- digoxin 125 μg daily
- atenolol 50 mg daily.
How would you recommend her cardiac status be monitored for the rest of her pregnancy?”
“A 34-year-old multigravida (G₂P₁) at 32 weeks gestation has been sent by her obstetrician to the Delivery Suite of a tertiary obstetric hospital for management of her hypertension. In the obstetrician’s rooms, she was complaining of severe headache and her blood pressure was noted to be elevated. She presents to Delivery Suite with the following observations:
Heart rate 85/minute, noted to be regular
Blood pressure 170/110 mmHg
She appears agitated. She is NOT in labour. How will you assess this patient?”
The labour ward staff call you to urgently see a patient. She is a primiparous 24-year-old immigrant with very limited English. She had a vaginal delivery 2 hours ago and has collapsed on the way back from the toilet. She is breathing and only just rousable. The obstetric team has been called. The patient weighs 120 kg but has no other known medical problems. The pregnancy itself was uncomplicated, but she had a prolonged second stage of labour and forceps were used for the delivery. What are your priorities when you arrive?
A 30-year-old lady with a diagnosis of Antiphospholipid Syndrome (lupus anticoagulant positive) has a past history of lower extremity deep vein thrombosis, and 2 previous mid- trimester fetal losses. She is 150 cm tall. Her weight at the first antenatal visit was 95 kg. She has a twin pregnancy. What is the rationale for anticoagulation therapy in this patient?
You are called by the obstetrician to see a 28 year-old woman who has presented following an intra-uterine fetal death at 31 weeks gestation. The obstetrician wants to begin an induction of labour as soon as possible and the patient is asking about analgesia for labour. Which specific elements of the patient’s history, examination and investigations will help you in the anaesthetic management of this patient?
You are asked to provide a labour epidural for a 32-year-old woman, who is at full term in her first pregnancy. As you arrive the midwife informs you that the woman has had breathing difficulties for the last five minutes and is now difficult to rouse. How would you assess and manage this situation?
You are asked to attend a trauma call in the Emergency Department at your regional hospital. A 34 year old woman, 32 weeks pregnant, is being brought in by road ambulance after accidently driving her car at medium speed into a gum tree. How does her pregnancy affect the initial assessment?
You go to the Emergency Department to assess a 15 year old, 55 kg girl who has been booked for D&C after a missed abortion. She has become increasingly unwell over the last four days with fevers, lower abdominal pain and vaginal discharge. She was too unwell to get out of bed today so was brought into hospital by ambulance. She has been given two litres of intravenous crystalloid in the last two hours. She now is flushed and looks very unwell. She is conscious and orientated. Her vital signs are: HR 126 BP 70/45 RR 24 Temp 38.8 SpO2 97% on air Describe your initial management.
“You are the anaesthetist for the emergency list. The gastroenterology registrar has contacted you to book a patient for endoscopic retrograde cholangiopancreatography. The patient is a 38-yr-old pregnant woman at 35 weeks’ gestation, who has presented to the emergency department with acute ascending cholangitis.
Her vital signs are:
Blood pressure 83/49 mmHg
Heart rate 115 /minute
Respiratory rate 24 /minute
SpO2 95% (on room air)
Temperature 39.1°C
Her full blood count shows: (reference range)
Haemoglobin 126 g/l (110–165)
White cell count 31 x 109 /l (3.5–11.0)
Platelets 224 x 109 /l (140–400)
What other condition may have a similar clinical presentation in this patient and how would you exclude this?”
A pregnant woman at 36 weeks’ gestation presents in respiratory distress to the emergency department of your hospital. The obstetrician asks you to review the patient as her condition is rapidly deteriorating and he wants to deliver the baby. How will you assess this patient?
You are the on-call consultant anaesthetist and currently at home. At 7 pm you receive a call from your first year trainee anaesthetic registrar who is attending a referral in the emergency department. The patient is a 36-year-old G3P2 pregnant woman, with vaginal bleeding at 33 weeks gestation. Her previous deliveries were by elective caesarean section. An ultrasound scan at 28 weeks apparently showed a low lying placenta, but she has not had further follow-up. What further information do you need and how will you advise the registrar?
You are the consultant covering labour ward at a regional hospital, where the anaesthetist is notified of all high-risk admissions. You have been called to see Mrs Smith, a 36-year-old primigravida at 38 weeks gestation, who was admitted one hour earlier with severe pre-eclampsia. Her blood pressure on arrival was 200/100 mmHg. She was commenced on oral labetalol by her obstetrician, who was then called away to an emergency caesarean section. When you enter her room with the midwife, you find Mrs Smith semiconscious on the floor, leaning against her bed. What is your immediate management of this situation?
“You have been asked by the obstetric registrar to review a patient in the antenatal clinic. She is a 22-year-old primigravida who is at 26 weeks gestation, and has cystic fibrosis. Her current medications are:
Ivacaftor
Pancrelipase (Creon)
Azithromycin
Montelukast (Singulair)
Dornase alfa (Pulmozyme)
Omeprazole
Mannitol
Salbutamol
Her most recent pulmonary function tests are:
Measured % predicted
FEV1 (litres) 2.04 67
FVC (litres) 3.06 85
FEV1 / FVC 67%
How will you assess this patient in the antenatal clinic?”
An obstetrician calls you from delivery suite to request an epidural for a woman who is in spontaneous labour. She is a 38-year-old G3P2 at 35 weeks gestation. She has had a previous caesarean section for breech presentation. What additional information do you require?
You are the anaesthetist on duty for obstetric emergencies. The obstetric registrar calls to tell you there is a patient on the way to theatre. The patient is 34 weeks pregnant and in labour with twins. The first twin has a prolapsed cord. The patient is Cambodian and speaks very little English. What will you ask the obstetric registrar?
“A 30-year-old pregnant woman, G5P4, at 24 weeks gestation presents to a tertiary hospital with a two day history of upper abdominal pain associated with nausea, vomiting and anorexia. She had a pre-pregnancy weight of 105 kg, and a body mass index of 41.Her past history includes:
Laparoscopic gastric banding five years ago
Pre-eclampsia with all previous pregnancies
Gestational diabetes not requiring insulin treatment
On examination, she is found to be in mild distress with significant epigastric and left upper quadrant tenderness without peritoneal signs.
Her vital signs are:
Heart rate 105 /minute
Blood pressure 105/65 mmHg
Respiratory rate 28 /minute
SpO2 94% on room air
Temperature 37.2°C
The surgeon has booked her for diagnostic laparoscopy to rule out complications related to her gastric band. Discuss your assessment of this patient.”
You are called to the labour ward to provide analgesia for early labour in a 30-year-old woman who is at term in her second pregnancy. A lower segment caesarean section under general anaesthesia for breech presentation with her first pregnancy was complicated by awareness and a vaginal birth is planned on this occasion. Her spinal X-ray is shown (available in PDF). She was seen at the pre-anaesthetic assessment clinic during her pregnancy, and is requesting an epidural for labour analgesia. What features of the history interest you in determining how to proceed?
“You attend the emergency department in a large regional hospital to assist with a patient involved in a motor vehicle accident. The patient is a 15-year-old female, who is 32 weeks pregnant and was the backseat
passenger in a stolen car in which the two front occupants died at the scene. She was not restrained by a seatbelt when the car crashed into a tree at an undetermined speed 45 minutes earlier. Upon your arrival, the emergency consultant informs you of the primary survey results:
Airway
o patent
o cervical collar in situ.
Breathing
o respiratory rate 30 /minute
o SpO2 98% on oxygen by mask
o normal examination
o chest X-ray: no abnormalities
Circulation
o pulse 115 /minute, blood pressure 110/70 mmHg
o cool periphery with slow capillary refill
o pain on compression of iliac crests
o small amount bright vaginal bleeding.
Disability
o slightly confused
o complaining of abdominal pain
o Glasgow Coma Scale score 13
o pupils equal and reactive, limbs gross motor normal.
What diagnoses are you considering?”
A 29-year-old female presents for a suction dilatation and uterine curettage in a day surgery unit attached to a large regional hospital. It is her fourth miscarriage of an IVF pregnancy. The admission nurse calls to say the patient had a cup of tea two hours ago and is crying profusely. The surgeon would like to proceed as soon as possible. How would you manage this situation?
You are asked to anaesthetise a 30-year-old woman in a major regional hospital for a planned lower segment caesarean section at 38 weeks gestation. The indications are breech presentation and previous caesarean section for failure to progress. In her previous labour she had a patchy epidural block that was augmented with a low volume spinal for her lower segment caesarean section. The spinal worked well and the caesarean delivery was uncomplicated. You meet the patient on the day of surgery. The patient has regularly attended antenatal care and has had an uneventful pregnancy. She underwent laparoscopic appendicectomy 18 months ago and was deemed a Cormack and Lehane Grade 2 laryngoscopy. The operation is the first case of a planned caesarean list with three cases booked for the morning. The patient requests a spinal anaesthetic. Which complications will you discuss in your preoperative consultation?
“You are called to the delivery suite at 0300 hours by the obstetric registrar to assist with analgesia for a 16-year-old nulliparous patient in labour at 38 weeks gestation. She is screaming with pain. She has attended antenatal clinic appointments since 10 weeks gestation but missed the last 2 appointments. She was a social smoker till 20 weeks, and had mild oligohydramnios on a 32 week ultrasound with normal blood results at that time. Her mother is present as a support person.
Her current blood results are:
* haemoglobin 115 g/l (115-165)
* white cell count 10.5 x109/l (3.5-12.0)
* platelets 90 x109/l (150-400)
* normal liver function tests
* Her blood pressure is 150/95 mmHg, heart rate 110/minute, and SpO2 98% on room
air.
What information will be important to help you decide on appropriate analgesia?”
You are the duty anaesthetist in a tertiary hospital and are asked to help a registrar manage a patient during elective LUSCS who has had a combined spinal/ epidural and is bleeding after delivery of the baby. The patient is 30-year-old primigravida, who had a breech presentation and has delivered a 4.2 kg baby. The patient’s weight is 70 kg What relevant information do you need from the registrar?
It is the weekend and you are the duty anaesthetist providing supervision from the operating theatres for an anaesthetic registrar in the birthing suite. The registrar telephones you for advice. The registrar informs you that about 25 minutes earlier they inserted an epidural in a 27-year-old woman, who has no medical problems and takes no regular medication The patient is G1P0, and in established labour at 34 weeks gestation. She had premature rupture of her membranes six days ago and has received steroids and magnesium sulfate. Epidural insertion was difficult and now the analgesia is inadequate. What specific information do you need to know before providing advice to the registrar?
You are the on-call anaesthetist at a regional hospital, and are contacted by the urological surgeon who would like to book a 30-year-old G1P0 woman at 34 weeks gestation with renal colic for ureteric stent placement ± laser distal ureteric stone. What other information would you like from the surgeon whilst you are considering this case?
You’re at home in the evening and a junior registrar at your tertiary referral hospital calls you to inform you about a patient in the delivery suite. They have been informed by the obstetric registrar that a 34-year-old G3P2 woman at 39 weeks gestation is in early labour, and a spontaneous vaginal delivery is planned. Her pre - pregnancy body mass index was 35.3 kg/m2 (170 cm, 102 kg). She is otherwise well and a practising Jehovah’s Witness. How does her obesity influence your management of her care?
You are asked to cover an elective obstetric list, at short notice, due to colleague illness, in a tertiary care centre. Your first patient on the list is a morbidly obese primigravida (pictured) with twins at 36 weeks gestation. She expresses a strong desire to be awake for the delivery. Her weight is 150 kg, height 170 cm, and body mass index 51.9 kg/m2. How would you assess this lady for a regional technique for her proposed elective lower uterine segment caesarean section?
A 25-year-old woman has been brought into hospital by ambulance after being struck by a falling branch in a storm. You are advised that she is obviously pregnant and is in a confused state. You are asked to attend the emergency department as part of the trauma call team. In addition to the routine assessment of airway, breathing and circulation, what are the important aspects of assessing this patient?
As the on-call anaesthetist in a general hospital you are asked by a midwife to place an epidural for a labouring patient in the delivery suite. The patient is 42 years old, G6P0, and has had three in vitro fertilisation (IVF) attempts. She is 38 weeks’ pregnant and labour was induced because of reduced fetal movements. When you go to assess the patient, the midwife shows you the following baseline cardiotocogram (CTG). The obstetrician and your anaesthetic registrar are not immediately available because they are in theatre with another case. Here is the CTG (see PDF). Please interpret it. What would you do?
“A 22-year-old woman who is 32 weeks pregnant presents to the High Risk Obstetric Anaesthesia Clinic for assessment and delivery planning.
She has a history of
- illicit drug use and is currently using methamphetamine
- previous bacterial endocarditis
What are the specific issues you would explore further during the initial antenatal assessment?”
“You are the consultant covering labour ward and have been phoned on the obstetric emergency phone to attend the delivery suite as part of the obstetric emergency team in a large tertiary hospital. The patient is a previously well 30-year-old primigravida (38/40) having an induction of labour for prolonged premature rupture of membranes. Her husband had alerted the delivery suite staff that she had become unconscious and was having a seizure. She has an 18 gauge cannula insitu and was recently started on patient controlled epidural analgesia. The obstetrician, neonatal team and midwives are present when you arrive. The seizure has terminated. She is not responsive.
Her observations on your arrival are:
HR 76 bpm
BP 92/55
SaO2 86% on oxygen 6l/min
Outline your management on arrival”
“You have been asked to put in an epidural in a G1 P0 woman who has presented in spontaneous labour at 38 weeks’ gestation. She has a normal body habitus. She has no significant past medical history. Her antenatal history is unremarkable other than stable gestational thrombocytopaenia. An FBE was performed at 36 weeks gestation.
Reference Range
Hb 129 g/L (115–165)
WCC 11 x 10^9 /L (4–11)
Plt 88 x 10^9 /L (150–400)
When you enter the room, she is in obvious pain and states “Just put the epidural in!” How will you assess this patient for an epidural?”
A 28-year-old female, 39/40 pregnant, presents to a tertiary referral hospital in early labour post induction for fetal macrosomia. Her history is significant for morbid obesity (BMI 44) and labile severe asthma. You have been consulted for a labour epidural. What information would you clarify prior to obtaining consent?
A 32-year-old woman attends the obstetric anaesthetic assessment clinic, having been referred by the obstetric registrar. She is 24 weeks into her first pregnancy, and has a history of multiple sclerosis. She uses a walking stick. Her other medical history includes anxiety. There are no other relevant obstetric, medical or anaesthetic issues. She had an uneventful general anaesthetic for an appendicectomy at this hospital last year for which you have the anaesthetic record. There were no airway issues. She takes escitalopram and has monthly ocrelizumab infusions, which have been withheld in pregnancy. She has no allergies. She wishes to discuss analgesia in labour and anaesthesia should a caesarean or other operative intervention be required. What further information do you require to address the patient’s concerns?
“You are asked to provide analgesia for a 26-year-old woman who presented in labour to your regional hospital on a weekday. She has a three month history of shortness of breath on minimal exertion. She has not presented for any antenatal care during the pregnancy and the shortness of breath has not been investigated. Otherwise she has had an uneventful pregnancy.
Her observations are:
Pulse 110/min
BP 110/80 mmHg
Respiratory rate 24/min
SpO2 95%
Height 170 cm
Weight 75 kg
How will you assess this patient?”
You are the anaesthetist on-call in a large regional hospital. You have been asked to provide epidural analgesia to a 32-year-old parturient (gravida 1, para 0, 41+1 weeks gestation) after commencement of induction of labour for prolonged pregnancy. The midwife informs you that the patient moved from Mongolia two years ago and is not fluent in English, although her husband in fluent in English. She has had an unremarkable antenatal course. Her past medical history includes mild asthma (no regular treatment required). When you attend the patient, she is noted to be in established labour with some distress due to contraction pain. She is 148 cm tall and weighs 55 kg. How will you obtain consent for this patient’s labour epidural?
“You are the anaesthetist covering the obstetric theatre at a large regional hospital. A 30-year-old primiparous woman at 38 weeks gestation is booked for a category 2 caesarean section for a non- reassuring cardiotocograph (CTG). The patient has pre-eclampsia with a blood pressure of 160/100 mmHg while on antihypertensive treatment. She is not currently in labour. She is morbidly obese with a BMI of 58 (height 168 cm, weight 164 kg). When you review the patient, she tells you that she is needle-phobic and wants to “be asleep” for the procedure.
Current medications
Labetalol 40 mg q30 min IV
Magnesium 1g / hour IV
Methyldopa 500 mg PO QID
Nifedipine IR 10 mg PO
There are no known drug allergies. How will you approach this situation?”
“A 35-year-old primiparous woman is referred to your high risk obstetric clinic at 28 weeks gestation as she is a Jehovah’s Witness.
Medications:
* pregnancy multivitamin
* iron supplement
She has no known allergies.
Height 175 cm
Weight 83 kg
BMI 27 kg/m2
Outline your assessment of this patient.”
“You are the obstetric anaesthetist on duty at a major public hospital. A 22-year-old primigravida is undergoing an induction of labour at 37 weeks gestation for fetal intrauterine growth restriction. Her BMI is 17 kg/m2 Her medical history includes previous oxycodone substance use disorder, for which she takes Suboxone (buprenorphine-naloxone 32 mg/8 mg sublingually every second day). She has no other medical or obstetric history
She had an epidural catheter inserted in early labour by a colleague for analgesia. Although she has difficult venous access, an ultrasound-guided 18-gauge cannula was successfully sited in her cubital fossa. The morning handover indicates that the patient has required two clinician epidural top-ups overnight due to unsatisfactory analgesia. There is now reduced fetal heart rate variability on
cardiotocogram (CTG) and the obstetric team would like to place a fetal scalp electrode. The patient is refusing this intervention due to persisting discomfort. The obstetric team requests your assistance to improve her analgesia. What are your initial priorities in the management of her pain?”
A 53-year-old male with Type I diabetes mellitus presents with recent deterioration in his vision, secondary to proliferative diabetic retinopathy in both eyes. A posterior segment haemorrhage and retinal detachment is found in his right eye. The patient is consented for “scleral buckle/laser” and prefers to avoid a general anaesthetic. The surgeon is agreeable to this request. Outline your approach to anaesthetising this patient’s eye.
You are called by the ophthalmology registrar to assist with a 20-year-old male patient in the Emergency Department. The patient was involved in a nightclub fight and was glassed in the face; he is uncooperative and agitated. The ophthalmology registrar suspects a penetrating eye injury and asks you to provide sedation in the Emergency Department so he can examine the patient. How will you assess this patient?
“The next patient on your emergency list is a 55-year-old Japanese tourist. He has a large fishhook deeply embedded in his eye following a game-fishing accident that occurred four hours earlier. The ophthalmologist has listed him for exploration of the globe +/- vitrectomy and repair and wishes to take him to surgery as soon as possible. The patient does not speak English. Additional information is available from the emergency department record.
Past medical history:
Hypertension
Chronic obstructive pulmonary disease
Smoker
Obese
Obstructive sleep apnoea
Hereditary angioedema
Medications:
Amlodipine
Salmeterol inhaler
Omeprazole
He has no known drug allergies. What additional specific information do you require before proceeding to theatre?”
You are at home and the junior night anaesthetic registrar rings you at 0200 to let you know about a case which has just been booked. There is an 18-year-old man with a right open eye injury which the ophthalmology team want to explore within the next six hours. Your registrar tells you he has just started an appendicectomy. He hasn’t done any eye cases before and would like to know what he should do. The history from the ophthalmology registrar is that the patient had been out drinking, has allegedly been assaulted, fell to the ground and has a large facial swelling and an open eye injury and is alert. He has taken a photo. The patient is uncooperative but the registrar doesn’t think there are any other injuries and the patient is otherwise well. Your registrar would like to do the case after the appendicectomy. What will you say?
An 86-year-old woman is admitted for repair of a fractured neck of femur following a fall. Her surgery has been postponed for several days because of lack of theatre time. You are seeing the patient for the first time in the holding bay of theatre. She has a past history of hypertension and congestive cardiac failure. Her regular medications are metoprolol 25 mg daily and frusemide 40 mg daily. How will you proceed?
A 50 yr old woman booked for multilevel posterior laminar interbody fusion of her lumbosacral spine. She is hypertensive and has a history of paroxysmal AF. She has no other past medical history.. The surgeon expects the surgery to take 6 hours. Medications: Paracetamol, Apirin, Cilazapril, Metoprolol and Oxycontin What strategies would you use to minimise blood loss for this patient?
You are in the pre-anaesthetic assessment clinic. An anxious 18-year-old female with idiopathic scoliosis is booked to have corrective surgery in two weeks time. She is taking slow-release oxycodone 10 mg bd for back pain related to her scoliosis. Her chest X-ray and pulmonary function tests are shown here. Both she and her mother are worried about her having to be on a breathing machine after surgery. How will you address their concerns?
“A 75 year old lady has presented with a comminuted displaced fracture of the distal tibia and fibula.
Past history:
myocardial infarction 9 months ago
one coronary stent
uneventful and uncomplicated recovery
no symptoms since discharge”
You are asked to anaesthetise a 32 year old female for corrective surgery for severe kyphoscoliosis.
You are performing the preoperative assessment of a 4 year old child for tonsillectomy and adenoidectomy. The child has a history of night time snoring and mild asthma. On what aspects of this child’s preoperative assessment would you concentrate?
You are performing a preoperative assessment in the Emergency Department on a 5 year old girl, scheduled for intramedullary nailing of a femoral fracture, sustained after being hit by a car. The accident was 2 hours earlier and she has no other injuries. She has no allergies and weighs 25kg. Her mother is concerned about her having an operation as she has a history of asthma and has been unwell with an upper respiratory tract infection for 4 days.
A 4 year old boy presents to the Emergency Department with a history of coughing up blood and being unwell with a fever for 24 hours.
“A 5 week old infant presents to the Emergency Department obtunded with a 7 day history of projectile vomiting. The child was born at 35 weeks gestation. His current weight is 3.2kg. The following blood results are available immediately.
Na+ 129 mmol/l
K+ 2.8 mmol/l
Cl- 80 mmol/l
HCO3
- 37 mmol/l
Glucose 2.2 mmol/l”
A three-year-old boy is scheduled for tonsillectomy. He is admitted on the day of surgery. The admitting nurse notifies you that he has a “runny nose”. He is 100cm tall and he weighs 18kg (90-95th percentile for body mass index). How will you assess this child to determine whether to proceed with surgery?
An 18-month-old boy has been admitted for hypospadias repair. Surgery is estimated to take 90 minutes. He was born prematurely at 29 weeks gestation and has been diagnosed as having mild bronchopulmonary dysplasia. He had a viral upper respiratory tract infection five weeks ago, but is now back to his usual self. How will you assess respiratory status before anaesthesia?
A four-year-old boy with osteomyelitis of the tibia is on the theatre emergency list for debridement and washout. He has a family history of malignant hyperthermia. He has no intravenous access at present, and venous access looks difficult. How would you induce anaesthesia?
A six-year-old girl with diabetes mellitus is scheduled for elective repair of a right inguinal hernia as a day case procedure. What further information do you require to assess her suitability for day surgery?
The Emergency Department calls for your assistance. A 2 year old girl has just been brought in my ambulance after being hurt in an explosion on her family’s boat. On entering the resuscitation bay of the Emergency Department you see a crying and distressed child with obvious burns to her face, chest and arms. She is pulling off her oxygen mask, as the junior doctor attempts to obtain intravenous access. What will you do on arrival?
A 4-year-old boy is on the emergency list requiring laparotomy for a suspected bowel obstruction. He has been transferred overnight from a small district hospital. How will you assess him pre-operatively?
You are the anaesthetist for an ear, nose and throat list at a freestanding day surgery centre. A 4-year-old, 20 kg boy is scheduled for adenoidectomy. He has an obvious runny nose and cough. How will you assess his respiratory symptoms in your preoperative consultation?
You receive a trauma call from Emergency Department requesting your attendance. A 4-year-old girl has been brought in by ambulance after a fall from a bunk bed. She is screaming and agitated and is accompanied by her mother. What will your approach be?
“You are asked to anaesthetise a 6-week-old infant, born at 36 weeks, with a five day history of vomiting. The provisional diagnosis is pyloric stenosis. His current weight is 3.6 kg.
His laboratory data is shown below.
Na⁺ 137 mmol/L (135–145 mmol/L)
K⁺ 5.4 mmol/L (4.5–5.5 mmol/L)
Cl⁻ 104 mmol/L (99–107 mmol/L)
HCO₃⁻ 17 mmol/L (17–24 mmol/L)
Glucose 4.5 mmol/L (3.6–5.4 mmol/L)
Creatinine 55 µmol/L (18–48 µmol/L)
How would you evaluate the volume status and hydration of this infant?”
You have been called by the on-call paediatric surgeon to book a 10-year-old boy for rigid bronchoscopy for suspected foreign body removal this evening. You are told he is obese and intellectually impaired. He has presented with a cough and shortness of breath following a choking incident earlier today while playing with Lego. He is currently in the Emergency Department and is uncooperative. Outline how you will decide how urgently to proceed to Theatre.
“A 5-year-old girl with a history of being “unwell for several days” has been seen by the surgical registrar in the Emergency Department. He believes she has a perforated appendix and has rung you at 5pm to discuss the case before booking her for a laparoscopic appendicectomy later that evening.
Her vital signs are:
Temperature 38.5 °C
HR 130 /min
BP 80/55 mmHg
SaO2 95% on room air
What else would you like to know from the surgical team and how will you further assess this patient?”
You are seeing an 8-year-old boy in the Pre-Admission Clinic. He is scheduled for adenotonsillectomy. He has mild obstructive sleep apnoea and mild asthma. He weighs 56 kg. His mother tells you he was anaesthetised one year ago but there was trouble finding a vein and he became uncooperative at induction. The mother was told he should be given a premed before his next anaesthetic. What will you do?
You are the consultant anaesthetist on duty for emergency cases. You are called to the operating theatre lobby to find a four-year-old boy accompanied by emergency department staff and a neurosurgeon. The child has come directly from the CT scanner. The scan shows a large left subdural haematoma with midline shift. The surgeon wishes to evacuate the haematoma as soon as possible. The patient weighs 18 kg, is in a hard collar, is not intubated and has one peripheral cannula. Outline your immediate management of this situation.
You are the on-call anaesthetist in a peripheral metropolitan hospital. The emergency department consultant asks for your assistance with intravenous access and blood collection in a ten-day-old girl, who has been brought in by her parents with a history of not feeding and being unwell. When you see the child she is listless and difficult to rouse, and is making grunting noises with respiration. How will you assess this child?
An 8 year old boy, weighing 40kg with a past history of asthma, is scheduled for tonsillectomy. He has travelled a long distance from his home and has been cancelled three times previously due to lack of beds. During your pre- operative assessment you note he has a cough and a runny nose. What further information do you require in order for you to make a decision on proceeding or not with the operation?
Your registrar is called to the emergency department of a small rural hospital to help the emergency staff with a 10-month-old girl brought in by her parents with apparent respiratory distress. Your registrar calls you to provide assistance. What will you want to know from him?
“You are the senior duty anaesthetist in a major metropolitan hospital. You receive a call from the emergency department regarding a four-year-old girl, who has presented with post-tonsillectomy haemorrhage.
Her initial surgery was performed in a private hospital seven days earlier. Her vital signs are:
Blood pressure 110/70 mmHg
Heart rate 110 /minute
Respiratory rate 30 /minute
SpO2 99% (on room air)
Temperature 37.3°C
Describe your initial assessment and management in the emergency department.”
It is 1100 hours on Monday morning, and you are the anaesthetist for the emergency list at a large regional hospital. The paediatric surgical registrar has booked a 12-year-old boy for drainage of a large pleural effusion. He thinks the effusion is secondary to pneumonia and tells you that the child looks unwell. The interventional radiologist will insert the chest drain, under ultrasound guidance, in the radiology suite. You send your anaesthetic registrar to the emergency department to assess the child. You are able to view the child’s chest X-ray on the radiology information system.Having seen this chest X-ray, what information will you require from the anaesthetic registrar?
“You are the on-call consultant anaesthetist in a tertiary referral hospital and are called to the emergency department. A previously fit and well 70 kg 16-year-old girl has presented with acute shortness of breath. She has been generally unwell for three months, with a cough, increasing breathlessness, and occasional fevers. She takes no regular medication.
Her observations are:
Blood pressure 100/70 mmHg
Heart rate 140 /minute
SpO2 92% (room air)
Respiratory rate 40 /minute
On entering the emergency department you see her chest X-ray on the viewing screen. What does the chest X-ray show, and how will you assess this patient?”
You are the on-call consultant anaesthetist, working on-site at a large regional hospital with a paediatric surgical service. The surgical registrar phones and asks you to review a four-year-old boy on the children’s ward who has an acute abdomen, for which his consultant would like to book urgent surgery. The boy has no significant past medical, family or social history. Describe your initial management.
You are called by the emergency department staff of a children’s hospital. A six-year-old girl is being brought by ambulance following a fall from a horse. She was not wearing a helmet. In the ambulance, her eyes are shut, she is moaning and not responding to pain. Her pupils are reacting sluggishly to light, but they are equal and not dilated. How will you prepare for her arrival in the emergency department?
You are the anaesthetist for a paediatric ENT list in a private hospital. The operating theatre list has been sent to you by the hospital on the day before surgery. On the list is a 25 kg four-year-old boy who has obstructive sleep apnoea and is scheduled for tonsillectomy. What are you going to discuss with the parent(s) during the preoperative phone call?
You are the on-call consultant anaesthetist at a large regional hospital. A four-year-old girl has been booked on the emergency list for laparoscopic +/-open appendicectomy. The surgeon tells you that the patient has had abdominal pain for three days, has been vomiting for one day, and looks unwell. She is currently on the children’s ward and intravenous fluids are in progress. How will you assess her preoperative fluid requirement?
You are the on-call consultant anaesthetist at a large regional hospital. You receive a telephone call from the emergency department registrar requesting your assistance. An 18-month-old girl has just been brought in by ambulance following a fall from her high chair onto a tiled floor. The general surgeons have also been requested to attend. On your arrival in the resuscitation bay you see a crying child with obvious swelling on the right side of her head. The emergency department staff are attempting to gain intravenous access. What will you do?
You are the on-call anaesthetist at a large regional hospital. A three-year-old girl has been admitted to the children’s ward with a diagnosis of lobar pneumonia, and a parapneumonic effusion which appears loculated on ultrasound examination. You have been asked to provide anaesthesia for the insertion of an intercostal catheter to drain the effusion. Her chest X-ray is shown here. Outline your assessment of this child in preparation for this procedure.
You are part of a surgical outreach service providing dental care for remote Aboriginal Australians. You are working in a geographically isolated 30-bed district hospital managed by local rural general practitioners, 1000 km from the nearest tertiary referral hospital. There are two operating theatres but no intensive care unit. Next on the list is a developmentally delayed 12-year-old boy booked for multiple dental fillings and one tooth extraction. He and his parent attended the preadmission clinic yesterday, and his pre-anaesthesia assessment was conducted by you. He is otherwise well and weighs 40 kg. You now meet him and his parent again in the anaesthetic bay. Unfortunately, he is crying and hiding under the sheets on the trolley. What is your plan to facilitate a smooth induction for this child?
You are at home on call for a tertiary paediatric hospital when your third year registrar contacts you for advice about managing a six-year-old boy with autism, who had a tonsillectomy a week ago and now has some bleeding. What information will you seek from your registrar?
Your first patient on the Monday morning radiotherapy list is a two-year-old boy beginning his last week of daily treatments for a retinoblastoma. Unfortunately his central line had to be removed after his last treatment due to persisting low grade fever. Following a discussion between the duty anaesthetist and parents on Friday a gaseous induction has been planned for today. You are informed by the admitting nurse that the boy has developed a runny nose over the weekend. What are your concerns regarding this child for this procedure?
You see a two-and-a-half-year-old girl in the anaesthetic clinic. She has cerebral palsy since birth and is booked for follow-up magnetic resonance imaging as part of review of her developmental progress. The parents are concerned because they have read about the potential adverse effects of anaesthesia on neurodevelopment. Describe your management of this consultation.
You are wheeling your last patient into the Recovery Room of a private hospital late in the day. You notice a colleague having difficulty with a child’s airway. The child’s oxygen saturation is 65% (and falling) and his heart rate is 60/minute. What will you do?
You’re the consultant covering the intensive care unit at a regional country hospital. You receive a call from an emergency consultant requesting you review a 17 year old girl who has been brought in by ambulance with ‘an acute exacerbation of asthma’, for potential admission to intensive care. You attend with your registrar. What are the important examination findings?
You are asked to see an 8-year-old boy who is, booked for open fixation of a fractured femur on the emergency list. You are in the Emergency Department about to take a history from his mother when he becomes unresponsive. What do you do now?
“You are the onsite anaesthetist at a tertiary paediatric hospital on a Saturday morning. You are contacted by the surgeon to inform you she has booked a 14-year-old girl with cystic fibrosis on the emergency list for surgery later today for a laparoscopic appendicectomy.
Her vital signs are:
Weight 45 kg
Temperature 38.5°C
Respiratory rate 20 /minute
Heart rate 120 /minute
Blood pressure 90/50 mmHg
Oxygen saturation 91% (room air)
Her medications are:
Azithromycin
Dornase nebuliser
Budenosine/fornoterol puffer
What are your priorities in her preoperative assessment?”
You are the anaesthetist in the pre-admission clinic of a paediatric hospital. The next patient is a 4-year-old boy with Down syndrome booked for bilateral congenital talipes equinovarus repair with osteotomy. He weighs 25 kg. Outline your assessment of this child.
You are the duty anaesthetist on for the emergency list at a tertiary referral hospital. A four-year-old boy has arrived by ambulance to the emergency department with a displaced supracondylar fracture of the right humerus. He is scheduled to come to theatre for open reduction and internal fixation of his humerus immediately due to neurovascular concerns. You have been informed that there is a family history of malignant hyperthermia. What specific information in the history and examination will you seek to determine your anaesthetic plan?
“You are the on-call anaesthetist at a major regional hospital. You are asked to attend a two- year-old child in the emergency department (ED) who has a grossly distended abdomen and whom the surgical Fellow wants to bring to theatre for an urgent laparotomy. On arrival in ED, you find the child in a cubicle with his mother. He is slightly mottled in appearance, lethargic and difficult to engage, and appears to have increased work of breathing. The mother gives a history of 24 hours of vomiting with several nappies of loose bowel motions. The child has had a 22G cannula inserted by the ED registrar who has since been called away. No fluids are running. His observations are:
Temperature 37.5℃
Blood pressure 85/50 mmHg
Heart rate 140 /minute
Respiratory rate 34 /minute
SpO2 94% on room air
What is your assessment of this patient’s clinical condition?”
“A six-year-old boy with cerebral palsy is on your morning eye list at a paediatric referral hospital. He is listed for strabismus surgery. You are reviewing his case notes when the admissions nurse comes over to tell you his mother has requested that her son receives a different premed before surgery today. You find previous anaesthetic charts relating to a series of upper and lower limb Botox injections. On the most recent it is documented that oral midazolam 8 mg was administered as premedication. It is also noted that the child was very anxious on arrival in theatre and became distressed and uncooperative during his inhalational induction. His weight is 16 kg.
Current medication:
Omeprazole 20 mg daily
Baclofen 20 mg bd
Clonazepam 1.5 mg bd
Carbamazepine 100 mg qid
Diazepam 2.5 mg prn
How will you approach your anaesthetic assessment of this boy?”
“You have been called to the emergency department to assess a 12-year-old boy who has been booked for an urgent scrotal exploration for suspected testicular torsion. On arrival you notice the child has Down syndrome, is obese and has a cough. His mother tells you that shortly after having his breakfast he suddenly started crying inconsolably, placing his hands over his genital area. She also tells you she is quite worried about him having surgery as a cold has exacerbated his asthma. You see the following information on his chart:
Height: 140 cm
Estimated body weight: 63 kg
Medications:
Budesonide 200 µg daily
Salmeterol 50 µg bd
How will you assess this situation?”
A 6-year-old child is scheduled for adenotonsillectomy in a regional hospital. You are seeing him for the first time on the morning of surgery. He is 35kg and on no medications. What are the important factors in your decision to proceed with this operation in a regional centre with this child?
You are the on-duty anaesthetist at a large regional hospital which provides general paediatric services. A 5-year-old boy has been knocked off his scooter in the driveway by his mother’s four-wheel drive (4WD) car. He is en route to the hospital by road ambulance, and the estimated time of arrival is in ten minutes. You have been called to the emergency department to assist in his resuscitation. What would you do in preparation before the child’s arrival?
“A 4-year-old girl presents on your ear, nose and throat (ENT) list at a small regional hospital for elective bilateral myringotomies and insertion of grommets. She has a history of recurrent middle ear infections and has also been diagnosed with asthma and atopic dermatitis. She is allergic to peanuts. On the preoperative-assessment questionnaire her mother has stated that the child currently has a runny nose and a cough.
The girl’s medications are:
fluticasone 100-200 mcg daily (via metered-dose inhaler plus spacer)
salbutamol 200-400 mcg PRN (via metered-dose inhaler plus spacer)
What further information do you want from the child’s mother?”
“A 12-year-old boy has been a late addition to your dental list today at a small private hospital. He is scheduled for removal of three carious teeth under general anaesthesia.
Past medical history
attention-deficit hyperactivity disorder (ADHD)
adenotonsillectomy aged 6 years
Medications
methylphenidate (controlled release) 60 mg mane, not taken today
Allergies
Nil known
Observations on admission
Height 152 cm
weight 45 kg
temperature 36.6 degrees Celsius
pulse rate 96 bpm
blood pressure 106/62 mmHg
SaO2 99% (room air)
In your preoperative review of this patient with his father, what specific information on history and examination will you seek?”
You are reviewing a child in the pre-anaesthetic clinic of a tertiary paediatric centre. He is a 20 kg 23-month-old boy with moderate global developmental delay and seizure activity on EEG. You are booked to anaesthetise him for an MRI brain scan the following day. Describe your approach to the assessment of this child.
“It is 0700 hrs and you are the anaesthetist rostered to a morning ENT list in a tertiary referral hospital. The first case is a three-year-old boy for tonsillectomy. The referral states he has attention deficit hyperactivity disorder (ADHD), asthma and had bilateral grommets inserted one year previously. He presents with his mother.
His current medications are:
dexamphetamine
clonidine
budesonide
salbutamol
What are the key features of your assessment?”
You are on call at the Children’s Hospital. At 1730 hours you are asked to review a 7-year-old girl with Down syndrome in the Emergency Department (ED) who sustained a supracondylar fracture of the left humerus whilst playing on a trampoline at a birthday party. She is booked on the emergency list for closed reduction and percutaneous pinning of the fracture. On your arrival in ED you are informed that the girl has received intranasal fentanyl 50 mcg following one unsuccessful attempt to gain IV access. The dose of fentanyl was based on a weight of 31 kg which had been documented in the girl’s case notes at a recent outpatient clinic appointment. How will you approach your anaesthetic assessment of this girl?
“You are the duty anaesthetist in a regional hospital. You are called to the Emergency Department by the surgical registrar to review an 8-year-old boy who has been booked for an urgent appendicectomy. The child has been previously well and presents with a three day history of abdominal pain and vomiting. An ultrasound has been performed which suggests appendicitis. The surgical registrar would like to operate as soon as possible as he thinks the child is showing signs of sepsis.
Initial information on booking:
weight 25 kg
pulse 150 bpm
BP 80/40 mmHg
RR 45 bpm
temp 37.3° C
FBC:
Hb 120 g/L (110- 155)
WCC 17 x109 /L (4-11.0)
Plt 250 x109/L (140-400)
Electrolytes and renal function:
Na 133 mmol/L (133-144)
K 3.5 mmol/L (3.6-5.3)
Cl 104 mmol/L (97-110)
Bicarbonate 8 mmol/L (22-29)
Urea 5.0 mmol/L (2.7-7.8)
Creatinine 35 µmol/L (20-44)
What specific features would you look for in the assessment of this child?”
You are the on-call consultant in a regional hospital. The anaesthesia registrar calls you at 22:30 hours reporting that there is a 14-month-old child in the emergency department with a foreign body in the mid-oesophagus visible on chest X-ray. There is an ENT consultant available to take the child to theatre. How would you assess the urgency of this case?
You have been asked to take over an elective list at a tertiary paediatric hospital as the regular anaesthetist is delayed in the postanaesthesia care unit by a postoperative airway event. The next patient is a five-year-old First Nations boy (weight 28 kg) from a regional centre who is accompanied by his grandmother who is his legal guardian. He is booked for adenotonsillectomy for the management of obstructive sleep apnoea. His surgery was previously cancelled three months ago. The anaesthetist had just administered ketamine and midazolam premedication to this child when he was called away. What further information do you require to proceed with this case.
“It is the weekend and you are the anaesthetist on-duty at a remote regional hospital. You receive a request from the Emergency Department (ED) registrar to assist in the management of an 8-year- old boy who has hit a tree whilst mountain biking. The boy has an open mid-shaft fracture of the femur and the ED staff have been unable to secure intravenous access.
Past Medical History:
* asthma
* attention-deficit/hyperactivity disorder
Medications:
* methylphenidate 20mg daily
* salbutamol 100 mcg prn
* fluticasone 100 mcg daily
What major issues are you going to consider in the management of this child?”
A 2-year-old girl weighing 13 kg presents for bilateral strabismus (squint) surgery at a stand-alone day surgery facility. She was born at 34 weeks gestation weighing 2.2 kg. How will you establish she is suitable for day surgery?
A 48-year-old man was admitted to a burn centre 24 hours ago with self-inflicted flame burns to 65% of his body, particularly affecting the upper body including the head and neck region. Initial fibreoptic bronchoscopic examination revealed severe inhalational injury. He is currently intubated in the intensive care unit on an FiO2 of 0.4, and is on haemofiltration to treat hyperkalaemia. His current blood results are as shown below: (see PDF) The surgeon would like to perform debridement of burns on the emergency list within the next two hours. What are the main areas of concern?
Your morning list has a 60-year-old woman with a large loculated malignant left pleural effusion for video-assisted thoracoscopic surgery and pleurodesis. Her intercostal catheter was removed three days ago due to poor drainage. This morning’s chest radiograph reveals complete “white-out” of the left side of the chest. She is dyspnoeic at rest and her SaO2 is 93% on 2 l/min oxygen via nasal prongs. What do you want to know about this patient prior to surgery?
A 55-year-old man is admitted to hospital with acute renal failure (creatinine > 400 µmol/L). Investigations reveal that he has large retroperitoneal and anterior mediastinal masses, suspected to be lymphoma. He is booked for urgent bilateral ureteric stents to relieve his acute renal failure. How will you assess the implications of the mediastinal mass for your anaesthetic plan?
A 60-year-old man with chronic obstructive pulmonary disease underwent lung volume reduction surgery four days ago. Over the last few hours his oxygen requirements have increased. He is on the ward, is febrile and is receiving oxygen via a non-rebreathing mask at 10 L/min. Please describe this chest X-ray, which has just been taken.
You are asked to review a 40-year-old man with pulmonary alveolar proteinosis, in the preadmission clinic. The patient’s only other comorbidity is asthma, for which he takes regular salbutamol and budesonide by inhaler. The patient is scheduled for whole lung lavage next week. A recent chest X-ray is available. What does the X-ray show? What assessment and preparation is necessary for this patient?
You are asked to see a 65 yr old, 125 kg man with oesophageal cancer who has had an Ivor Lewis oesophagectomy. He is un-well day 4 postop, and has had a Barium swallow showing contrast down bronchus. He requires surgery. What are the important things you would like to know before anaesthetising this man?
“You arrive in the emergency department to assist with the management of an acutely unwell 76-year-old man, who has re-presented to hospital a week after thoracoscopy and talc pleurodesis for a non-resolving left-sided malignant pleural effusion. Two years ago he was diagnosed with lung cancer, which was treated with chemotherapy and radiotherapy. Twelve months ago he developed atrial fibrillation and also suffered a mild left cerebrovascular accident, with no residual deficit. He has more recently been referred to the Palliative Care team who have commenced him on pregabalin and tramadol because he is intolerant to morphine, fentanyl and oxycodone. His current medications are: digoxin 0.125 mg daily warfarin 4 mg daily pregabalin 150 mg bd tramadol (sustained release) 150 mg bd
His vital signs are:
Blood pressure 83/41 mmHg
Heart rate 125 /minute
Respiratory rate 26 /minute
SpO2 93% (O2 6 l/minute via Hudson mask)
Temperature 36.1°C
Please comment on his chest X-ray and how it will influence your initial management.”
“You are seeing a 72-year-old man, scheduled for right upper lobectomy for resection of a right upper lobe carcinoma, on the morning of his surgery. At a chest case conference he was assessed as suitable for surgery, but he has not attended a pre-anaesthesia assessment clinic. From the limited information available on the hospital’s electronic medical records and the case conference letter, it appears he is on quinapril for hypertension, but has no other medical problems. Investigations located on the electronic medical system are shown here.
Pulmonary function tests:
Most recent blood tests: (Reference range)
Haemoglobin 132 g/l (130–170 g/l)
White cell count 7.9 x 109/l (3.5–11 x 109/l)
Platelets 78 x 109/l (150–140 x 109/l)
What is your interpretation of his pulmonary function tests?”
A 58-year-old woman with an undiagnosed acute infiltrative lung disease is scheduled for lung biopsy on your thoracic list. She was admitted to hospital five days ago. For the last three days she has been intubated and ventilated in the Intensive Care Unit. She has not improved despite high dose steroids. Her chest X-ray is shown here. What are the issues that concern you?
“The first patient on your emergency list is a 60- year-old man for palliative video-assisted thoracoscopic surgery using three ports. He has a history of mesothelioma and hypertension, is 174 cm tall and weighs 62 kg.
His current medications are:
* · irbesartan 300 mg daily
* · metoprolol 50 mg bd
* · paracetamol 1 g qid
* · oxycodone (slow release) 40 mg bd
* · pregabalin 150 mg bd
* · escitalopram 20 mg daily
Please comment on this man’s chest X-ray (see PDF)”
“You are a consultant working at a tertiary referral hospital. A 70-year-old man has been added to your elective thoracic list for the next day for an open minithoracotomy and wedge resection of a left-sided lung mass. The patient has a 50 pack-year smoking history. He has a background of chronic obstructive pulmonary disease with no recent infective exacerbations. Three years ago he had a left mandibulectomy and radiotherapy for a squamous cell carcinoma of the floor of his mouth.
He weighs 65 kg and is 175 cm tall.
Current medications:
Tiotropium bromide – 2 puffs daily
Salbutamol inhaler prn
How will you assess his airway?”
“A 45-year-old female inpatient at your tertiary hospital was admitted with increasing shortness of breath. Her imaging and investigations confirmed a right sided empyema. Her surgeon indicated that it is complex and not amenable to percutaneous drainage so has booked her on your list for a right thoracotomy and decortication. Past medical history:
systemic sclerosis (scleroderma)
interstitial lung disease (pulmonary fibrosis)
Raynaud’s phenomenon
gastroesophageal reflux
Current medications:
omeprazole 40mg mane
mycophenolate 1g bd
amlodipine 10mg mane
ceftriaxone 1g intravenously bd
metronidazole 500mg intravenously bd
How will you assess her cardiorespiratory function in preparation anaesthesia with one-lung ventilation?”
“You are seeing a 64-year-old man in the Preadmission Clinic who is booked for a left lower lobectomy for non-small cell carcinoma. He is on your list in two weeks’ time.
Past Medical History:
* hypertension
* type II diabetes mellitus
* paroxysmal atrial fibrillation
Medications
perindopril 2mg bd
rivaroxaban 20 mg daily
metoprolol 50mg bd
empagliflozin 10mg daily
He currently smokes 1 packet cigarettes per day and has a 42 pack-year history. Height - 177cm Weight - 74 kg BMI 23.6 kg/m2 His chest CT scan is displayed below. How will you assess this patient’s suitability to undergo a lobectomy?”
You are the anaesthetist on duty and receive a call: “A 64 year old man has presented with severe abdominal pain and has a leaking abdominal aortic aneurysm diagnosed by computerised tomography (CT). His BP is 90/40 mmHg and his haemoglobin level (Hb) is 74g/L. He also has ischaemic heart disease.” He is in transit from radiology to your operating theatre. What is the essential information you require before proceeding?
“A 65-year-old man has been involved in a head-on car crash (both cars travelling at 50km/h) the day previously, and is currently in ICU intubated & ventilated. He was restrained but with no airbag. He has already had CT scans of head, neck, chest, abdomen, and pelvis, and his known injuries include:
o a thoracic aortic transection/rupture which currently forms a contained pseudoaneurysm,
o a closed head injury (GCS 13 (E3V4M6) at scene) with a left frontal lobe cerebral contusion on CT but no collection/mass effect
o rib fractures (3rd, 4th & 5th on right) and pulmonary contusions.
His only usual medication is Metoprolol CR 190mg daily for hypertension. You are to take him to Angiography/Radiology for endoluminal stenting of his aorta. What are your considerations for safe transport to the radiology suite?”
A 70-year-old man presents for elective open abdominal aortic aneurysm (AAA)repair. What history do you want from this man in the pre-admission clinic?
“A 76-year-old woman presents with right lower limb ischaemic pain and gangrene of her great toe. She is scheduled for urgent right femoro-popliteal bypass on your list tomorrow. Two months previously, she developed acute myocardial infarction that was
managed with a drug-eluting left anterior descending coronary artery stent. She also has a history of hypertension. What are the key issues for this patient requiring urgent femoro- popliteal bypass surgery?”
An 89-year-old woman with peripheral vascular disease has been admitted to hospital for management of severe rest pain in her right leg and and a non-healing ulcer on her right foot. She had a femoral artery stent placed nine months previously, but continues to have severe claudication. She has type 2 diabetes treated with insulin, and is taking clopidogrel. A harsh ejection systolic murmur has been noted on examination. She is being considered for a below knee amputation. You have been asked to review her on the vascular ward regarding her suitability for the proposed operation. Briefly describe your main concerns about anaesthetising this woman.
A 70-year-old man with type 2 diabetes who is on insulin, has presented with chest pain. He has undergone coronary angiography which has confirmed diffuse coronary disease. The femoral artery has been damaged during the procedure and his leg is now ischaemic. The vascular surgeon insists that urgent surgery is required. What key information do you need from the surgeon?
“A 74-year-old man was admitted to the ward 2 days ago with acute transient ischemic attacks (TIAs). He is booked for a left carotid endarterectomy (CEA) tomorrow.
He has a history of:
Ischaemic heart disease
Type II diabetes mellitus
Recurrent transient ischaemic attacks
His current medications are:
Aspirin 100mg daily
Clopidogrel 75mg daily
Atorvastatin 40mg daily
Human Insulin 16 IU twice daily s/c
Metformin 500mg daily
Internal carotid artery ultrasound report:
Left side 70-99% stenosis
Right side 50-74% stenosis
Please comment on this patient’s most recent ECG.”
“A 79-year-old woman with a painful ulcer on her right heel is scheduled for an angiogram and possible angioplasty or stenting of her posterior tibial artery next week. She presents to pre- anaesthetic clinic for assessment. The procedure would take about 60 minutes. She is very anxious but does not wish to have a general anaesthetic. She stopped smoking 6 months ago. She has a BMI of 35.
Her medications are
Gliclazide
Metformin
candesartan hydrochlorthiazide12.5mg digoxin
paracetamol
oxycodone slow release frusemide
potassium chloride aspirin
esomeprazole
She has no known drug allergies.
Please comment on her ECG below (provided see PDF)”
“The vascular surgical registrar at your teaching hospital informs you that they are scheduling a 77-year-old gentleman on your interventional radiology list for tomorrow. The patient needs urgent right iliac artery stenting.
The patient has:
* hypertension
* ischaemic heart disease with old myocardial infarction
* renal impairment
His current medications are:
* aspirin 100 mg daily
* atorvastatin 80 mg daily
* carvedilol 25 mg bd
* quinapril 20 mg daily
* vitamin D3 1.25 mg once a month
His renal function tests are shown. Please discuss the major issues concerning this case.”
“You are asked to review a 76-year-old man for planned thoracic endovascular aortic repair (TEVAR) on your list in seven days’ time. His aneurysm includes the origin of the left subclavian artery and extends distally to the renal arteries. He has a background of hypertension, type 2 diabetes mellitus and a transient ischaemic attack (TIA). He has a body mass index of 32 kg/ m2 and has a 24 pack-year history of smoking, having ceased smoking when he suffered the TIA four months ago.
His medications include
clopidogrel 75mg daily
rosuvastatin 10mg daily
perindopril 4mg mane
metoprolol 25mg bd
metformin 1000mg bd
How will you assess this man’s risk of spinal cord injury during this procedure?”
“You are called to assess a 40-year-old woman in the surgical ward while working as the evening on-call anaesthetist. The patient has had an anterior cervical discectomy and fusion (ACDF) for cervical radicular symptoms caused by C5/6 disc prolapse. The operation was completed six hours ago with 500mL of blood loss. You have been asked to review the patient because of increasing neck pain.
Medications:
paracetamol 1g orally q6h
ibuprofen 400 mg orally q8h
pregabalin 75 mg orally q12hh
oxycodone-with-naloxone controlled-release 20/10 mg orally q12h
oxycodone 5 to10mg orally q4h
How would you assess this patient with increasing neck pain?”
“You are seeing a 58-year-old man in preadmission clinic prior to elective open abdominal aortic aneurysm repair. He is known to have hypertension, hypercholesterolaemia and aortic regurgitation. He stopped smoking when his aneurysm was diagnosed 6 years ago. He lives independently in his own home with his wife. He uses a stationary exercise bike three times per week for 20 minutes and enjoys gardening.
Current medications:
* amlodipine 5 mg daily
* aspirin (enteric-coated) 100 mg daily
* enalapril 10 mg daily
* simvastatin 20 mg daily
How will you evaluate this man for the proposed procedure?”
A 50-year-old man presents to the Emergency Department of your tertiary centre two
hours after the onset of swelling in the face and lips.
Medications:
Ramipril 5 mg once daily
Rosuvastatin 10 mg once daily
Allergies:
Shellfish
As the duty anaesthetist, you have been telephoned by the Emergency Medicine
physician and asked to assist with airway management.
How will you respond to the request?
AREAS COVERED:
1. 2. 3. Management of anticipated difficult intubation with massive angioedema
Airway plan and management following a failed awake fibreoptic intubation
Management of QA bullying complaint
You are the anaesthetist for an upper limb orthopaedic list. The next patient on your
list is a 55-year-old man for a left arthroscopic acromioplasty and rotator cuff repair in
the beach chair position. The expected duration of surgery is 2.5 hours.
Medical history:
Currently well, generally active
Hypertension
Obesity (body mass index 35 kg/m2)
Smoker 10/day
No known allergies
Observations:
Height 178 cm
Weight 110 kg
Heart rate 65 beats per minute
Blood pressure 125/75 mmHg
Medications:
Irbesartan 150 mg mane
Discuss your plan for this patient’s perioperative analgesia.
AREAS COVERED:
1. 2. 3. Perioperative analgesia with interscalene block +/- catheter
Management of upright beach-chair position including discussion of cerebral
perfusion
Discussion of differential diagnosis and management in a patient who is slow to
wake from anaesthesia
You are reviewing a 10-year-old child on the day of surgery who has been scheduled
for bilateral proximal femoral derotation osteotomies and tendon transfers with a left
pelvic osteotomy. The child has a history of cerebral palsy, epilepsy, autism
spectrum disorder and intellectual disability, attending a school for children with
disability in year 3. They have had previous surgery on the right hip and can mobilise
short distances with a frame but they use a wheelchair when out of the house.
Weight 25 kg.
Medications:
Baclofen 2.5 mg TDS
Gabapentin 30 mg nocte
Omeprazole 5 mg daily
Sodium valproate 250 mg BD
Describe your key points of discussion with the parents of this child.
AREAS COVERED:
1. Epidural placement: discussion of technique, level, medications etc
2. Intraoperative hypotension: assessment and management
3. Management of decreased leg movement post-operatively
An 88-year-old woman is scheduled for an elective transcatheter aortic valve
implantation (TAVI). Her aortic stenosis has been under surveillance since she
underwent two-vessel coronary bypass grafting nine years ago. She has experienced
worsening exertional dyspnoea (NYHA III) and orthopnoea over the past six months
and had an admission for heart failure last month that responded to diuretics.
She is an ex-smoker (30 pack-year history) with moderate chronic obstructive
pulmonary disease. Her current FEV1 is 1.2 litres (56% predicted). She had one
admission for a respiratory illness in 2021 when she experienced COVID-19
pneumonitis.
Medications:
Amlodipine 5 mg mane
Aspirin 100 mg mane
Pantoprazole 20 mg mane
Ramipril 5 mg mane
Symbicort Turbuhaler (budesonide/formoterol) 200/6 mcg 2 puffs inhaled bd
Salbutamol inhaled prn
Biometrics:
Height 178 cm
Weight 58 kg
Body mass index 18.3 kg/m2
Body surface area 1.73 m2
What further information do you require on history?
AREAS COVERED:
1. 2. 3. Assessment of and anaesthetic plan for a patient for Transcatheter Aortic Valve
Implantation (TAVI)
Conversion from sedation to general anaesthetic mid procedure. Management of
bronchospasm
Hypotension in recovery – differential diagnosis and management
You are the anaesthetist in a regional hospital assigned to the morning emergency
list. Your next patient is a 45-year-old man admitted the previous evening with
haematuria and flank pain from a renal stone. He is booked for cystoscopy, laser
litholapaxy and JJ stent insertion.
Past Medical History:
Hypertension
Type 2 diabetes mellitus
Ryanodine receptor mutation
Medications:
Metformin 500 mg BD
Ramipril 5 mg mane
CT Abdomen and Pelvis report conclusion:
The appearance of the left kidney is in keeping with obstructive uropathy and
superimposed pyelonephritis.
What discussion points will you highlight in your pre-anaesthesia consultation with
this patient?
AREAS COVERED:
1. 2. 3. Preparation of operating theatre for a Malignant Hyperthermia case
Differential of intraoperative fever, including consideration of sepsis and
Malignant Hyperthermia
Awareness under general anaesthesia secondary to a tissued cannula
You are working in the MRI suite of a major regional hospital providing general
anaesthesia.
Your next patient is a 35 year-old woman with sensorineural hearing loss for a brain
MRI. She requires anaesthesia because of severe anxiety and claustrophobia. Her
completed Patient Health Questionnaire is attached.
She received oral diazepam 10 mg one hour ago as premedication. She is calm and
mildly sedated in the anaesthesia room adjacent to the MRI scanner.
Outline your preoperative assessment of this patient.
AREAS COVERED:
1. Assessment and anaesthetic plan
2. 3. Management of a slightly compromised LMA towards the end of the scan
Management of an unanticipated difficult airway in MRI
You are called by the obstetrics registrar to insert an epidural catheter in a patient
with preeclampsia for blood pressure management and labour analgesia. The patient
is a 30-year-old primiparous woman who is at 35+5 weeks gestation. She has
gestational diabetes (diet-controlled) and her only medication is low-dose aspirin for
a family history of hypertension.
How will you assess this patient?
AREAS COVERED:
1. Assessment and initial management of a patient with severe pre-eclampsia
2. Management of eclamptic seizure on labour ward
3. Anaesthestic plan for caesarean section in eclamptic patient
You are the anaesthetist on-call for trauma in a major tertiary hospital. You have
been asked to attend the Emergency Department for a 48-year-old man brought in
by paramedics with stab wounds following a pub fight in a regional town two hours
away.
He has no previous medical conditions and no known allergies.
You notice that he is obese. He is irritable but is obeying commands. He has a cut
and bruising over his left eye. There are wounds on his abdomen covered by blood-
soaked gauze packs, and further wounds on his leg, with a tourniquet on his upper
thigh.
His observations are:
Heart rate 125 bpm
Blood pressure 110/68 mmHg
Respiratory rate 26 breaths per minute
SpO2 96% on 10LO2/min via Hudson mask
The ED physician tells you that an eFAST scan is positive. The surgeon asks if you
are able to go straight to theatre for exploratory laparotomy.
Would you prioritise Computed Tomography (CT) scanning for this patient before
going to the operating theatre?
AREAS COVERED:
1. 2. 3. Management of a haemodynamically unstable patient with positive FAST scan
Management of intraoperative hypotension in a trauma patient
Management of cardiac arrest in a trauma patient
As the duty anaesthetist, you receive a phone call from the emergency department
physician at your regional hospital, requesting help with an incoming trauma patient.
A 35-year-old male incarcerated person is being transferred by paramedics following
an assault two hours ago whilst he was having his meal. He was kicked and punched
in both the face and chest and there was an attempted strangulation.
You are informed over the phone that the patient has difficulty breathing and has
bruising and swelling to the front of his neck but otherwise appears stable.
He has no allergies and is not on any medications. He smokes 10-15 cigarettes a
day and has no significant medical history.
What actions will you take prior to the patient’s arrival?
AREAS COVERED:
1. 2. 3. Assessment and management of a patient with suspected airway trauma
Formulation of a safe anaesthetic plan and justification of choice in a patient with
a partial tracheal transection
Differential diagnosis and management of persistent hypoxia after securing the
airway
You are reviewing a 72-year-old woman in the anaesthetic bay of a metropolitan
hospital. She is scheduled for an elective left total hip replacement and was
previously assessed in the pre-anaesthetic clinic two months ago. Two years prior
she underwent a right total knee replacement, which was complicated by a 48-hour
High Dependency Admission as she was “slow to wake up” and required
supplemental oxygen therapy.
Medical history:
Hypertension
Obesity (body mass index 41 kg/m2)
Medications:
Atorvastatin 20 mg daily
Candesartan/hydrochlorothiazide 16/12.5 mg daily
Outline the areas of focus in your preoperative assessment of this patient.
AREAS COVERED:
1. 2. 3. Preoperative assessment of suspected Obstructive Sleep Apnoea (OSA)
Management of intraoperative cardiac arrest
Management of chest pain post-Cardiopulmonary Resuscitation (CPR)
You are the on-duty anaesthetist in a large regional centre with a paediatric surgical
service. You are called to assist in the Emergency Department with the management
of an otherwise healthy 2-year-old girl who has ingested drain cleaner (sodium
hydroxide) at home about two hours ago. There are concerns about likely aspiration
or inhalational injury from vapours.
The Emergency Medicine specialist tells you that the girl was brought in to hospital
by her mother after she found the child sitting on the kitchen floor with a pile of drain
cleaner crystals next to her and crystals on her hands and inside her mouth. The
mother was unable to quantify the amount ingested. The girl vomited once on the
way to hospital and is currently drooling and distressed in her mother’s arms.
What are the important issues to consider in the assessment of this child?
AREAS COVERED:
1. 2. 3. Assessment of airway injury and airway management
Management and differential diagnosis of hypoxia during the procedure
Diagnosis and management of pneumothorax intraoperatively from barotrauma
You are working in the preadmission clinic, reviewing your patients for a respiratory
medicine bronchoscopy list in a week’s time. Your next patient is a 74-year-old man
who has been scheduled for an elective bronchoscopic lung volume reduction
procedure by insertion of endobronchial valves.
Medical history:
Emphysematous chronic obstructive airways disease - home oxygen 1L/min, 18
hours per day
Ex-smoker – ceased 5 years ago, 60+ pack-year smoking history
Osteoarthritis
Peripheral vascular disease
Atrial fibrillation
Medications:
Ipratropium bromide MDI (21 mcg/puff) 2 puffs bd
Seretide Inhaler (fluticasone/salmeterol 25/250 mcg) 2 puffs bd
Salbutamol (100 mcg/puff) 2 puffs prn
Paracetamol 1 g tds
Rivaroxaban 15 mg daily
The respiratory physician will perform the procedure in the bronchoscopy suite of
your major tertiary hospital and has requested the case be performed under general
anaesthesia with spontaneous ventilation. The patient has a planned admission for
two days post-procedure to monitor for any complications.
What further information about the procedure would you seek from the respiratory
physician?
AREAS COVERED:
1. 2. 3. Assessment of and anaesthetic plan for a patient with advanced chronic
obstructive pulmonary disease (COPD)
Management of Type II respiratory failure during the procedure
Ventilatory strategies and patient disposition/extubation planning in the above
patient
You are an anaesthetist at a tertiary hospital working in the pre-anaesthetic
clinic. You review a 74-year-old man who is booked for robotic assisted partial
nephrectomy for a 5 cm right upper pole renal cell cancer.
Medical History:
Obesity
Controlled hypertension
Type 2 diabetes mellitus
Obstructive sleep apnoea (treated with CPAP)
Hypercholesterolaemia
Medications:
Amlodipine 10 mg daily
Aspirin 100 mg daily
Atorvastatin 40 mg daily
Empagliflozin 10 mg daily
Metformin 1000 mg twice daily
Perindopril 8 mg daily
Biometrics:
Height 183 cm
Weight 144 kg
BMI 43 kg/m2
How would you assess this patient for their partial nephrectomy?
AREAS COVERED:
1. 2. Assessment of patient and anaesthetic planning for case
Management following intraoperative renal vein injury and conversion to an open
procedure
3. Postoperative analgesic plan including regional options
You are on-call from home for a private hospital that has Intensive Care and
Interventional Radiological facilities. You receive a call from the Upper GI surgeon
you regularly work with regarding a patient you anaesthetised ten days prior. The
patient is a 72-year-old man who had a laparoscopic distal pancreatectomy to
remove a pancreatic mass found incidentally.
Medical history:
Hypertension
Hypercholesterolaemia
Obesity (weight 108 kg, body mass index 34 kg/m2)
Medications:
Irbesartan 150 mg daily
Rosuvastatin 20 mg daily
The patient’s surgery and recovery were uneventful and he was discharged home on
day four postoperatively. The patient’s anaesthetic, including airway management,
was also uneventful.
The surgeon is calling you because the patient has just gone to radiology for coiling
of a possible bleeding vessel and the surgeon wants to give you a “heads up” in case
the patient needs to go to the operating theatre.
What further information would you like from the surgeon?
AREAS COVERED:
1. 2. 3. Response and management of a potentially bleeding patient after hours in a
remote location without a skilled anaesthetic assistant immediately available
Management of an agitated, unstable patient in Interventional Radiology,
including the consideration of when to convert to a general anaesthetic
Assessment and management of self-limiting runs of non-sustained ventricular
tachycardia (NSVT)
A 28-year-old primigravida currently at 24 weeks gestation attends your high-risk
obstetric clinic with her partner. She has recently been diagnosed with myotonic
dystrophy and is under the care of a neurologist at your hospital. Her presenting
symptoms were progressive muscle weakness and difficulty swallowing over a period
of two years. She has had three hospital admissions for aspiration pneumonia, none
of which required intensive care unit admission. The pregnancy has been confirmed
with an early first trimester ultrasound scan and has been uneventful to date.
Observations:
Height 1.62 m
Weight 65 kg
Pulse rate 90 beats per minute
Blood pressure 100/60 mmHg
Respiratory rate 20 breaths per minute
SpO2 96% on room air
Airway examination:
Thyromental distance less than 5 cm
Mallampati class III
Inter-incisor distance greater than 4.5 cm
Good neck extension
Normal jaw protrusion
Cardiovascular examination:
Normal heart sounds with no additional sounds
No signs of heart failure
Neurologic examination:
Proximal muscle weakness
Presence of persistent grip with handshake
Mild scoliosis with curvature to left
Medications:
Bisoprolol 2.5 mg mane
Frusemide 20 mg mane
A recent transthoracic echocardiogram shows mild biventricular and biatrial dilation;
left ventricular ejection fraction 45%; and pulmonary artery systolic pressure of 25
mmHg.
She would like to discuss options for labour analgesia. What would you recommend
as the best option for her?
AREAS COVERED:
1. Discussion of myotonic dystrophy with assessment and planning for delivery
2. Diagnosis and management of cardiovascular instability / cardiomyopathy-related
arrhythmia in this patient during labour
3. Planning and conduct of anaesthesia for an urgent caesarean section
You are urgently directed to Resuscitation in a metropolitan Major Trauma Centre. A
24-year-old driver was in a motor vehicle accident between his car and a lorry
carrying 12-cm diameter pine logs, with a log penetrating the windshield and impaling
his right thorax. The time since injury is 25 minutes. He is fully conscious and in
severe pain.
His initial observations are:
Heart rate 128 bpm
Blood pressure 88/68 mmHg
Respiratory rate 28 breaths per minute
What further information do you require?
AREAS COVERED:
1. Assessment and management of life-threatening complications of penetrating
2. 3. chest trauma
Indications and planning for an Emergency Department thoracotomy
Discussion of options and preferred technique for lung isolation
You are the anaesthetist on-call in a tertiary hospital. The vascular surgeon calls you to
advise that an 82-year-old man with a contained rupture of an abdominal aortic aneurysm has
just arrived in the emergency department, transferred from a secondary centre.
The surgeon advises that the aneurysm is not suitable for an endovascular technique and the
patient requires an urgent open repair. You immediately attend the emergency department.
What is your initial assessment of this patient?
2023.2 Final Examination
AREAS COVERED:
1. 2. 3. Initial assessment and management of hypertensive patient, and early planning of case
whilst still in ED.
Intraoperative management including cardiac ischaemia and goals relating to aortic cross
clamping. ROTEM interpretation.
Differential diagnosis of ventilatory difficulties at end of case. Management of pulmonary
oedema.
You are the anaesthetist in the pre-assessment clinic of a day surgery hospital.
You are reviewing an 86-year-old man who has been booked for a wide local excision of a left
temple squamous cell carcinoma (SCC) and rotational flap under local anaesthetic and
sedation.
Medical History
Dementia (moderate) – lives in hostel
Atrial fibrillation (stable)
Hypertension (stable)
Medications
Dabigatran 110 mg twice daily
Diltiazem 180 mg mane
Confidential draft
He has been brought to the clinic by his son, his enduring guardian, who is very concerned
about the possibility for conversion to a general anaesthetic. The patient had a general
anaesthetic for removal of an SCC on his leg last year and experienced postoperative
delirium requiring a three-day hospital stay.
His observations and results of blood tests are included below.
What issues would you address during the preoperative visit?
Weight 65 kg
Body mass index 23 kg/m2
Heart rate 75 beats per minute
Blood pressure 140/80 mmHg
Respiratory rate 15 breaths per minute
SpO2 97 % on room air
result reference range
Haemoglobin 120 (115 – 160 g/L)
White cell count 7.0 (4.0 – 11.0 x 109/L)
Platelet count 300 (150 – 400 x 109/L)
2023.2 Final Examination
result reference range
Sodium 136 (135-145 mmol/L)
Potassium 3.5 (3.5 – 5.2 mmol/L)
Urea 4.0 (3.0 – 8.0 mmol/L)
Creatinine 70 (45 – 90 µmol/L)
AREAS COVERED:
1. Discussion of issues regarding previous post operative cognitive dysfunction and its impact
on anaesthetic planning, including suitability for day surgery and management of regular
medications.
2. Anaesthetic management, including intra-operative conversion from sedation to general
anaesthesia.
3. Management of post operative cognitive dysfunction.
You are anaesthetising for an ophthalmology list at a standalone day-surgery unit with two
operating theatres.
Your first patient is a 55-year-old man with a rapidly-progressive posterior subcapsular
cataract in the right eye for a cataract extraction and intraocular lens insertion.
Confidential draft
Medical history
Type 2 diabetes mellitus
· oral hypoglycaemics
· HbA1c 6.8% (51 mmol/mol)
· no known end-organ disease other than cataracts
Highly myopic, blind in left eye after previous macular retinal detachment at age 51 years
Medications
Metformin 1 g twice daily
Allergies:
Nil
Examination is unremarkable, with body mass index of 24 kg/m2. Preoperative investigations
have been reviewed and are normal. The patient is fasted and has been considered
appropriate for day surgery.
What are your considerations when evaluating your options for anaesthesia for this patient’s
cataract surgery?
AREAS COVERED:
1. Discussion of patient specific risks and their impact on anaesthetic planning. Details of
sub-Tenon’s block.
2. Surgical plan changes to vitrectomy due to posterior capsule rupture. Patient increasingly
restless, requiring eventual conversion to GA.
2023.2 Final Examination
3. PACU assessment of separate patient with hypoxaemia and sedation from benzodiazepine
overdose. Management of clinical scenario and consideration of professional issues of
colleagues performance.
You are the visiting anaesthetist in a small rural town where you work once a month with a
visiting urologist. You receive a telephone call from a nurse in the Emergency Department
asking if you could come and assist the GP anaesthetist.
The ambulance service have just brought in a 25-year-old man who fell from the roof of a
farm shed, where he had been sitting and drinking with friends. The GP anaesthetist
intubated the patient on arrival to hospital because of reduced Glasgow Coma Scale (GCS)
score and is requesting your assistance with ongoing management.
Describe your initial response to this request.
AREAS COVERED:
1. Assessment of multi trauma patient in a rural setting with limited resources. Prioritising
management and interpretation of blood gas results and plain film imaging.
2. Ongoing trauma resuscitation and recognition of multiple potential causes of shock
(hypovolaemic and neurogenic).
3. Worsening acidosis and development of profound bradycardia from neurogenic shock.
Confidential draft
Management of heart rate, including discussion of inotropes, pacing and appropriate volume
replacement.
A 70-year-old man presents on the day of surgery for an elective L2–L5 posterior lumbar
intervertebral fusion (PLIF), with a plan for intraoperative neurophysiological monitoring.
Medical History
Back pain
Hypertension
Type 2 diabetes mellitus
Paroxysmal atrial fibrillation
Smoker
Medications
Sotalol 80 mg twice daily
Enalapril 10 mg twice daily
Metformin 500 mg twice daily
Paracetamol 667 mg three times daily
Meloxicam 15 mg daily
Gabapentin 600 mg three times daily
Tramadol 100 mg three times daily PRN
2023.2 Final Examination
Vital Statistics
Height 1.80 m
Weight 101 kg
Body Mass Index (BMI) 31 kg/m2
Preoperative blood test results are included below. An electrocardiogram (ECG) was
performed this morning on admission to hospital and is attached.
What further history would you seek from this patient before proceeding with his anaesthesia?
Full blood count test results:
result reference range
haemoglobin 148 (115 – 160 g/L)
white cell count 5.2 (4.0 – 11.0 x 109/L)
red cell count 4.1 (3.80 – 4.80 x 1012/L)
platelet count haematocrit mean cell haemoglobin 398 0.40 30.0 (150 – 400 x 109/L)
(0.37 – 0.47 L/L)
(27.0 – 32.0 pg)
mean cell volume 89 (80 – 100 fL)
mean cell haemoglobin concentration 340 (320 – 360 g/L)
Biochemistry test results:
Confidential draft
result reference range
sodium 138 (135 – 145 mmol/L)
potassium 4.2 (3.5 – 5.2 mmol/L)
bicarbonate 26 (22 – 32 mmol/L)
urea 6.1 (3.0 – 8.0 mmol/L)
creatinine 89 (45 – 90 µmol/L)
eGFR 69 (> 80 mL/min/m2)
2023.2 Final Examination
AREAS COVERED:
Confidential draft
1. Patient assessment, and discussion of how neuro monitoring and presence of prolonged
QT interval on ECG will impact anaesthetic planning.
2. Management of intra-operative dysrhythmia (Torsade de Pointe and VF).
3. Management of adverse event with inadvertent administration of adrenaline into arterial
line during resuscitation.
You are working in a standalone day surgery facility.
A 2-year-old boy weighing 15 kg presents for the combined procedures of release of bilateral
hand syndactyly and open orchidopexy for a unilateral undescended testis.
He has had a clear runny nose for the past three days, which his parents attribute to teething.
A COVID-19 rapid antigen test performed this morning returned a negative result.
How will you decide whether to proceed with surgery or defer until a later date?
AREAS COVERED:
1. Consideration of appropriateness of proceeding with surgery in setting of recent URTI.
2. Discussion of caudal block, including timing relative to surgery, details of technique.
3. Anaesthetic technique including details of inhalational induction and management of airway
obstruction without IV access. Post operative hypoxaemia management and discussion of
discharge criteria.
You are the duty anaesthetist in a tertiary hospital. You receive a telephone call from the
obstetric consultant working in the antenatal clinic today. She is with a 24-year-old woman
who is currently 35/40 in her first pregnancy. The woman has been booked for a caesarean
section in a week’s time for intrauterine growth retardation.
Medical history
Severe scoliosis – corrective posterior fusion surgery at 16 years of age
Height 1.45 m
Weight 42 kg
Body mass index 20 kg/m2
What are the important issues to consider in your assessment of this patient?
AREAS COVERED:
1. Discussion of anaesthetic options in a patient with spinal fusion and restrictive lung
disease. Consent issues.
Confidential draft
2. Details of CSE approach including dosing. Pain during surgery, assessment and options to
manage.
3. Postoperative discussion with patient about pain experience. Appropriate management of
complaint.
You are the anaesthetist on evening duty at your tertiary-level hospital. You receive a call
from the emergency medicine physician who requests your assistance with the management
of a 50-year-old man who experienced a choking episode while eating a sausage for lunch.
The patient is thought to have aspirated a piece of the sausage and is somewhat distressed.
The patient has a history of brittle asthma and chronic obstructive pulmonary disease with a
60 pack-year history of smoking. He has known tracheal stenosis for which he receives
regular tracheal dilations.
Medications
Fluticasone 250 mcg via metered-dose inhaler (MDI) 2 puffs twice daily
Salmeterol 50 mcg via metered-dose inhaler (MDI) 2 puffs twice daily
Salbutamol 100 mcg via metered-dose inhaler (MDI) 2 puffs twice daily
The emergency medicine physician would like to transfer the patient straight to theatre.
2023.2 Final Examination
How will you assess his suitability for urgent transfer to theatre?
AREAS COVERED:
1. Assessment and optimisation of patient in ED. Discussion of merit of facilitating a
preoperative CT scan versus proceeding straight to theatre.
2. Anaesthesia technique for flexible then proceeding to rigid bronchoscopy for retrieval of
airway foreign body.
3. Assessment and management of hypoxaemia and chest pain in PACU, development of
pneumomediastinum and subcutaneous emphysema.
You are the anaesthetist allocated to the emergency theatre at a tertiary hospital.
You attend the holding bay to assess your next patient, who is a 30-year-old woman booked
for an open repair of a right common femoral artery pseudoaneurysm. She has a history of
substance use disorder and the injury is a result of self-injection.
The 18-gauge intravenous cannula inserted under ultrasound guidance in the emergency
department is no longer patent. There is a radial arterial line in-situ.
Prehospital and emergency department management has included:
Ketamine 50 mg intravenously
Confidential draft
Fentanyl 100 mcg (in aliquots) intravenously
Hartmann’s solution one litre intravenously
The patient’s biometric data and results of an arterial blood gas are included below.
Outline how the history of substance use disorder influences your assessment of the patient.
Biometric data
Height 1.70 m
Weight 50 kg
Body mass index 17.3 kg/m2
Body surface area 1.51 m2
Ideal body weight 61 kg
Arterial blood gas (FiO2 0.44, Temperature 38.0°C)
value reference range
(7.35–7.45)
pH 7.30 pCO2 32
4.26
(35–45 mmHg)
(4.6–6.0 kPa)
2023.2 Final Examination
pO2 120
15.99
(80–100 mmHg)
(10.7–13.3 kPa)
bicarbonate 15.2 (21–28 mmol/L)
base excess -9.9 (-3.0–3.0 mmol/L)
lactate 3.5 (< 1.3 mmol/L)
haemoglobin 108 (115–160 g/L)
Na+ 131 (137–147 mmol/L)
K+ 5.7 (3.5–5.0 mmol/L)
Ca2+ 0.75 (0.85–1.25 mmol/L)
Cl- 108 (96–109 mmol/L)
HCO3- 15.2 (22–26 mmol/L)
glucose 6.4 (3.0–7.7 mmol/L)
AREAS COVERED:
Confidential draft
1. Initial assessment and discussion of recent methamphetamine use, proceeding to
management of conscious ventricular tachycardia.
2. Anaesthetic planning and management of progressive hypotension. Interpretation of TEG
and its guidance on blood product administration.
3. Post-operative pain and management of compartment syndrome.
You are the weekend on-call anaesthetist at a tertiary regional hospital.
A 45-year-old man has been transferred to your emergency department for management of a
femoral fracture after he fell last night. He received a femoral nerve block and splint at the
referral hospital before being transferred to your facility.
The surgeons are worried about increasing swelling and gross deformity of his thigh and
would like to place a femoral nail as soon as possible.
The patient has a past history of chronic back pain that was associated with OxyContin
misuse. The OxyContin was converted to buprenorphine after a spinal fusion four months
ago.
He had breakfast six hours ago.
2023.2 Final Examination
Medications
Buprenorphine 18 mg sublingual once daily
Pantoprazole 40 mg once daily
Observations
Heart rate 100 beats per minute
Blood pressure 110/85 mmHg
SpO2 95% on room air
Respiratory rate 16 breaths per minute
Weight 160 kg
Height 1.86 m
Body mass index 45 kg/m2
How would the previous opioid misuse and current buprenorphine use affect your approach to
the management of this patient?
AREAS COVERED:
1. Impact of opioid misuse on planning, discussion of anaesthetic technique including impact
of recent dose of Clexane.
2. PACU assessment of impaired consciousness and hypoxaemia, differentials and
management.
3. Management of severe pain and confusion, discussion of analgesic options and differential
diagnoses.
You are working in a standalone psychiatry hospital covering a colleague’s electroconvulsive
therapy (ECT) list.
The first patient is a 53-year-old anaesthetist colleague with severe depression who has been
booked for ECT titration (first ECT treatment). You review her in her room prior to the start of
the ECT list. She is visibly upset.
Medical history
Type 2 diabetes mellitus
Hypertension
Paroxysmal atrial fibrillation
Medications
Candesartan 16 mg daily
Rivaroxaban 10 mg daily
Venlafaxine 75 mg daily
What are the requirements to safely proceed with ECT in this standalone facility?
2023.2 Final Examination
AREAS COVERED:
1. Discussion of relevant ANZCA professional documents. Approach to patient assessment
and consent.
2. Anaesthesia for ECT. Profound vagal response post ECT resulting in short duration of
asystole. Prolonged seizure, management.
3. Tongue laceration with bleeding and airway obstruction noted in PACU requiring urgent
intubation.
You are the on-call anaesthetist attending a trauma call in a major trauma centre.
A 30-year-old male driver was brought to the hospital by paramedics after a high-speed
collision with a tree. A laryngeal mask airway was inserted en route due a fall in his level of
consciousness. There is extensive bruising over his right chest and a prominent seatbelt sign.
He has polycystic kidney disease and attends the hospital for regular haemodialysis through
an arteriovenous (AV) fistula in his left arm. He is otherwise well and has no known allergies.
How does end-stage renal failure affect the trauma management of this patient?
AREAS COVERED:
Confidential draft
1. Assessment and ED management. Discussion of treatment priorities in a patient with multi
trauma on a background of chronic renal failure requiring haemodialysis.
2. Anaesthetic care and resuscitation. How the presence of a severe traumatic brain injury
may alter the trauma management.
3. Progressive hypoxaemia in theatre, differentials and management.
You are called to the neurosurgical theatre by your provisional fellow who requests your
assistance. They have anaesthetised a 36-year-old woman for a craniotomy and open
clipping of a saccular middle cerebral artery aneurysm.
The patient was admitted with a sudden onset of headache two days prior and was
diagnosed with a World Federation of Neurosurgical Societies (WFNS) grade I subarachnoid
haemorrhage.
Medical History
Hypertension
Asthma
Medications
Nimodipine 60 mg four-hourly
2023.2 Final Examination
Irbesartan 150 mg daily
Salmeterol/Fluticasone 250/25 mcg via metered-dose inhaler (MDI) daily
Salbutamol 100 mcg via metered-dose inhaler (MDI) PRN
Allergies
Morphine (rash)
When you attend the theatre, you discover that the patient was induced, intubated and
anaesthetised uneventfully. She is currently being maintained on total intravenous
anaesthesia, with target-controlled infusions of remifentanil and propofol. The patient has an
arterial line and a central venous catheter in situ. Surgery is underway.
The provisional fellow is concerned that the patient has developed high airway pressures and
a heart rate of 120 beats per minute.
What is your approach to this situation?
AREAS COVERED:
1. Initial approach to the situation, progressing to management of grade 2 anaphylaxis.
2. Hypertensive episode resulting in aneurysm rupture requiring temporary clipping,
discussion of haemodynamic goals.
3. Approach to failure to emerge from anaesthesia, consider differentials and management.
A 12-year-old boy with an intellectual disability presents for dental restoration and extractions
at your tertiary paediatric hospital.
His parents report that he is fearful of needles and has been worried about the surgery and
anaesthetic.
Weight 133 kg
Height 1.60 m
Body mass index 52 kg/m2
What specific concerns would you seek to clarify on history.
AREAS COVERED:
1. Discussion of sleep disordered breathing including interpretation of sleep study and how
the diagnosis would impact the postoperative planning.
2. Appropriate premedication to manage uncooperative patient, then details of anaesthetic
technique.
3. Management of tachycardia and hypoxaemia after surgical injection of local anaesthetic
with adrenaline.
You are the anaesthetist in the obstetric anaesthesia clinic in a tertiary hospital.
You are asked to assess a 28-year-old woman with paraplegia in her first pregnancy. She is
34 weeks pregnant and is booked for an elective caesarean section at term.
The referral states:
Paraplegia following a horse riding accident aged 16 years
IVF conception/pregnancy
Medications
Amitriptyline 50 mg daily
Cranberry capsules
Folic acid
Iron supplements
Baclofen – discontinued with pregnancy
What specific information on history and examination will help you develop an anaesthetic
management plan for this patient?
AREAS COVERED:
1. Assessment of spinal cord injury including discussion of autonomic dysreflexia, and plan
for the anaesthesia for the caesarean section.
Confidential draft
2. Detail of spinal anaesthesia for urgent delivery, assessment of block height in a patient with
a complete motor and sensory injury at T6.
3. Dyspnoea in PACU with consideration of differentials and management, particularly
including high spinal.
You are providing anaesthesia for an elective ENT list. A 63-year-old man with a laryngeal
squamous cell carcinoma is booked for a salvage laryngectomy, and neck dissection with a
free flap reconstruction.
The patient was seen ten days ago in the preadmission clinic. He was documented to have
chronic obstructive airway disease only, with no known cardiovascular disease.
His observations this morning were:
Heart rate 86 beats per minute
Blood pressure 128/75 mmHg
SpO2 95% on room air
Body mass index 18 kg/m2
Medications
Salbutamol 100 mcg via metered-dose inhaler (MDI) PRN
Salmeterol/fluticasone (50/250 mcg) via metered-dose inhaler (MDI) 2 puffs twice daily
2023.2 Final Examination
In addition to the routine examination, please describe how you will assess this patient’s
airway.
AREAS COVERED:
1. Discussion of patient, surgery and anaesthesia issues. Appropriate plan to secure the
airway.
2. Intraoperative ST depression, management and discussion of competing interests of
defending coronary perfusion versus concerns of flap viability with vasopressor agents.
3. Professional issues in managing surgical fatigue.
You have been scheduled to an MRI list at a paediatric hospital. The first patient on the list is
an 8-year-old girl for MRI brain and a lumbar puncture.
The girl has autism spectrum disorder. She has a mild speech delay but communicates well.
Six months ago she had an episode of venous sinus thrombosis with raised intracranial
pressure secondary to mastoiditis.
The indication for the MRI and lumbar puncture is worsening headache with intermittent low
grade fever (highest temperature 37.8°C) over the past two weeks.
She weighs 25 kg.
She is admitted to the paediatric ward the day before the procedure and you are asked to perform a preoperative assessment.
What information do you seek during your preoperative visit?
Issues:
1. Preop assessment and anaesthesia planning for non-compliant patient for an MRI
2. Anaesthetic planning including recognition and management of bronchospasm
3. Assessment and management of postoperative delirium
You are required to anaesthetise a 75-year-old woman undergoing a posterior C1 – 2 fusion
after a low-velocity mechanical fall at home yesterday evening. Her only injury is a mildly
displaced type II dens fracture which has resulted in posterior neck pain and mild left hand
weakness. She is currently immobilised in a hard cervical collar.
She weighs 52 kg and is 155 cm tall. Her observations are within normal limits.
Her relevant active comorbidity is osteoporosis managed with a bisphosphonate. She lives
alone and is independent in her activities of daily living.
A colleague has performed a preoperative review. The airway comments are:
Confide ntial draft
2024.1 Final Examination
Thin face with minimal cheek tissue
Mallampati 2, mouth opening 3 cm
Thyromental distance 3 cm
Upper partial plate (removable), lower natural dentition
Neck extension not assessed
Previous GA in 2020: Grade II, MAC 3. No comment on bag mask ventilation
What additional information would complete your airway assessment?
Issues:
1. Airway management of a patient with an unstable cervical spine
2. management of inadvertent airway dislodgement in the prone position
3. - diagnosis and management of a postoperative TIA/stroke
An 18-year-old man was brought to the Emergency Department of your major hospital via
ambulance after suffering a seizure at university this afternoon. A CT brain has been
performed which reveals the suspicion of an arteriovenous malformation (AVM) with
associated haemorrhage. There is also a 15 mm diameter cerebral aneurysm that appears
intact.
The patient requires digital subtraction angiography of his cerebral vessels and you are asked
to provide anaesthetic support for this procedure.
Describe the focus of your assessment of this patient.
1. Assessment and management of a patient with raised intracranial pressure (ICP)
2. Anaesthetic plan for embolisation of arteriovenous malformation (AVM).
Management of intraoperative AVM rupture.
3. Management of raised ICP post-operatively
You are working in a large hospital when you are called by a surgeon regarding a 51-year-old
woman who is in the Emergency Department. She requires an urgent laparotomy following a
single stab wound to the abdomen after an assault at home.
Immediate history:
Arrived to hospital via ambulance within 30 minutes of injury
Since arrival to ED:
· primary survey (no obvious other injuries)
· IV access and blood tests taken
· 1000 mL Normal Saline and analgesia administered
· FAST scan reveals free air and fluid
Observations:
Airway patent. GCS 15
SpO2 95% on 6L/min oxygen via Hudson mask
BP 80 mmHg systolic
HR 38 bpm
Dressing on anterior abdomen with some blood leaking through
Medical History:
Height – 153 cm
Weight – 90 kg
BMI 38.4 kg/m2
Type 2 diabetes mellitus, poorly compliant with prescribed treatment
Mild chronic renal impairment
Recent syncopal episodes and referred to cardiology – awaiting appointment
Smoker – 20 cigarettes per day
Medications:
Perindopril 4 mg once daily
Metformin 500 mg twice daily
Confide ntial draft
2024.1 Final Examination
Glipizide 5 mg twice daily
Atorvastatin 20 mg once daily
Initial blood test results and a 12-lead ECG are displayed below.
How will you assess this patient’s volume status before theatre?
1. Assessment of a trauma patient and management of co-morbid symptomatic
bradycardia
2. : Induction of anaesthesia in a complex trauma patient, facilitation of pacing and
management of haemorrhage
3. Diagnosis and management of myocardial ischaemia in a paced, anaesthetised
patient
A 35-year-old woman with a history of myasthenia gravis was recently assessed for
worsening symptoms. She was found to have an anterior mediastinal mass consistent with a
thymoma.
She is booked for a VATS thymectomy due to difficult medical control of her symptoms and
recent development of bulbar dysfunction as well as some clinical evidence of mass effect
from the thymoma.
You are assessing the patient in the Preadmission Clinic a week prior to her scheduled
surgery.
Medications:
Pyridostigmine 180 mg qid
Prednisolone 15 mg mane
What are the important elements in the assessment of this patient?
1. Assessment of myasthenia gravis severity and preoperative optimisation
2. : Anaesthesia for VATS thymectomy for myasthenia gravis
3. Management of post-operative hypoxaemia
You are working in the preadmission clinic. An 86-year-old woman is booked for a
laparoscopic ultra-low anterior resection for rectal cancer on your list in three weeks’ time.
She lives alone at home and can comfortably walk 100 metres on the flat.
Medical history:
Hypertension
Transient ischaemic attack two years ago
Ex-smoker – 60 pack-year history
Confide ntial draft
2024.1 Final Examination
Chronic renal impairment
Medications:
Perindopril 4 mg daily
Aspirin 100 mg daily
Observations:
BP 160/85 mmHg
HR 82 bpm, regular
SpO2 94% on room air
Height 1.70 m
Weight 51 kg
BMI 17.6 kg/m2
A CT abdomen shows a large low rectal cancer and a single nodule in the liver.
The surgeon has asked you to provide an opinion on this patient’s perioperative risk. How would you assess this?
1. Assessment of perioperative risk, application of risk scoring system(s) and
discussion of prehabilitation.
2. Management of intraoperative hypercapnia
3. Diagnosis and management of a retroperitoneal haematoma
You are the anaesthetist on call overnight in a regional hospital.
You are called to the Birth Suite at 2 am to review a 19-year-old woman who has just been
brought in by ambulance in labour with sudden onset of shortness of breath within the last
6 to 12 hours.
The woman is G1 P0 and is 32 weeks gestation by dates. She has not had any antenatal
care during the pregnancy.
The cardiotocogram (CTG) is normal.
Confide ntial draft
2024.1 Final Examination
The patient denies any prior medical history. She weighs 65 kg and has a BMI of 23 kg/m2
.
The midwife informs you that the patient’s oxygen saturation is 94% on 15 L/min oxygen via
non-rebreather mask.
The obstetric registrar and your anaesthetic registrar are currently performing a Category 2
caesarean section in the emergency theatre.
How will you approach this situation?
1. - Assessment and initial management of hypoxia and severe hypertension in
pregnancy
2. Anaesthetic management for Category 1 caesarean section in patient with severe
pre-eclampsia and acute pulmonary oedema
3. Resuscitation of the pre-term neonate
You have been contacted by a vascular surgeon to inform you of an upcoming case at your
metropolitan hospital.
The patient is a 50-year-old previously well man who had a ten-day ICU admission for severe
sepsis due to COVID-19 pneumonitis with Streptococcus pneumoniae superinfection, for
which he was intubated and ventilated as well as treated with inotropes and vasopressors.
He was extubated one week ago and vasopressor support was weaned. He is currently on a
medical ward.
He requires bilateral below-elbow amputations and a right below-knee amputation for
vasopressor-induced irreversible ischaemia associated with intractable pain.
He has a resolving lung injury and remains on oxygen at 3 L/minute via nasal prongs.
Current observations:
SpO2 89% on room air
SpO2 95% on oxygen 3L/minute
Weight 80 kg (premorbid weight 85 kg)
Height 179 cm
BMI 25 kg/m2 (premorbid BMI 26.5)
Confide ntial draft
2024.1 Final Examination
Medications:
Ampicillin 1 g QID
Paracetamol 1 g QID
Pregabalin 50 mg TDS
Oxycodone/naloxone 20 mg/10 mg BD
Oxycodone infusion – 4 mg per hour intravenously
(Note - ketamine was trialled and removed due to nightmares and dysphoria)
The surgeon anticipates a surgical time of eight hours and will involve the use of bilateral
upper limb tourniquets and skin grafting.
What factors would influence your decision regarding the timing of surgery?
1. Demonstrate a considered approach to the perioperative management of a
complex patient requiring complex surgery. Approach should adequate patient assessment
and an awareness of the logistic challenges in the case
2. Demonstrate a reasonable approach to analgesic requirements in a complex
patient. Appropriately respond to local anaesthetic toxicity.
3. Assessment and management of hypoxaemia and tachycardia in PACU
A 2-year-old boy has presented for left VATS and decortication for an empyema after a chest infection. He has been on the ward in your tertiary referral centre for three days receiving antibiotics.
There is no significant past medical history and he has normal developmental milestones.
Vital signs:
HR 130 bpm
BP 95/52 mmHg
respiratory rate 28 bpm
temperature 37.5oC
weight 12 kg
He has been discussed at a multidisciplinary meeting and has been deemed suitable for surgery. His chest X-ray is presented here.
How would you assess this patient preoperatively?
1. Assessment of an acutely unwell child and consideration of options for one lung
ventilation
2. Management of intraoperative paediatric arrest
3. Pain management plan post thoracotomy
You are in working in the preadmission clinic, assessing a 40-year-old man who is booked for
transsphenoidal resection of pituitary adenoma on your neurosurgical list next week.
Medical history:
Acromegaly
Obstructive sleep apnoea
Cardiomyopathy
Hypertension
Medications:
Candesartan 16 mg mane, recently increased from 8 mg mane
Frusemide 80 mg bd
Metoprolol 47.5 mg bd
Octreotide 40 mg monthly by intramuscular injection
Cabergoline 1.5 mg weekly
There are no known drug allergies.
Vital signs:
Height 185 cm
Weight 125 kg
BMI 36.5 kg/m2
HR 80 bpm
BP 145/70 mmHg
SpO2 96% on room air
His only previous anaesthetic was for adenotonsillectomy as a child.
What information will you seek on history to address your key concerns about this patient?
1. Preoperative assessment of acromegaly
2. Anaesthetic and airway plan for transsphenoidal resection of pituitary adenoma
3. Management of hypoxia / obstructed airway in PACU secondary to retained
throat pack
A 40-year-old woman is booked on your list tomorrow for posterior fossa craniotomy and
resection of medulloblastoma. She presents with a three-week history of worsening
headache, ataxia and dysarthria.
She has a medical history of mild intellectual impairment, obesity (BMI 35) and an unrepaired
Ebstein’s congenital cardiac anomaly.
The last cardiology review that is recorded in the hospital medical record was ten years ago
and revealed the following:
Moderate tricuspid regurgitation (posterior and septal leaflets adherent)
Some right ventricular enlargement
Atrial septal defect
No evidence of tachyarrhythmias
The neurosurgeon wishes to perform the surgery with the patient prone with reverse
Trendelenberg (head-up) positioning. A sagittal image of the patient’s brain MRI is displayed.
What specific information do you seek in your history and examination?
1. Assessment prior to posterior fossa craniotomy in a patient with congenital
cardiac disease and intellectual impairment
2. Anaesthetic planning for posterior fossa craniotomy. Recognition and treatment of
venous air embolism
3. Management of post-craniotomy bleeding in recovery with associated atrial
fibrillation
You are part of the trauma team at a metropolitan hospital. You are notified that a 67-year-old
woman is being brought in by road ambulance after a motor vehicle accident.
She was the driver and sole occupant of a car that veered off a country road and down an
embankment. The extraction was complicated, and it is now three hours since her initial
injuries.
On arrival, her airway is patent, and her cervical spine is immobilised. She has two 16G IV
cannulas in situ.
Paramedics hand over that she is confused and has weakness in her left arm and leg.
Prehospital eFAST is positive, with free fluid in the pelvis. She has an open fractured right
tibia and fibula.
Vital signs:
HR 105 bpm (atrial fibrillation)
BP 220/110 mmHg
RR 24 breaths/minute
Sp02 98% (6 L/minute oxygen via mask)
What are your priorities when evaluating her neurological status?
1. Assessment of trauma patient with acute medical comorbidity (CVA)
2. Anaesthetic management of emergency clot retrieval in a trauma patient
3. Recognition and management of a deteriorating trauma patient with signs of
occult bleeding.
You are the anaesthetist on call in a tertiary hospital. It is after-hours and you are still on site.
You are called to the Cardiac Catheterisation Laboratory to assist with the care of a 73-yearold man who is undergoing an urgent coronary angiogram after presenting to the emergency
department an hour earlier with an ST-segment elevation myocardial infarct. His management
was expedited as a “code STEMI”.
You are informed that he was initially stable and neurologically intact on arrival to hospital but
is now hypotensive with a blood pressure of 73/48 mmHg, and is agitated and moving
around.
What information do you seek on arrival at the Cardiac Catheterisation Laboratory?
1. : Assessment and management of an unstable patient in the cardiac catheterisation
lab
2. Provision of safe care to an unstable patient requiring urgent transfer to theatre for emergency CABG
3. Assessment and management of a deteriorating patient immediately post cardiac
surgery
You are the anaesthetist covering the weekday morning emergency list in a tertiary
metropolitan hospital. The emergency medicine physician on duty calls to inform you about a
55-year-old man who has presented with haematemesis. The only known past medical
history is significant alcohol intake and his current medications are unknown.
He is currently in the Emergency Department.
His observations at the time of the call are:
Confide ntial draft
2024.1 Final Examination
HR 100 bpm
NIBP 90/60 mmHg
SpO2 96% on oxygen 6 L/min via Hudson mask
Height 1.7 m
Weight 72 kg
BMI 24.9kg/m2
His blood test results are as displayed.
He has a 20G IV cannula in his left basilic vein and is receiving 1 L crystalloid.
The gastroenterologist plans to perform an emergency endoscopy.
What will you prioritise in your initial assessment and management of this patient?
1. Initial resuscitation and anaesthetic management for a patient with bleeding
varices
2.Management of acute deterioration during endoscopic procedure
3. Management of lung isolation for thoracotomy and oesophageal repair. Post
operative options for management of analgesia.
You are the on-call anaesthetist working at a regional hospital on a Saturday evening.
You are requested to attend the Emergency Department as part of a Trauma Call activation.
The ambulance service has pre-notified of the imminent arrival of a 25-year-old woman who
is 32 weeks pregnant. She was the restrained driver in a car involved in a high-speed collision
with a large tree at an estimated speed of 90 km/h.
Pre-notification information from paramedics:
Confide ntial draft
2024.1 Final Examination
Maintaining her airway
SpO2 stable at 96% on oxygen via a non-rebreathing mask
RR 22 breaths per minute and stable
PR 115 beats per minute and stable
BP 105/60 mmHg and stable
GCS 15 and stable
Complaining of severe abdominal pain
Significant bruising across lower and left upper quadrant of abdomen
The patient is estimated to be 15 minutes away and you have been requested to be the Team
Leader at the Trauma Call.
How would you prepare for the patient’s arrival?
1. - Assessment and initial management of trauma in an obstetric patient
2. - Anaesthetic plan and management for urgent caesarean section. LMA to ETT
conversion
3. Management of hypotension and concealed haemorrhage post induction
You are in the preadmission clinic reviewing a 38-year-old woman with breast cancer whom
you will anaesthetise for bilateral mastectomy and immediate deep inferior epigastric
perforator (DIEP) flap reconstruction in two weeks’ time.
Medical History:
Type 2 diabetes mellitus – diagnosed two years ago
Antiphospholipid syndrome – diagnosed three months ago after first episode of lower limb
deep vein thrombosis with pulmonary embolism
Medications:
Rivaroxaban 20 mg daily
Metformin 500 mg twice daily
Empagliflozin 10 mg daily
Semaglutide 2 mg weekly (subcutaneously)
Examination
Weight 80 kg
Height 160 cm
BMI 31 kg/m2
Examination is otherwise unremarkable
Confide ntial draft
2024.1 Final Examination
Investigations
Full blood examination – normal, with haemoglobin 125 g/dL (120 – 160 g/dL)
Electrolytes, urea and creatinine – normal
Coagulation studies – normal
HbA1c - 5.8% (normal value < 6%)
- 40 (normal value < 42 mmol/mol)
ECG – sinus rhythm
What advice would you give the patient regarding her medications in preparation for surgery?
1. Preoperative optimisation including medication plan
2. Optimise conditions for free flap survival, identify regional anaesthesia options for
bilateral mastectomy/DIEP surgery and arterial blood gas interpretation.
3. Management of gastric content aspiration
A 10-year-old girl has been added to your dental list today at a tertiary hospital. She is
scheduled for dental extractions and treatment under general
anaesthesia.
Medical history
Rheumatic mitral valvular disease
Medications
Frusemide 30 mg BD
Spironolactone 25 mg once daily
Allergies
Nil known
Observations on admission
Height 150 cm
Weight 35 kg
BMI 15 kg/m2
HR 117 bpm
BP 95/60 mmHg
SpO2 96% on room air
In your preoperative review of this patient with her mother, what information
on history and examination will you seek?
1. Anaesthetic management of a paediatric patient with rheumatic mitral regurgitation undergoing
urgent dental procedure under general anaesthesia.
2. Diagnosis and management of intraoperative atrial tachyarrhythmia
3. Management of post-extubation nasal bleeding with associated airway compromise.
It is 11:00 pm and you are on-call at your major metropolitan hospital. The first-year
anaesthetic registrar telephones you at home and asks you to attend for a case that has
been booked with a surgical urgency of one hour. The patient is a 55-year-old woman
who has been booked for a T7 – L3 debridement and laminectomy for spontaneous
epidural abscess.
Observations:
Heart Rate: 130 - 140 bpm, atrial fibrillation
Blood Pressure: 110/60 mmHg on noradrenaline (norepinephrine) infusion running at
0.25 mcg / kg / min (14.5 mcg / min based on estimated lean body weight)
SpO2: 90% on high-flow nasal oxygen 40 L/min
Respiratory Rate: 25 breaths per minute
Temperature: 38.5 °C
GCS: 11 (E2 V4 M5)
Weight: 130 kg
Height: 155 cm
BMI: 57
Past Medical History:
Guillain-Barré Syndrome 20 years previously with ICU admission and mechanical
ventilation via tracheostomy (since closed)
Current smoking, 40-pack-year history
Type 2 diabetes mellitus (on insulin)
Hypertension
The registrar is concerned that the patient may require urgent intubation due to a falling
level of consciousness. You are 20 minutes away from the hospital.
How would you approach the management of this situation?
1. Airway management of a hypoxic, haemodynamically unstable patient in multiorgan failure
secondary to sepsis +/- spinal shock
2. Management of impaired ventilation parameters and hypoxia immediately following the
patient’s transfer to the operating theatre, as well as during the intraoperative period whilst the
patient is in the prone position
3. Management of sudden loss of central and peripheral intravenous access in a patient with
exceptionally difficult vascular access on high-dose vasopressor
You are the duty anaesthetist at a major hospital. You receive a call from the emergency
physician to advise of a 20-year-old woman en route to the hospital with an anterior
chest wall stab wound.
The paramedics report that on their arrival, the patient was hypoxic and agitated. A 15
cm hunting knife was found nearby. The patient was intubated following administration
of diazepam 10 mg and rocuronium 100 mg intravenously. She had bilateral
thoracostomies.
The patient has a known intravenous substance use disorder with heroin.
The last set of vital signs recorded by the paramedics are as follows:
BP 70/55 mmHg
HR 130 bpm
SpO2 92% on FiO2 0.9
RR 15 breaths per minute, ventilated
What are your main considerations as you head down to the Emergency Department?
1. Management of a trauma patient with a penetrating chest wall injury
2. Management of a patient with life-threatening cardiac tamponade, including post-release of
tamponade. Management of the perioperative issues in a patient with penetrating chest injury
with pericardial tamponade.
3. Management of patient with awareness under general anaesthesia
You are the emergency theatre anaesthetist in a tertiary level hospital.
A 68-year-old man with end-stage renal disease is booked for debridement and washout
of an infected brachiocephalic arteriovenous fistula (AVF) +/- fistuloplasty. He has
missed his scheduled haemodialysis today due to
48 hours of progressive swelling and redness in his arm and intermittent chills. A perifistular collection is confirmed on ultrasound.
Medications
Amlodipine-Valsartan 5 mg/ 160 mg once daily
Calcium Carbonate 500 mg TDS
Epoetin alfa 6000 units SCI 3x weekly
Frusemide 160 mg once daily
Hydralazine 25 mg BD
Metoprolol 50 mg BD
Prazosin 1 mg nocte
Observations
Weight 115 kg / Height 1.82 metres /BMI 35 kg/m2
HR 80 bpm
BP 105/47 mmHg
RR 18 breaths per minute
SpO2 96% on room air
Temperature 37.5°C
GCS 15 - lethargic but lucid
Examination Findings
Heart sounds dual
JVP +2 cm
Breath sounds bilateral and vesicular
Bilateral +1 pedal oedema
Outline your clinical priorities in the preoperative assessment of this patient.
1. Preoperative assessment and optimisation of a medically complex patient with End Stage
Renal Disease presenting for a washout of an infected arteriovenous fistula.
2. Clinical discussion regarding the options and relative risks and benefits of regional anaesthesia
for arteriovenous fistula surgery. Management of intraoperative hypotension in the context of a
septic patient at risk of fluid overload.
3. Diagnosis and management of acute pulmonary oedema secondary to fluid overload and a
sympathetic crisis in the PACU.
You are the anaesthetist for a gynaecology list in a standalone day surgical unit. The
next patient is a 15-year-old girl presenting for termination of pregnancy.
She weighs 60 kg and is not on any regular medications. She denies any comorbidities.
She has attended alone due to parental objection to abortion. She is 11 weeks pregnant
confirmed on ultrasound and therefore unsuitable for medical termination.
Describe the elements necessary for informed consent in this patient.
1. Discussion of the elements necessary for informed consent in a well 15-year-old female
presenting for termination of pregnancy.
2. Considerations in the management of excessive pain in a day surgical centre.
3. Management of an immediate emergency laparoscopy complicated by grade 2 anaphylaxis.
You are the anaesthetist at a tertiary referral centre. You have been asked to urgently
review a 40-year-old woman in the emergency department who has presented with neck
swelling and shortness of breath.
She is distressed and anxious.
Medications
Nil regular
Allergies
Nil known drug allergies
Observations
SpO2 98% on room air
RR 20 breaths per minute
HR 120 bpm
BP 130/70 mmHg
Temperature 37°C
Investigations
Full Blood Count:
Results within normal limits
Urea and Electrolytes:
Results within normal limits
Thyroid function tests:
TSH <0.1 mIU/mL (0.5–5 mIU/mL)
Free T4 30 pmol/L (9–19 pmol/L)
Free T3 9.0 pmol/L (3–6.5 pmol/L)
What are the key issues that you would address in your assessment?
1. Assessment and deciding on timing of surgery in a patient with a large retrosternal thyroid
causing airway compromise, as well as hyperthyroidism.
2. Comprehensive airway planning in a patient who refuses awake fibreoptic intubation (AFOI)
and provide details of a plan for extubation.
3. Recognition and management of a post thyroidectomy bleed.
A 77-year-old man is booked for a posterior-approach left total hip joint arthroplasty
(performed in the lateral position) on your list in a private hospital next week. You have
taken over the list on behalf of an unwell colleague who provides their preoperative
notes.
Medical History
Hypertension
Fibromyalgia
Obstructive sleep apnoea on nocturnal CPAP
Gastro-oesophageal reflux disease
Left total knee arthroplasty completed under spinal anaesthesia two years ago
Height 170 cm, Weight 130 kg, BMI 45 kg/m2
Medications
Amlodipine 10 mg once daily
Enalapril 20 mg once daily
Hydrochlorothiazide 50 mg once daily
Omeprazole 20 mg nocte
Naltrexone 4.5 mg once daily
Semaglutide 0.5mg SCI weekly
He has an unfilled prescription from his general practitioner for rivaroxaban 20 mg PO
daily.
Allergies
Nil known drug allergies
An ECG has been performed and is attached below (atrial flutter). A transthoracic echocardiogram will
be performed prior to surgery. What are your concerns about this patient’s presenting cardiac rhythm?
1. Management of pre-operative assessment of an elderly comorbid patient prior to THJR.
Unexpected findings of Atrial Flutter with variable block on ECG. Decision making around
proceeding or delaying.
2. Anaesthetic plan on the day of surgery. The patient elects for spinal and regional block for post
op analgesia (PENG, Fascia iliaca block or High dose local infiltration). The surgery
commences but the patient becomes restless and ultimately requires conversion to GA in the
lateral position.
3. The patient develops tachycardia in PACU. Differential diagnosis and management of a cardiovascularly stable patient with atrial flutter with 2:1 block.
You are the consultant in the high-risk obstetric clinic. Your first patient is a 38-year-old
woman, G3P2, who is 25 weeks pregnant. She has had two previous lower uterine
segment caesarean sections under regional anaesthesia.
Her BMI is 55 kg/m2 (height 174 cm, weight 167 kg). She was diagnosed with severe
obstructive sleep apnoea one year ago and has used nocturnal CPAP since with
improvement in her symptoms.
Medical History
Mild-moderate pulmonary hypertension
Congestive cardiac failure
Cardiorenal syndrome (GFR 41 mL/min)
Obstructive sleep apnoea on nocturnal CPAP
Medications
Aspirin 100 mg once daily
Spironolactone 50 mg mane
Frusemide 40 mg mane, 20 mg nocte
Labetalol 150 mg TDS
Nifedipine SR 30 mg nocte
Enoxaparin 100 mg SC once daily
Observations
HR 90 bpm
BP 130/80 mmHg
RR 22 breaths per minute
SpO2 96% on room air
She has been referred to your tertiary hospital for management of this pregnancy and
planned delivery via repeat lower uterine segment caesarean section
What are the key concerns in assessing this woman for planned LSCS?
1. Morbidly obese 24 weeks’ pregnant woman in High Risk Obstetric Clinic with CCF and cardiorenal syndrome, severe OSA compliant on CPAP, mild-moderate pulmonary hypertension, preeclampsia for planning for Caesarean. Two previous LSCS for pre-eclampsia under regional
and one PPH.
2. Worsening heart failure at 32 weeks, anticoagulated, assessment for LSCS under GA.
3. Patient has post partum haemorrhage. Discussion of appropriate management
including safedisposition in the after-hours period.
You are called to the Emergency Department as part of the trauma team in a tertiary
paediatric hospital.
The paramedics are en route and 5 minutes out from arrival.
You are told that a 2.5-year-old boy is being brought in following a motor vehicle
accident. The handover reports obvious left facial and chest injuries as well as deformity
and swelling of the left upper arm and thigh.
His observations as communicated by the paramedics are
HR 160 bpm
BP 70 mmHg systolic
RR 40 breaths per minute
SpO2 91% on 6L/min via Hudson mask
He responds to voice.
As the anaesthetist in the team, how you will prepare for the child’s arrival?
1. Assessment of a paediatric trauma patient and acute management of hypoxia and
haemodynamic instability in the setting of neurological injury
2. Airway management and anaesthesia to facilitate CT
3. Patient develops severe post-extubation stridor in ICU
You are the anaesthetist covering the intensive care unit and operating theatre in a small
regional hospital. The emergency physician asks for your assistance with transferring a
patient to radiology.
The patient is a 25-year-old man who was found wandering in a worksite.
He was agitated and confused, and an ambulance was called. In the ambulance, the patient
initially complained of a headache, then become drowsy.
The emergency physician has requested a CT brain but is unable to transfer the patient
due to clinical workload.
Medical History
Type 1 diabetes mellitus
Depression
Medications
Insulin pump 70 units Novorapid by subcutaneous infusion every 24 hours
Venlafaxine 150 mg daily
The observations recorded by the paramedics are as follows:
BP 140/85 mmHg
HR 84 bpm sinus rhythm
SpO2 99% on room air
RR 20 breaths per minute
Eye opening: open to voice
Best verbal response: mumbling words inappropriately
Best motor response: localising to pain
How will you assess this patient prior to transfer to radiology?
1. Initial assessment and resuscitation of an undifferentiated neurological problem leading to
intubation to facilitate a brain CT.
2. Recognition and management of raised intracranial pressure during CT scan, initially
manifesting as hypertension.
3. Recognition and management of likely/impending brain death – ongoing haemodynamic
disturbance and polyuria. Support for organ donation.
You are requested to attend a trauma call in the emergency department of a tertiary
hospital.
Paramedics have transferred a 38 year old man with a stab injury to his neck inflicted
during an assault an hour ago.
His GCS is 15 (E4 V5 M6) and has the following initial observations:
Heart rate 95/min
BP 140/90 mmHg
Resp rate 18/min
SpO2 96% on oxygen at 6 LPM via a Hudson mask
A stab wound can be seen about 2 cm above the right clavicle. He has a 20-gauge
cannula in his right antecubital vein and is currently receiving a litre of Hartmann’s
solution. His Hb is 125 g/L on admission.
He does not have any prior medical conditions or allergies and does not take any
medications.
On initial assessment, what clinical features will you focus on?
1. Focused assessment of a patient with a penetrating neck injury
* Differential diagnoses and management of evolving hypotension and hypoxia
* Role of ultrasound in thoracic trauma
2. * Management of sudden haemodynamic collapse following chest drain
* Induction of anaesthesia in a critically unwell patient
* Management of a continuous gas leak from a traumatic bronchopleural fistula
3. Management of analgesia in ICU to facilitate extubation following thoracotomy
* Criteria and suitability for extubation post- thoracotomy
You are tasked with reviewing a 50-year-old male in the preadmission clinic, who is
scheduled for a transjugular intrahepatic portosystemic shunt (TIPS) procedure in one
week.
Past medical history
Cirrhosis secondary to non-alcoholic steatohepatitis leading to previous variceal bleeding
and recurrent ascites.
Type 2 diabetes mellitus.
Medications
Frusemide 40mg once daily
Lactulose 10ml three times daily
Metformin 1g once daily
Pantoprazole 40 mg once daily
Propranolol 10 mg twice daily
Rifaximin 550mg twice daily
Vital signs
Height 160cm
Weight 80 kg
BMI 31 kg/m2
BP 105/ 65 mmHg
HR 85 bpm
SpO2 95% on room air
Temp 36.5 o C
Under what circumstances may a TIPS be unsuitable for a patient with liver failure?
1. Evaluation of a cirrhotic patient in the pre-anaesthesia clinic, focusing on contraindications for a
TIPS procedure and pre-surgical optimization.
2. Diagnosis and management of minor haemorrhage as a complication of TIPS.
3. Assessment and management of a confused patient post TIPS in recovery without intravenous
access.
You are reviewing a 55 Yrs old patient presenting for an elective release of
perioral and anterior neck contracture scars 12 months following a burn injury
involving 60% of her body surface area . The surgical plan will also involve a free
flap to the anterior neck as well as perioral scar release.
Her medications are
Salbutamol MDI 100mcg 2 puffs QID PRN
Salmeterol/Fluticasone MDI 250/25mcg 2 puffs BD
For cultural reasons the patient has requested an all female team for her care in
the operating theatre .How will you
address her request for an all
female team ?
1. Management of a patient who requests an all-female team in theatre for her operation.
Involvement in a multidisciplinary team discussion in the context complex Head and Neck surgery.
2. Assessment and management of a patient for major head and neck with free flap surgery.
3. Management of a patient with a compromised free flap requiring an urgent return to theatre.
You are the duty anaesthetist for a regional hospital. The general surgeons have booked
a 69-year-old male for an urgent ureteric stent insertion at 1900 hrs.
Medical history
Hypertension
Type 2 Diabetes mellitus
Obstructive Sleep Apnoea on CPAP
Bronchial asthma
Currently smokes 10 cigarettes/day with a 50-pack year history
Drinks alcohol socially
Medications
Ramipril 5 mg daily
Empagliflozin 10 mg daily
Gliclazide XR 60 mg daily
Salbutamol inhalers 100 mcg 1-2 puffs q4h PRN
Vitals
Pulse rate 110 bpm
BP – 99/62 mm Hg
SpO2 94% on room air
Temperature 37.8 C
Patient’s body weight is 108 kilograms, height 167 cm
Calculated BMI is 38.7 kg/m2
ECG – sinus tachycardia 110 bpm
CT KUB – right sided hydronephrosis with a 12 mm calculus at the vesicoureteric
junction
What further information would you like to determine the urgency of this case?
1. Preoperative assessment of a sick patient with an endocrine (euglycemic ketoacidosis)
emergency presenting with obstructive urosepsis
2. Perioperative management of a patient in euglycemic ketoacidosis
3. Investigation and management of hypoxaemia and tachypnoea in the Post-Anaesthetic Care
Unit (PACU).
A 17-year-old boy presents to pre-admission clinic in your tertiary referral centre. He
requires assessment prior to a T2- L4 posterior spinal fusion for kyphoscoliosis with a
50 degree curvature.
He has a cervico-thoracic syrinx and had a foramen magnum decompression at 3 years
of age for progressive lower limb weakness.
He is particularly apprehensive about the surgery and is worried about post-operative
pain.
Height 168 cm, Weight 95 kg, BMI 33.6 kg/m2
What do you consider to be the most important features that need to be addressed
during the preoperative assessment?
1. Assessment of patient for major spinal including consideration of relevant investigations.
Discussion of pain management options and formulating an appropriate plan.
2. Discussion of anaesthesia plan specific for major spinal surgery involving intra-operative spinal
cord monitoring. Period of loss of spinal cord signals occurs during the procedure with
consideration of possible causes and immediate management.
3. Postoperative onset of profound right arm sensory impairment and muscle weakness.
Discussion of potential differential diagnoses and appropriate early management.
You are called to review patient in recovery post elective LUSCS under neuraxial
anaesthesia. The patient is increasingly short of breath and has ongoing oxygen
requirement and the nursing staff are concerned about sending her back to the ward.
The lower uterine segment caesarean section (LUSCS) was performed at 36weeks
gestation for intrauterine growth restriction (IUGR). A healthy infant was delivered 2
hours ago.
Biometrics:
Age 28 years
Weight 88kg
Height 160cm
BMI 34.4
Past obstetric and medical history:
G3P0
Systemic Lupus Erythematosus
Medications:
Clexane 40mg od (ceased yesterday)
Aspirin 100mg
Hydroxychloroquine 200mg od
Nil Known Drug allergies
Outline your initial management.
1. Management of an obstetric patient who is short of breath and hypoxic in recovery, following the delivery of a baby via Lower Segment Caesarean Section.
2. Management of cardiovascular collapse in radiology and treatment in a remote environment.
3. Management of tachyarrhythmia, accompanied by a loss of consciousness in ICU